Patho Final Review (ALL)

Réussis tes devoirs et examens dès maintenant avec Quizwiz!

32. A patient taking fluoxetine (Prozac) complains of decreased sexual interest. A prescriber orders a "drug holiday." What teaching by the nurse would best describe a drug holiday? a. "Cut the tablet in half anytime to reduce the dosage." b. "Discontinue the drug for 1 week." c. "Don't take the medication on Friday and Saturday." d. "Take the drug every other day."

c. "Don't take the medication on Friday and Saturday. Sexual dysfunction may be managed by having the patient take a drug holiday, which involves discontinuing medication on Fridays and Saturdays. Cutting the tablet in half anytime to reduce the dosage is an inappropriate way to manage drug administration effectively. In addition, it does not describe a drug holiday. The patient should not take the drug every other day, nor should it be discontinued for a week at a time, because this would diminish the therapeutic levels of the drug, thereby minimizing the therapeutic effects. In addition, neither of those options describe a drug holiday.

What is the purpose of the implementation phase of the nursing process?

-Intended to assist the patient to achieve the stated outcomes for the nursing diagnosis that is being addressed -Should change or affect the specific etiology of that problem -Step of writing the nursing orders, or individualized interventions -It is the heart of the nursing care plan -It is derived from the etiology or R/T from the nursing diagnosis

A patient is admitted to the hospital with unstable angina and will undergo a percutaneous coronary intervention. Which drug regimen will the nurse expect to administer to prevent thrombosis in this patient? A) Aspirin, clopidogrel, omeprazole B) Aspirin, heparin, abciximab (ReoPro) C) Enoxaparin (Lovenox), prasugrel (Effient), warfarin (Coumadin) D) Heparin, alteplase, abciximab (ReoPro)

B) Aspirin, heparin, abciximab (ReoPro)

A postoperative patient reports pain in the left lower extremity. The nurse notes swelling in the lower leg, which feels warm to touch. The nurse will anticipate giving which medication? A) Aspirin B) Clopidogrel (Piavix) C) Enoxaparin (Lovenox) D) Warfarin (Coumadin)

C) Enoxaparin (Lovenox)

patient using Afrin nasal spray complains of worsening cold symptoms and tells the nurse, "I don't understand why this is not working. I am using it almost every 3 hours!" The nurse's response is based on knowledge that

) the patient is suffering from rebound congestion related to excessive use of the Afrin nasal spray.

In patients with severe bone marrow supression, the nurse be knowledgeable of what principal early signs of infection?

* fever- greater than or equal to 100.5 degrees F * Chills or shakes * Sudden onset of unexplained pain

A patient has been taking donepezil (Aricept) for 2 weeks as part of the treatment for early stages of Alzheimer's disease. Her daughter calls the physician's office and is upset because "Mother has not improved one bit!" Which response by the nurse is appropriate?

. "It may take up to 6 weeks to see an improvement."

75-year-old woman comes into the clinic with the complaint of indigestion, stomach pain, and frequent belching. She tells the nurse that she has been taking sodium bicarbonate 5 or 6 times a day for the past 3 weeks. The nurse knows that which possible hazard exists with the use of sodium bicarbonate?

. Metabolic alkalosis may result from excessive use.

The nurse is administering parenteral drugs. Which statement is true regarding parenteral drugs? Choose one answer.

. Parenteral drugs bypass the first-pass effect.

A patient with type 1 diabetes reports mixing NPH and regular insulin to allow for one injection. What should the nurse tell the patient? A) This is an acceptable practice. B) These two forms of insulin are not compatible and cannot be mixed. C) Mixing these two forms of insulin may Increase the overall potency of the products. D) NPH insulin should only be mixed with insulin glargine.

A) This is an acceptable practice.

A patient is beginning therapy with oral methotrexate (Rheumatrex) for rheumatoid arthritis. The nurse will teach this patient about the importance of: A) having routine renal and hepatic function tests. B) limiting folic acid consumption. C) reporting alopecia and rash. D) taking the medication on a daily basis.

A) having routine renal and hepatic function tests.

The peripheral parenteral nutrition bag that has been infusing into an assigned patient has run dry before a new bag has been ordered. The nurse should immediately hang with of the following available solutions until the new bag arrives?

10% dextrose in water

A nursing student is discussing with a nurse the plan of care for a patient about to undergo a third round of chemotherapy with cisplatin. Which statement by the nursing student about the treatment of CINV is correct? A) "Aprepitant (Emend) will be necessary to treat CINV caused by cisplatin." B) "Antiemetics are most effective if given just as the chemotherapy is finished." C) "Lorazepam probably would not be helpful for this patient." D) "This patient will need intravenous antiemetics for best effects."

A) "Aprepitant (Emend) will be necessary to treat CINV caused by cisplatin."

A nurse is giving aspirin to a patient during acute management of STEMI. The patient asks why a chewable tablet is given. Which response by the nurse is correct? A) "Aspirin is absorbed more quickly when it is chewed." B) "Chewing aspirin prevents it from being metabolized by the liver." C) "Chewing aspirin prevents stomach irritation." D) "More of the drug is absorbed when aspirin is chewed."

A) "Aspirin is absorbed more quickly when it is chewed."

A nurse is discussing how beta blockers work to decrease blood pressure with a nursing student. Which statement by the student indicates a need for further teaching? A) "Beta blockers block the actions of angiotensin II." B) "Beta blockers decrease heart rate and contractility." C) "Beta blockers decrease peripheral vascular resistance." D) "Beta blockers decrease the release of renin."

A) "Beta blockers block the actions of angiotensin II."

A patient is taking a calcium channel blocker (CCB) for stable angina. The patient's spouse asks how calcium channel blockers relieve pain. The nurse will explain that CCBs: A) help relax peripheral arterioles to reduce afterload. B) Improve coronary artery perfusion. C) increase the heart rate to improve myocardial contractility. D) increase the QT interval.

A) help relax peripheral arterioles to reduce afterload.

what should a nurse hang if a parenteral nutritional bag runs dry and a new bag has not been ordered?

5 to 10% dextrose, cause rebound hypoglycemia can occur

A nurse is discussing the administration of an intravenous infusion of rituximab (Rituxan) with a nursing student. Which statement by the student indicates a need for further education about the care of a patient receiving this drug? A) "Angioedema and hypersensitivity may occur, but they are usually self-limiting and mild." B) "I should be prepared to administer epinephrine, glucocorticoids, and oxygen if needed." C) "I will administer an antihistamine and acetaminophen before beginning the infusion." D) "I will monitor this patient's blood pressure, respiratory rate, and oxygen saturation closely."

A) "Angioedema and hypersensitivity may occur, but they are usually self-limiting and mild."

A nurse is teaching a group of nursing students about antihistamines. Which statement by a student indicates an understanding of the mechanism of action of the antihistamines? A) "Antihistamines block H, receptors to prevent actions of histamine at these sites." B) "Antihistamines block release of histamine from mast cells and basophiis." C) "H1antagonists can bind to H1 receptors, H2 receptors, and muscarinic receptors." D) "First-generation antihistamines are more selective than second-generation antihistamines."

A) "Antihistamines block H, receptors to prevent actions of histamine at these sites."

When would a patient receive folic acid treatment that is newly diagnosed with anemia?

A patient must be determined anemic before starting folic acid treatment. and it should be emphasized that folic acid should not be used to treat anemias until the underlying cause and type of anemia have been determined.q

A patient who has been taking an antihypertensive medication for several years is recovering from a myocardial infarction. The prescriber changes the patient's medication to an ACE inhibitor. The patient asks the nurse why a new drug is necessary. What is the nurse's response? A) "ACE inhibitors can prevent or reverse pathologic changes in the heart's structure." B) "ACE inhibitors help lower LDL cholesterol and raise HDL cholesterol." C) "ACE inhibitors increase venous return to the heart, improving cardiac output." D) "ACE inhibitors regulate electrolytes that affect the cardiac rhythm."

A) "ACE inhibitors can prevent or reverse pathologic changes in the heart's structure."

A patient who has chronic allergies takes loratadine (Ciaritin) and develops a severe reaction to beestings. The patient asks the nurse why the antihistamine did not prevent the reaction. What will the nurse say? A) "Allergy symptoms that are severe are caused by mediators other than histamine." B) "H1 blockers do not prevent the release of histamine from mast cells and basophils." C) "Second-generation H1 blockers contain less active drug and do not work in severe reactions." D) "Severe allergic reactions occur through actions on muscarinic receptors."

A) "Allergy symptoms that are severe are caused by mediators other than histamine."

A patient who reports regular consumption of two or three alcoholic beverages per day asks about taking acetaminophen when needed for occasional, recurrent pain. What will the nurse tell the patient? A) "Do not take more than 2 gm of acetaminophen a day." B) "Do not take more than 3 gm of acetaminophen a day." C) "Do not take more than 4 gm of acetaminophen a day." D) "Do not take a fixed-dose preparation with opioid analgesics."

A) "Do not take more than 2 gm of acetaminophen a day."

A nurse explains to a nursing student why opiold antidiarrheal medications are classified as drugs with little or no abuse potential. Which statement by the student indicates·a need for further teaching? A) "Formulations for the treatment of diarrhea have very short half-lives." B) "Opioid antidiarrheal drugs contain other drugs with unpleasant side effects at higher doses." C) "Some opioid antidiarrheal drugs do not cross the blood-brain barrier." D) "Some opioid antidiarrheal medications are not water soluble and cannot be given parenterally."

A) "Formulations for the treatment of diarrhea have very short half-lives."

A nurse is discussing glucocorticoids with a group of nursing students. Which statement by a student indicates understanding of the teaching? A) "Giucocorticoids have both endocrine and nonendocrine uses." B) "Patients treated for adrenocortical insufficiency receive pharmacologic doses." C) "Pharmacologic effects are achieved with low doses of glucocorticoids." D) "Physiologic doses are used to treat inflammatory disorders."

A) "Giucocorticoids have both endocrine and nonendocrine uses."

A nurse is teaching a patient who will begin taking verapamil (Calan) for hypertension about the drug's side effects. Which statement by the patient indicates understanding of the teaching? A) "I may become constipated, so Ishould increase fluids and fiber." B) "I may experience a rapid heart rate as a result of taking this drug." C) "I may have swelling of my hands and feet, but this will subside." D) "I may need to increase my digoxin dose while taking this drug."

A) "I may become constipated, so Ishould increase fluids and fiber."

A nurse is providing teaching for an adult patient with arthritis who has been instructed to take ibuprofen (Motrin) for discomfort. Which statement by the patient indicates a need for further teaching? A) "I may experience tinnitus with higher doses of this medication." B) "I may take up to 800 mg 4 times daily for pain." C) "I should limit alcohol intake to fewer than three drinks a day." D) "I will take this medication with meals to help prevent stomach upset."

A) "I may experience tinnitus with higher doses of this medication."

A 7-year-old child with asthma uses a daily inhaled glucocorticoid and an albuterol MDI as needed. The provider has added montelukast (Singulair) to the child's regimen. Which statement by the child's parent indicates understanding of this medication? A) "I may notice mood changes in my child." B) "I should give this medication twice daily." C) "I will give my child one 4-mg chewable tablet daily." D) "This drug can alleviate symptoms during an acute attack."

A) "I may notice mood changes in my child."

A nurse is teaching a patient who has taken glucocorticoids for over a year about glucocorticoid withdrawal. Which statement by the patient indicates a need for further teaching? A) "I should reduce the dose by half each day until Istop taking the drug." B) "I will need to have cortisol levels monitored during the withdrawal process." C) "The withdrawal schedule may take several months." D) "If Ihave surgery, Imay need to take the drug for a while, even after Ihave stopped."

A) "I should reduce the dose by half each day until Istop taking the drug."

A patient with a history of angina and hypertension is being started on nicotinic acid (Niacin). The nurse is providing patient education. What statement made by the patient demonstrates a need for further teaching? A) "I will take Tylenol with my medication to reduce the inflammatory effects." B) "I will be cautious taking this medication, because I have mild liver damage." C) "I will take an aspirin 30 minutes before my niacin to reduce flushing." D) "This medication will lower my triglyceride levels."

A) "I will take Tylenol with my medication to reduce the inflammatory effects."

A nursing student is helping to care for a patient who takes verapamil for stable angina. The nurse asks the student to explain the purpose of verapamil in the treatment of this patient. Which statement by the student indicates a need for further teaching? A) "It relaxes coronary artery spasms." B) "It reduces peripheral resistance to reduce oxygen demands." C) "It reduces the heart rate, AV conduction, and contractility." D) "It relaxes the peripheral arterioles to reduce afterload."

A) "It relaxes coronary artery spasms."

A nurse is discussing methicillin-resistant Staphylococcus aureus (MRSA) with a group of nursing students. Which statement by a student correctly identifies the basis for MRSA resistance? A) "MRSA bacteria have developed PBPs with a low affinity for penicillins." B) "MRSA bacteria produce penicillinases that render penicillin ineffective." C) "MRSA occurs because of host resistance to penicillins." D) "MRSA strains replicate faster than other Staphylococcus aureus strains."

A) "MRSA bacteria have developed PBPs with a low affinity for penicillins."

Lovastatin (Mevacor) is prescribed for a patient for the first time. The nurse should provide the patient with which instruction? A) "Take lovastatin with your evening meal." B) "Take this medicine before breakfast." C) "You may take lovastatin without regard to meals." D) "Take this medicine on an empty stomach."

A) "Take lovastatin with your evening meal."

The nurse is providing education to a patient who has been prescribed both an antacid and ranitidine (Zantac). Which instruction should the nurse give the patient about taking the medications? A) "Take the antacid 1hour after the ranitidine." B) "The antacid and ranitidine should be taken at the same time for better effect." C) "Take the antacid 15 minutes before the ranitidine." D) "Take the antacid 30 minutes after the ranitidine."

A) "Take the antacid 1hour after the ranitidine."

A pregnant patient with fever, flank pain, and chills has a history of two previous bladder infections before getting pregnant. She is allergic to several antibiotics. She reports having taken methenamine successfully in the past. What will the nurse tell her? A) "This agent is not effective against infections of the upper urinary tract." B) "This antiseptic agent is safe for use during pregnancy and has no drug resistance." C) "This drug is linked to many serious birth defects and is not recommended during pregnancy." D) "You will need to take this medication with meals to avoid gastric upset."

A) "This agent is not effective against infections of the upper urinary tract."

A patient has heart failure and is taking an ACE inhibitor. The patient has developed fibrotic changes in the heart and vessels. The nurse expects the provider to order which medication to counter this development? A) Aldosterone antagonist B) Angiotensin IIreceptor blocker (ARB) C) Beta blocker D) Direct renin inhibitor (DRI)

A) Aldosterone antagonist

An adolescent patient recently attended a health fair and had a serum glucose test. The patient telephones the nurse and says, "My level was 125 mg/dl. Does that mean Ihave diabetes?" What is the nurse's most accurate response? A) "Unless you were fasting for longer than 8 hours, this does not necessarily mean you have diabetes." B) "At this level, you probably have diabetes. You will need an oral glucose tolerance test this week." C) "This level is conclusive evidence that you have diabetes." D) "This level is conclusive evidence that you do not have diabetes."

A) "Unless you were fasting for longer than 8 hours, this does not necessarily mean you have diabetes."

A patient has just received a prescription for fluticasone/salmeterol (Advair Diskus). What will the nurse include as part of the teaching for this patient about the use of this device? A) "You do not need good hand-lung coordination to use this device." B) "You will begin to inhale before activating the device." C) "You will need to use a spacer to help control the medication." D) "You will take 2 inhalations twice daily."

A) "You do not need good hand-lung coordination to use this device."

The prescriber orders 20 mg of hydrocortisone orally once each day. The nurse will make sure the drug is scheduled to be administered at what time? A) 8:00AM B) 12:00 PM C) 4:00 PM D) 9:00 PM

A) 8:00AM

A patient with diabetes develops hypertension. The nurse will anticipate administering which type of medication to treat hypertension in this patient? A) ACE inhibitors B) Beta blockers C) Direct-acting vasodilators D) Thiazide diuretics

A) ACE inhibitors

A diabetic patient is recovering from a myocardial infarction but does not have symptoms of heart failure. The nurse will expect to teach this patient about: A) ACE inhibitors and beta blockers. B) biventricular pacemakers. C) dietary supplements and exercise. D) diuretics and digoxin.

A) ACE inhibitors and beta blockers.

A nurse is caring for a patient who is receiving verapamil (Calan) for hypertension and digoxin (Lanoxin) for heart failure. The nurse will observe this patient for: A) AV blockade. B) gingival hyperplasia. C) migraine headaches. D) reflex tachycardia.

A) AV blockade.

A patient will be discharged home to complete treatment with intravenous cefotetan with the assistance of a home nurse. The home care nurse will include which instruction when teaching the patient about this drug treatment? A) Abstain from alcohol consumption during therapy. B) Avoid dairy products while taking this drug. C) Take an antihistamine if a rash occurs. D) Use nonsteroidal anti-inflammatory drugs (NSAIDs), not acetaminophen, for pain.

A) Abstain from alcohol consumption during therapy.

An adolescent is brought to the emergency department by a parent who reports that the patient took a whole bottle of extended-release acetaminophen tablets somewhere between 8 and 10 hours ago. The nurse will anticipate administering which of the following? A) Acetylcysteine (Mucomyst) B) Activated charcoal C) Hemodialysis D) Respiratory support

A) Acetylcysteine (Mucomyst)

A prescriber has ordered cefoxltin for a patient who has an infection caused by a gram-negative bacteria. The nurse taking the medication history learns that the patient experienced a maculopapular rash when taking amoxicillln (Amoxil) several years earlier. What will the nurse do? A) Administer the cefoxitin and observe for any side effects. B) Give the cefoxitin and have epinephrine and respiratory support available. C) Request an order for a different, nonpenicillin, noncephalosporin antibiotic. D) Request an order to administer a skin test before giving the cefoxitin.

A) Administer the cefoxitin and observe for any side effects.

A patient with stable exertlonal angina has been receiving a beta blocker. Before giving the drug, the nurse notes a resting heart rate of 55 beats per minute. Which is an appropriate nursing action? A) Administer the drug as ordered, because this is a desired effect. B) Withhold the dose and notify the provider of the heart rate. C) Request an order for a lower dose of the medication. D) Request an order to change to another antianginal medication.

A) Administer the drug as ordered, because this is a desired effect.

A patient with renal failure is undergoing chronic hemodialysis. The patient's hemoglobin is 10.6 gm/dL. The provider orders sodium-ferric gluconate complex (SFGC [Ferrlecit]). What will the nurse expect to do? A) Administer the drug intravenously with erythropoietin. B) Give a test dose before each administration of the drug. C) Have epinephrine on hand to treat anaphylaxis if needed. D) Infuse the drug rapidly to achieve maximum effects quickly.

A) Administer the drug intravenously with erythropoietin.

A provider has ordered ceftriaxone 4 gm once daily for a patient with renal impairment. What will the nurse do? A) Administer the medication as prescribed. B) Contact the provider to ask about giving the drug in divided doses. C) Discuss increasing the interval between doses with the provider. D) Discuss reducing the dose with the provider.

A) Administer the medication as prescribed.

The patient with which of the following is most at risk for folic acid deficiency? A) Alcoholism B) Sprue C) Gastrectomy D) Peptic ulcer disease

A) Alcoholism

A client is given tamsulosin (Flomax) for benign prostatic hyperplasia (BPH). The nurse knows that the action of this drug is due to selective blockade of which one of the following receptors?

A) Alpha-1

A patient is diagnosed with peptic ulcer disease. The patient is otherwise healthy. The nurse learns that the patient does not smoke and that he drinks 1or 2 glasses of wine with meals each week. The nurse anticipates that the provider will prescribe which drugs? A) Amoxicillin (Amoxil), clarithromycin, and omeprazole (Prilosec) B) Amoxicillin (Amoxil), metronidazole (Fiagyl), and cimetidine (Tagamet), C) Clarlthromycin, metronidazole (Fiagyl), and omeprazole (Prilosec) D) Tetracycline, cimetidine (Tagamet), and lansoprazole (Prevacid)

A) Amoxicillin (Amoxil), clarithromycin, and omeprazole (Prilosec)

A pregnant patient asks the nurse if she can take antihistamines for seasonal allergies during her pregnancy. What will the nurse tell the patient? A) Antihistamines are contraindicated during the third trimester of pregnancy. B) Second-generation antihistamines are safer than first-generation antihistamines. C) Antihistamines should not be taken during pregnancy, but may be taken when breast-feeding. D) The margin of safety for antihistamines is clearly understood for pregnant patients.

A) Antihistamines are contraindicated during the third trimester of pregnancy.

A nurse is taking a history on a clinic patient who reports being constipated. Upon further questioning, the nurse learns that the patient's last stool was 4 days ago, that it was of normal, soft consistency, and the patient defecated without straining. The patient's abdomen is not distended, and bowel sounds are present. The patient reports usually having a stool every 1to 2 days. What will the nurse do? A) Ask about recent food and fluid intake. B) Discuss the use of polyethylene glycol (MiraLax). C) Recommend a bulk laxative. D) Suggest using a bisacodyl (Dulcolax) suppository.

A) Ask about recent food and fluid intake.

A patient taking a glucocorticoid for arthritis reports feeling bloated; The nurse notes edema of the patient's hands and feet. Which action by the nurse is correct? A) Ask the patient about sodium intake. B) Obtain a blood glucose level. C) Suggest the patient limit potassium intake. D) Tell the patient to stop taking the drug.

A) Ask the patient about sodium intake.

A patient has been taking levothyroxine for several years and reports that "for the past 2 weeks, the drug doesn't seem to work as well as before." What will the nurse do? A) Ask the patient when the prescription was last refilled. B) Expect the patient to have an elevated temperature and tachycardia. C) Suggest that the patient begin taking calcium supplements. D) Tell the patient to try taking the medication with food.

A) Ask the patient when the prescription was last refilled.

A patient who uses an inhaled glucocorticoid for chronic asthma calls the nurse to report hoarseness. What will the nurse do? A) Ask whether the patient is gargling after each dose of the glucocorticoid. B) Request an order for an antifungal medication. C) Suggest that the patient be tested for a bronchial infection. D) Tell the patient to discontinue use of the glucocorticoid.

A) Ask whether the patient is gargling after each dose of the glucocorticoid.

A patient arrives in the emergency department complaining of chest pain that has lasted longer than 1 hour and is unrelieved by nitroglycerin. The patient's electrocardiogram reveals elevation of the ST segment. Initial cardiac troponin levels are negative. The patient is receiving oxygen via nasal cannula. Which drug should be given immediately? A) Aspirin 325 mg chewable B) Beta blocker given IV C) Ibuprofen 400 mg orally D) Morphine intravenously

A) Aspirin 325 mg chewable

When metronidazole (Fiagyl) is a component of the H. pylori treatment regimen, the patient must be instructed to do what? A) Avoid any alcoholic beverages B) Avoid foods containing tyramine C) Take the drug on an empty stomach D) Take the drug with food

A) Avoid any alcoholic beverages

A patient who is in her first trimester of pregnancy asks the nurse to recommend nonpharmaceuticai therapies for morning sickness. What will the nurse suggest? A) Avoiding fatty and spicy foods B) Consuming extra clear fluids C) Eating three meals daily D) Taking foods later in the day

A) Avoiding fatty and spicy foods

The potassium-sparing diuretic spironolactone (Aidactone) prolongs survival and improves heart failure symptoms by which mechanism? A) Blocking aldosterone receptors B) Increasing diuresis C) Reducing venous pressure D) Reducing afterload

A) Blocking aldosterone receptors

A male patient with hepatitis C will begin triple drug therapy with pegylated interferon alfa 2a (Pegasys), ribavirin (Ribasphere), and boceprevir (Victrelis). The patient tells the nurse that his wife is pregnant. What will the nurse tell him? A) Boceprevir is contraindicated in males whose partners are pregnant. B) He should use a barrier contraceptive when having sex. C) He should use dual drug therapy with pegylated interferon alfa and ribavirin only. D) This combination drug therapy is safe for him to use.

A) Boceprevir is contraindicated in males whose partners are pregnant.

The nurse is caring for an older adult patient after a right hip open reduction internal fixation (ORIF). The patient is taking an opioid every 6 hours as needed for pain. The nurse discusses obtaining an order from the prescriber for which medication? A) Docusate sodium (Colace) B) GoLYTELY C) Lactulose D) Polyethylene glycol (Miralax)

A) Docusate sodium (Colace)

A 5-year-old child with seasonal allergies has been taking 2.5 rill of cetirizine (Zyrtec) syrup once daily. The parents tell the nurse that the child does not like the syrup, and they do not think that the drug is effective. The nurse will suggest they discuss which drug with their child's healthcare provider? A) Cetirizine (Zyrtec) 5 mg chewable tablet once daily B) Loratadine (Ciaritin) 10 mg chewable tablet once daily C) Fexofenadine (Allegra) syrup 5 ml twice daily D) Desloratadine (Ciarinex) 5 mg rapid-disintegrating tablet once daily

A) Cetirizine (Zyrtec) 5 mg chewable tablet once daily

A patient has been taking digoxin (Lanoxin) 0.25 mg, and furosemide (Lasix) 40 mg, daily. Upon routine assessment by the nurse, the patient states, "I see yellow halos around the lights." The nurse should perform which action based on this assessment? A) Check the patient for other symptoms of digitalis toxicity. B) Withhold the next dose of furosemide. C) Continue to monitor the patient for heart failure. D) Document the findings and reassess in 1hour.

A) Check the patient for other symptoms of digitalis toxicity.

Which condition would cause the nurse to withhold a PRN order for magnesium hydroxide? A) Chronic renal failure B) Cirrhosis C) Hemorrhoids D) Prostatitis

A) Chronic renal failure

A patient with hypertension who has been taking captopril (Capoten) for several months is admitted to the hospital. The patient reports that food "tastes funny." What will the nurse do? A) Compare the patient's admission weight to a previous weight. B) Notify the provider to request a different antihypertensive medication. C) Reassure the patient that this is a temporary side effect of the drug. D) Request an order for a white blood cell count (WBC) with differential.

A) Compare the patient's admission weight to a previous weight.

A patient with no known drug allergies is receiving amoxicillin (Amoxil) PO twice daily. Twenty minutes after being given a dose, the patient complains of shortness of breath. The patient's blood pressure is 100/58 mm Hg. What will the nurse do? A) Contact the provider and prepare to administer epinephrine. B) Notify the provider if the patient develops a rash. C) Request an order for a skin test to evaluate possible PCN allergy. D) Withhold the next dose until symptoms subside.

A) Contact the provider and prepare to administer epinephrine.

A patient taking high doses of a glucocorticoid develops weakness in the muscles of the upper arms and in the legs. What will the nurse do? A) Contact the provider to ask about reducing the dose. B) Encourage the patient to restrict sodium intake. C) Reassure the patient that this is an expected side effect. D) Tell the patient to stop taking the drug.

A) Contact the provider to ask about reducing the dose.

A patient with angina who uses sublingual nitroglycerin tells the nurse that the episodes are increasing in frequency and usually occur when the patient walks the dog. The patient reports needing almost daily doses of the nitroglycerin and states that one tablet usually provides complete relief. What will the nurse do? A) Contact the provider to suggest ordering a transdermal patch for this patient. B) Question the patient about consumption of grapefruit juice. C) Suggest that the patient limit walking the dog to shorter distances less frequently. D) Suggest that the patient take two tablets of nitroglycerin each time, because the symptoms are increasing in frequency.

A) Contact the provider to suggest ordering a transdermal patch for this patient.

A patient who has been newly diagnosed with asthma is referred to an asthma clinic. The patient reports daily symptoms requiring short-acting beta,-agonist treatments for relief. The patient has used oral glucocorticoids three times in the past 3 months and reports awakening at night with symptoms about once a week. The patient's forced expiratory volume in 1 second (FEV1) is 75% of predicted values. The nurse will expect this patient to be started on which regimen? A) Daily low-dose inhaled glucocorticoid/LABA with a SABA as needed B) Daily low-dose inhaled glucocorticoid and a SABA as needed C) Daily medium-dose inhaled glucocorticoid/LABA combination D) No daily medications; just a SABA as needed

A) Daily low-dose inhaled glucocorticoid/LABA with a SABA as needed

The nurse is caring for a patient who is taking a vasodilator that dilates capacitance vessels. The nurse will expect which effect in this patient? A) Decrease in cardiac work B) Increase in cardiac output C) Increase in tissue perfusion D) Increase in venous return

A) Decrease in cardiac work

Before giving methenamine (Hiprex) to a patient, it is important for the nurse to review the patient's history for evidence of which problem? A) Elevated blood urea nitrogen and creatinine B) History of reactions to antibiotic agents C) Possibility of pregnancy D) Previous resistance to antiseptic agents

A) Elevated blood urea nitrogen and creatinine

An older adult patient with congestive heart failure develops crackles in both lungs and pitting edema of all extremities. The physician orders hydrochlorothiazide (HydroDIURIL). Before administering this medication, the nurse reviews the patient's chart. Which laboratory value causes the nurse the most concern? A) Elevated creatinine clearance B) Elevated serum potassium level C) Normal blood glucose level D) Low levels of low-density lipoprotein (LDL) cholesterol

A) Elevated creatinine clearance

A nurse is obtaining a medication history on a newly admitted patient, who reports taking minoxidil for hypertension. Admission vital signs reveal a heart rate of 78 beats per minute and a blood pressure of 120/80 mm Hg. What is an important part of the initial assessment for this patient? A) Evaluating ankle edema B) Monitoring for nausea and vomiting C) Noting the presence of hypertrichosis D) Obtaining a blood glucose

A) Evaluating ankle edema

The nurse assesses a newly diagnosed patient for short-term complications of diabetes. What does this assessment include? A) Evaluation for hyperglycemia, hypoglycemia, and ketoacidosis B) Cranial nerve testing for peripheral neuropathy C) Pedal pulse palpation for arterial insufficiency D) Auscultation of the carotids for bruits associated with atherosclerosis

A) Evaluation for hyperglycemia, hypoglycemia, and ketoacidosis

A patient is to undergo orthopedic surgery, and the prescriber will order a cephalosporin to be given preoperatively as prophylaxis against infection. The nurse expects the provider to order which cephalosporin? A) First-generation cephalosporin B) Second-generation cephalosporin C) Third-generation cephalosporin D) Fourth-generation cephalosporin

A) First-generation cephalosporin

A patient is brought to the emergency department with shortness of breath, a respiratory rate of 30 breaths per minute, intercostal retractions, and frothy, pink sputum. The nurse caring for this patient will expect to administer which drug? A) Furosemide (Lasix) B) Hydrochlorothiazide (HydroDIURIL) C) Mannitol (Osmitrol) D) Spironolactone (Aidactone)

A) Furosemide (Lasix)

A nurse transcribes a new prescription for potassium penicillin G given intravenously (IV) every 8 hours and gentamicin given IV every 12 hours. Which is the best schedule for administering these drugs? A) Give the penicillin at 0800, 1600, and 2400; give the gentamicin (Garamycin) at 1800 and 0600. B) Give the penicillin at 0800, 1600, and 2400; give the gentamicin (Garamycin) at 1200 and 2400. C) Give the penicillin at 0600, 1400, and 2200; give the gentamicin (Garamycin) at 0600 and 1800. D) Give the penicillin every 8 hours; give the gentamicin (Garamycin) simultaneously with two of the penicillin doses.

A) Give the penicillin at 0800, 1600, and 2400; give the gentamicin (Garamycin) at 1800 and 0600.

A parent asks a nurse about growth suppression resulting from the use of an inhaled glucocorticoid in children. What will the nurse tell the parent? A) Growth may be slowed, but eventual adult height·will not be reduced. B) The growth rate is not impaired, but overall height will be reduced. C) The growth rate slows while the drug is used but resumes when the drug is stopped. D) Long-term use of the drug results in a decrease in adult height.

A) Growth may be slowed, but eventual adult height·will not be reduced.

Your patient is receiving dobutamine as a continuous infusion. Titration of this medication is based upon which factors? (Select all that apply.)

A) Heart rate B) Blood pressure C) Urine output

Which two-drug regimen woulq pe appropriate for a patient with hypertension who does not have other compelling conditions? A) Hydrochlorothiazide and nadolol B) Hydralazine and minoxidil C) Spironolactone and amiloride D) Trichlormethiazide and hydrochlorothiazide

A) Hydrochlorothiazide and nadolol

A patient has three separate blood pressure (BP) readings of 120/100 mm Hg, 138/92 mm Hg, and 126/96 mm Hg. Which category describes this patient's BP? A) Hypertension B) Isolated systolic hypertension C) Normal D) Prehypertension

A) Hypertension

A patient with vitamin B12 deficiency is admitted with symptoms of hypoxia, anemia, numbness of hands and feet, and oral stomatitis. The nurse expects the prescriber to order which of the following therapies? A) IM cyanocobalamin and folic acid B) IM cyanocobalamin and antibiotics C) PO cyanocobalamin and folic acid D) PO cyanocobalamin and blood transfusions

A) IM cyanocobalamin and folic acid

A patient with moderate heart failure begins taking a thiazide dil.lretic. The nurse will tell the patient to expect which outcome when taking this drug? A) Improved exercise tolerance B) Increased cardiac output C) Prevention of cardiac remodeling D) Prolonged survival

A) Improved exercise tolerance

A nurse administers an ACE inhibitor to a patient who is taking the drug for the first time. What will the nurse do? A) Instruct the patient not to get up without assistance. B) Make sure the patient takes a potassium supplement. C) Report the presence of a dry cough to the prescriber. D) Request an order for a diuretic to counter the side effects of the ACE inhibitor.

A) Instruct the patient not to get up without assistance.

A patient has an infection caused by Pseudomonas aeruginosa. The prescriber has ordered piperacillin and amikacin, both to be given intravenously. What will the nurse do? A) Make sure to administer the drugs at different times using different IV tubing. B) Suggest giving larger doses of plperacillin and discontinuing the amikacin. C) Suggest that a fixed-dose combination of piperacillin and tazobactam (Zosyn) be used. D) Watch the patient closely for allergic reactions, because this risk Is increased With this combination.

A) Make sure to administer the drugs at different times using different IV tubing.

Which statement is correct about the contrast between acarbose and mlglitol? A) Miglitol has not been associated with hepatic dysfunction. B) With miglitol, sucrose can be used to treat hypoglycemia. C) Miglitol is less effective in African Americans. D) Miglitol has no gastrointestinal side effects.

A) Miglitol has not been associated with hepatic dysfunction.

A patient with hypertension with a blood pressure of 168/110 mm Hg begins taking hydrochlorothiazide and verapamil. The patient returns to the clinic after 2 weeks of drug therapy,, and the nurse notes a blood pressure of 140/85 mm Hg and a heart rate of 98 beats per minute. What will the nurse do? A) Notify the provider and ask about adding a beta blocker medication. B) Reassure the patient that the medications are working. C) Remind the patient to move slowly from sitting to standing. D) Request an order for an electrocardiogram.

A) Notify the provider and ask about adding a beta blocker medication.

A hospitalized patient has a blood pressure of 145/96 mm Hg. The nurse caring for this patient notes that the blood pressure the day before was 132/98 mm Hg. The patient reports ambulatory blood pressure readings of 136/98 and 138/92 mm Hg. The patient has a history of a previous myocardial infarction and has adopted a lifestyle that includes use of the DASH diet and regular exercise. What will the nurse do? A) Notify the provider and discuss ordering a beta blocker for this patient. B) Notify the provider and suggest a thiazide diuretic as initial therapy. C) Order a diet low in sodium and high in potassium for this patient. D) Recheck the patient's blood pressure in 4 hours to verify the result.

A) Notify the provider and discuss ordering a beta blocker for this patient.

A patient with HIV and mucocutaneous HSV is being treated with foscarnet after failing treatment with acyclovir. After 2 weeks, the patient's dose is increased to 90 mg/kg over 2 hours from 40 mg/kg over 1 hour. The patient reports numbness in the extremities and perioral tingling. What will the nurse do? A) Notify the provider and request an order for a serum calcium level. B) Notify the provider of potential foscarnet overdose. C) Request an order for a creatinine clearance level. D) Request an order of IV saline to be given before the next dose.

A) Notify the provider and request an order for a serum calcium level.

A patient has undergone liver transplantation. The provider orders cyclosporine (Sandimmune), prednisone, and sirolimus (Rapamune). What will the nurse do? A) Question the order for sirolimus. B) Request an order for a serum glucose level. C) Request an order for a macrolide antibiotic. D) Suggest changing the cyclosporine to tacrolimus.

A) Question the order for sirolimus.

A patient is receiving oral iron for iron deficiency anemia. Which antibiotic drug, taken concurrently with iron, would most concern the nurse? A) Tetracycline B) Cephalosporin C) Metronidazole (Flagyl) D) Penicillin

A) Tetracycline

A patient will begin taking hydroxychloroquine (Piaquenil) for rheumatoid arthritis. The patient is currently taking high-dose NSAIDs and methotrexate. What will the nurse teach the patient? A) That an eye exam is necessary at the beginning of therapy with this drug B) That the dose of NSAIDs may be decreased when beginning hydroxychloroquine C) To obtain tests of renal and hepatic function while taking this drug D) To stop taking methotrexate when starting hydroxychloroquine

A) That an eye exam is necessary at the beginning of therapy with this drug

The nurse working on a high-acuity medical-surgical unit is prioritizing care for four patients who were just admitted. Which patient should the nurse assess first? A) The NPO patient with a blood glucose level of 80 mg/dL who just received 20 units of 70/30 Novolin insulin B) The patient with a pulse of 58 beats per minute who is about to receive digoxin {Lanoxin) C) The patient with a blood pressure of 136/92 mm Hg who complains of having a headache D) The patient with an allergy to penicillin who Is receiving an infusion of vancomycin (Vancocin)

A) The NPO patient with a blood glucose level of 80 mg/dL who just received 20 units of 70/30 Novolin insulin

A nurse is reviewing the medications of a patient with diabetes before discharge. The nurse realizes that the patient will be going home on cole yelam, a bile acid sequestrant, and insulin. What patient education should the nurse provide in the discharge teaching for this patient? A) The patient needs to monitor t'h'e;.' lood sugar carefully, because colesevelam can cause hypoglycemia. B) The patient needs to monitor the blood sugar carefully, because colesevelam can cause hyperglycemia. C) The patient needs to take the insulin at least 3 hours before the colesevelam. D) The patient needs to use an oral antidiabetic agent or agents, not insulin, with colesevelam.

A) The patient needs to monitor t'h'e;.' lood sugar carefully, because colesevelam can cause hypoglycemia.

A 20-year-old female patient has suprapubic discomfort, pyuria, dysuria, and bacteriuria greater than 100,000/mL of urine. Which are the most likely diagnosis and treatment? A) Uncomplicated lower urinary tract infection treatable with short-course therapy B) Complicated lower urinary tract infection treatable with single-dose therapy C) Uncomplicated upper urinary tract Infection requiring 14 days of oral antibiotics D) Complicated upper urinary tract infection requiring parenteral antibiotics

A) Uncomplicated lower urinary tract infection treatable with short-course therapy

A patient who has recently immigrated to the United States from an impoverished country appears malnourished. The patient's folic acid levels are low, and the vitamin B12 levels are normal. The nurse expects this patient's treatment to Include: A) a diet high in folic acid. B) intramuscular folic acid. C) oral folic acid and vitamin B12. D) oral folic acid supplements.

A) a diet high in folic acid.

A patient with a recent onset of nephrosclerosis has been taking an ACE inhibitor and a thiazide diuretic. The patient's initial blood pressure was 148/100 mm Hg. After 1 month of drug therapy, the patient's blood pressure is 130/90 mm Hg. The nurse will contact the provider to discuss: A) adding a calcium channel blocker to this patient's drug regimen. B) lowering doses of the antihypertensive medications. C) ordering a high potassium diet. D) adding spironolactone to the drug regimen.

A) adding a calcium channel blocker to this patient's drug regimen.

A female patient with essential hypertension is being treated with hydralazine (Apresoline) 25 mg twice daily. The nurse assesses the patient and notes a heart rate of 96 beats per minute and a blood pressure of 110/72 mm Hg. The nurse will request an order to: A) administer a beta blocker. B) administer a drug that dilates veins. C) reduce the dose of hydralazine. D) give the patient a diuretic.

A) administer a beta blocker.

A patient with atrial fibrillation is receiving warfarin (Coumadin). The nurse notes that the patient's INR is 2.7. Before to giving the next dose of warfarin, the nurse will notify the provider and: A) administer the dose as ordered. B) request an order to decrease the dose. C) request an order to give vitamin K (phytonadione). D) request an order to increase the dose.

A) administer the dose as ordered.

A nurse is caring for a patient who has been taking low-dose aspirin for several days. The nurse notes that the patient has copious amounts of watery nasal secretions and an urticarial rash. The nurse will contact the provider to discuss: A) administering epinephrine. B) changing to a first-generation NSAID. C) reducing the dose of aspirin. D) giving an antihistamine.

A) administering epinephrine.

A patient who took an overdose of verapamil has been treated with gastric lavage and a cathartic. The emergency department nurse assesses the patient and notes a heart rate of 50 beats per minute and a blood pressure of 90/50 mm Hg. The nurse will anticipate: A) administering intravenous norepinephrine (NE) and isoproterenol. B) assisting with direct-current (DC) cardioversion. C) placing the patient in an upright position. D) preparing to administer a beta blocker.

A) administering intravenous norepinephrine (NE) and isoproterenol.

A child with an ear infection is not responding to treatment with amoxicillin (Amoxil). The nurse will expect the provider to order: A) amoxicillin-clavulanic acid (Augmentin). B) ampicillin. C) nafcillin. D) penicillin G (Benzylpenicillin).

A) amoxicillin-clavulanic acid (Augmentin).

A patient with a history of lung transplantation is admitted for treatment for a respiratory infection. The patient has been taking cyclosporlne (Sandimmune), prednisone, and azathioprine (Imuran) for 8 months. The provider has ordered azithromycin (Zithromax) to treat the infection and acetaminophen (Tylenol) as needed for fever. The nurse will contact the provider to: A) ask whether a different antibiotic can be used. B) ask that the prednisone be discontinued until the infection clears. C) suggest increasing the dose of cyclosporine. D) suggest using ibuprofen instead of acetaminophen.

A) ask whether a different antibiotic can be used.

A patient with heart failure who has been given digoxin (Lanoxin) daily for a week complains of nausea. Before giving the next dose, the nurse will: A) assess the heart rate (HR) and give the dose if the HR Is greater than 60 beats per minute. B) contact the provider to report digoxin toxicity. C) request an order for a decreased dose of digoxin. D) review the serum electrolyte values and withhold the dose if the potassium level is greater than 3.5 mEq/L.

A) assess the heart rate (HR) and give the dose if the HR Is greater than 60 beats per minute.

A patient has a positive urine culture 1week after completion of a 3-day course of antibiotics. The nurse anticipates that the prescriber will: A) begin a 2-week course of antibiotics. B) evaluate for a structural abnormality of the urinary tract. C) initiate long-term prophylaxis with low-dose antibiotics. D) treat the patient with intravenous antibiotics.

A) begin a 2-week course of antibiotics.

A patient with a history of renal calculi has fever, flank pain, and bacteriuria. The nurse caring for this patient understands that it is important for the provider to: A) begin antibiotic therapy after urine culture and sensitivity results are available. B) give prophylactic antibiotics for 6 weeks after the acute Infection has cleared. C) initiate immediate treatment with broad-spectrum antibiotics. D) refer the patient for intravenous antibiotics and hospitalization.

A) begin antibiotic therapy after urine culture and sensitivity results are available.

A nursing student asks the nurse how antibodies provide immune suppression. The nurse responds by telling the student that antibodies: A) block T-cell function. B) boost immune responses. C) reduce proliferation of B cells. D) suppress interferon production.

A) block T-cell function.

The nurse has just received an order for tenecteplase (TNKase) for a patient experiencing an acute myocardial infarction. The nurse should administer this drug: A) by bolus injection. B) by infusion pump over 24 hours. C) slowly over 90 minutes. D) via monitored, prolonged infusion.

A) by bolus injection.

A patient is taking a vasodilator that relaxes smooth muscles in veins. To help minimize drug side effects, the nurse caring for this patient will: A) caution the patient not to get up without assistance. B) encourage the patient to increase fluid intake. C) tell the patient to report shortness of breath. D) warn the patient about the possibility of bradycardia.

A) caution the patient not to get up without assistance.

A 1-year-old child with cretinism has been receiving 8 mcg/kg/day of levothyroxine (Synthroid). The child comes to the clinic for a well-child check up. The nurse will expect the provider to: A) change the dose of levothyroxine to 6 mcg/kg/day. B) discontinue the drug if the child's physical and mental development is normal. C) increase the dose to accommodate the child's increased growth. D) stop the drug for 4 weeks and check the child's TSH level.

A) change the dose of levothyroxine to 6 mcg/kg/day.

A patient newly diagnosed with PUD reports taking low-dose aspirin (ASA) for prevention of cardiovascular disease. The nurse learns that the patient drinks 2 to 3 cups of coffee each day and has a glass of wine with dinner 3 or 4 nights per week. The patient eats three meals a day. The nurse will counsel this patient to: A) change the meal pattern to five or six smaller meals per day. B) discontinue taking aspirin, because it can irritate the stomach. C) stop drinking wine or any other alcoholic beverage. D) switch to a decaffeinated coffee and reduce the number of servings.

A) change the meal pattern to five or six smaller meals per day.

A patient newly diagnosed with heart failure is admitted to the hospital. The nurse notes a pulse of 90 beats per minute. The nurse will observe this patient dosely for: A) decreased urine output. B) increased blood pressure. C) jugular vein distension. D) shortness of breath.

A) decreased urine output.

A nurse is reviewing a patient's most recent blood count and notes that the patient has a hemoglobin of 9.6 gm/dL and a hematocrit of 33%. The nurse will notify the provider and will expect initial treatment to include: A) determining the cause of the anemia. B) giving intravenous iron dextran. C) giving oral carbonyl iron (Feosol). D) teaching about dietary iron.

A) determining the cause of the anemia.

A patient has been receiving intravenous ketorolac 30 mg every 6 hours for postoperative pain for 4 days. The patient will begin taking oral ketorolac 10 mg every 4 to 6 hours to prepare for discharge in 1 or 2 days. The patient asks the nurse whether this drug will be prescribed for management of pain after discharge. The nurse will respond by telling the patient that the provider will prescribe a(n): A) different nonsteroidal anti-inflammatory drug for home management of pain. B) fixed-dose opioid analgesic/nonsteroidal anti-inflammatory medication. C) lower dose of the oral ketorolac for long-term pain management. D) intranasal preparation of ketorolac for pain management at home.

A) different nonsteroidal anti-inflammatory drug for home management of pain.

A patient with allergic rhinitis is taking a compound product of loratadine/pseudoephedrine (Ciaritin-D) every 12 hours. The patient complains of insomnia. The nurse notes that the patient is restless and anxious. The patient's heart rate is 90 beats per minute, and the blood pressure is 130/85 mm Hg. The nurse will contact the provider to: A) discuss using an intranasal glucocorticoid and loratadine (Ciaritin). B) report acute toxicity caused by pseudoephedrine. C) suggest using an agent with a sympathomimetic drug only. D) suggest using a topical decongestant to minimize systemic symptoms.

A) discuss using an intranasal glucocorticoid and loratadine (Ciaritin).

A patient with Stage C heart failure (HF) who has been taking an ACE inhibitor, a beta blocker, and a diuretic begins to have increased dyspnea, weight gain, and decreased urine output. The provider orders spironolactone (Aidactone). The nurse will make sure that the patient: A) does not take potassium supplements. B) monitors for a decreased heart rate. C) takes extra fluids. D) uses a salt substitute instead of salt.

A) does not take potassium supplements.

The nurse is caring for a patient receiving intravenous acyclovir (Zovirax). To prevent nephrotoxicity associated with intravenous acyclovir, the nurse will: A) hydrate the patient during the infusion and for 2 hours after the infusion. B) increase the patient's intake of foods rich In vitamin C. C) monitor urinary output every 30 minutes. D) provide a low-protein diet for 1 day before and 2 days after the acyclovir infusion.

A) hydrate the patient during the infusion and for 2 hours after the infusion.

A 5-year-old patient seen in an outpatient clinic is noted to have hypertension on three separate visits. Ambulatory blood pressure monitoring confirms that the child has hypertension. As an initial intervention with the child's parents, the nurse will expect to: A) perform a detailed health history on the child. B) provide teaching about antihypertensive medications. C) reassure the parents that their child may outgrow this condition. D) teach the parents about lifestyle changes and a special diet.

A) perform a detailed health history on the child.

A patient who is taking warfarin (Coumadin) has just vomited blood. The nurse notifies the provider, who orders lab work revealing a PT of 42 seconds and an INR of 3.5. The nurse will expect to administer: A) phytonadione (vitamin Kt) 1mg IV over 1hour. B) phytonadione (vitamin K1) 2.5 mg PO. C) protamine sulfate 20 mg PO. D) protamine sulfate 20 mg slow IV push.

A) phytonadione (vitamin Kt) 1mg IV over 1hour.

A patient who was in a motor vehicle accident sustained a severe head injury and is brought into the emergency department. The provider orders intravenous mannitol (Osmitrol). The nurse knows that this is given to: A) reduce intracranial pressure. B) reduce renal perfusion. C) reduce peripheral edema. D) restore extracellular fluid.

A) reduce intracranial pressure.

A patient comes to the clinic and receives valacyclovir (Valtrex) for a herpes-zoster virus. The nurse instructs the patient to take the medication: A) without regard to meals. B) without any dairy products. C) each morning. D) on a empty stomach

A) without regard to meals.

After receiving a nebulizer treatment with a beta-agonist, a patient complains of feeling slightly nervous and wonders if her asthma is getting worse. What is the nurse's best response?

A. "This is an expected adverse effect. Let me take your pulse."

The nurse is about to administer a stat dose of atropine sulfate to a patient who is experiencing a symptomatic cardiac dysrhythmia. During administration of this drug, the nurse should monitor the patient closely for which adverse effect?

A. Tachycardia

A patient with a history of chronic alcohol abuse has been admitted to the unit with cirrhosis. Upon review of the patient's laboratory test results, the nurse notes that the patient's ammonia level is elevated at 218 mcg/dl. What medication should the nurse prepare to administer? A) 0.9% NS B) Docusate sodium (Colace) C) Lactulose D) Polyethylene glycol (Miralax)

C) Lactulose

What medication is first line treatment for hypertension patients

Ace inhibitors

Patient teaching on tamsulosin (Flomax) that is taking it to reduce urinary obstruction due to BPH

Adverse effects include: headache, abnormal ejaculation, rhinitis. Interacting drugs include: alpha blockers, calcium channel blockers and erectile dysfunction drugs. This drug must be taken as directed and with cautious use by patients with by patients with blood pressure problems. This drug should also be used with caution by the elderly and while driving or engaging in other activities requiring alertness, because the adverse effects of this drug include blurred vision, dizziness, and drowsiness.

A pt is recovering from general anesthesia. What is the nurses main concern during immediate postoperative period

Airway

Do not drink this while taking phenobarbital.

Alcohol

A patient with variant angina wants to know why a beta blocker cannot be used to treat the angina. Which response by the nurse is correct? A) "A beta1-selective beta blocker could be used for variant angina." B) "Beta blockers do not helprelax coronary artery spasm." C) "Beta blockers do not help to improve the cardiac oxygen supply." D) "Beta blockers promote constriction of arterial smooth muscle."

B) "Beta blockers do not helprelax coronary artery spasm."

Signs and symptoms of hypersensitivity reaction?

Also know as allergic reaction, skin erythema, mild rash to severe, constriction of bronchial airways and tachycardia. vomiting and abdominal pain.

A patient is admitted to the coronary care unit from the emergency department after initial management of STEMI. A primary percutaneous coronary intervention has been performed. The nurse notes an initial heart rate of 56 beats per minute and a blood pressure of 120/80 mm/Hg. The patient has a history of stroke and a previous myocardial infarction. Which order will the nurse question? A) Aspirin B) Beta blocker C) Clopidogrel D) Heparin

B) Beta blocker

What is the most reliable measure for assessing diabetes control over a 3 - month period? cont.....

Another important caution to emphasize is the need to monitor liver function by reviewing ALT levels before drug therapy is initiated and periodically thereafter, such as every 3 months as ordered.

A patient who has been taking a glucocorticoid for several months arrives in the clinic. The nurse notes that the patient's cheeks appear full and that a prominent hump of fat is present on the upper back. The nurse will ask the provider to order which test(s)? A) Liver function tests B) Serum electrolytes C) Tuberculin skin test D) Vitamin D levels

B) Serum electrolytes

A nurse is teaching a patient who will begin taking methimazole (Tapazole) for Graves' disease about the medication. Which statement by the patient indicates understanding of the teaching? A) "Because of the risk for liver toxicity, Iwill need frequent liver function tests." B) "I should report a sore throat or fever to my provider if either occurs." C) "I will need a complete blood count every few months." D) "It is safe to get pregnant while taking this medication."

B) "I should report a sore throat or fever to my provider if either occurs."

What drug is used to reverse succinylcholine?

Anticholinesterase drugs such as neostigmine, pyridostigmine, and edrophonium are antidotes and are used to reverse muscle paralysis.

When the nurse completes patient teaching on a new medication, which method will the nurse use to evaluate the patient's understanding of the information

Ask specific questions to evaluate understanding.

A patient who recently started therapy with an HMG-COA reductase inhibitor asks the nurse, "How long will it take until I see an effect on my LDL cholesterol?" The nurse gives which correct answer? A) "At least 6 months is required to see a change." B) "A reduction usually is seen within 2 weeks." C) "Blood levels normalize immediately after the drug is started." D) "Cholesterol will not be affected, but triglycerides will fall within the first week."

B) "A reduction usually is seen within 2 weeks."

A child is to begin long-term glucocorticoid therapy. The parents ask the nurse about the effects of this drug on the child's growth. Which response by the nurse is correct? A) "A smaller dose may be indicated for your child." B) "Ask your provider about every other day dosing." C) "Long-acting glucocorticoid preparations should prevent growth suppression." D) "Oral glucocorticoids rarely cause growth suppression."

B) "Ask your provider about every other day dosing."

A pregnant patient in her third trimester asks the nurse whether she can take aspirin for headaches. Which response by the nurse is correct? A) "Aspirin is safe during the second and third trimesters of pregnancy." B) "Aspirin may cause premature closure of the ductus arteriosus in your baby." C) "Aspirin may induce premature labor and should be avoided in the third trimester." D) "You should use a first-generation nonsteroidal anti-inflammatory medication."

B) "Aspirin may cause premature closure of the ductus arteriosus in your baby."

A nurse provides teaching to a patient with angina who also has type 2 diabetes mellitus, asthma, and hypertension. Which statement by the patient indicates a need for further teaching? A) "An ACE inhibitor, in addition to nitroglycerin, will lower my risk of cardiovascular death." B) "Beta blockers can help control hypertension." C) "I should begin regular aerobic exercise." D) "Long-acting, slow-release calcium channel blockers can help with anginal pain."

B) "Beta blockers can help control hypertension."

A nurse is teaching a nursing student what is meant by "generations" of cephalosporins. Which statement by the student indicates understanding of the teaching? A) "Cephalosporins are assigned to generations based on their relative costs to administer." B) "Cephalosporins have increased activity against gram-negative bacteria with each generation." C) "First-generation cephalosporins have better penetration of the cerebrospinal fluid." D) "Later generations of cephalosporins have lower resistance to destruction by beta-lactamases."

B) "Cephalosporins have increased activity against gram-negative bacteria with each generation."

A nurse is teaching a group of nursing students about cyclooxygenase inhibitors. Which statement by a student indicates understanding of the teaching? A) "Cyclooxygenase-2 inhibition causes gastrointestinal side effects." B) "Cyclooxygenase-2 is considered the 'bad COX."' C) "Inhibition of cyclooxygenase-1 promotes myocardial infarction and stroke." D) "Inhibition of cyclooxygenase-1 results in suppression of inflammation."

B) "Cyclooxygenase-2 is considered the 'bad COX."'

A nurse is discussing the use of immunosuppressants for the treatment of inflammatory bowel disease (IBD) with a group of nursing students. Which statement by a student Indicates understanding of the teaching? A) "Azathioprine (Imuran) helps induce rapid remission of IBD." B) "Cyclosporine (Sandimmune) can be used to induce remission of IBD." C) "Cyclosporine (Sandimmune) does not have serious adverse effects." D) "Methotrexate is used long term to maintain remission of IBD."

B) "Cyclosporine (Sandimmune) can be used to induce remission of IBD."

A nurse and a nursing student are reviewing the care of a 30-kg patient who will receive intravenous aminophylline. Which statement by the student indicates an understanding of the administration of this medication? A) "After the loading dose has been given, the patient will receive 6 mg/kg/hr." B) "Dosing is titrated based on the serum theophylline levels." C) "If the patient's serum theophylline level is less than 15 mcg/mL, the rate should be reduced." D) "The patient will receive a loading dose of 180 mg over 5 minutes."

B) "Dosing is titrated based on the serum theophylline levels."

A patient has a viral sinus infection, and the provider tells the patient that antibiotics will not be prescribed. The patient wants to take an antibiotic and asks the nurse what possible harm could occur by taking an antibiotic. Which response by the nurse is correct? A) "Antibiotics are mutagenic and can produce changes that cause resistance," B) "Even normal flora can develop resistance and transfer this to pathogens." C) "Host cells become resistant to antibiotics with repeated use." D) "Patients who overuse antibiotics are more likely to have nosocomial infections."

B) "Even normal flora can develop resistance and transfer this to pathogens."

A patient who Is a long-distance runner has been diagnosed with rheumatoid arthritis in both knees and will begin glucocorticoid therapy. When teaching the patient about the medication, the nurse will include what information? A) "By reducing inflammation, this drug will slow the progression of your disease." B) "Giucocorticoids are used as adjunctive therapy during acute flare-ups." C) "Oral glucocorticoids cause less toxicity than intra-articular injections." D) "You may resume running when the pain and swelling improve."

B) "Giucocorticoids are used as adjunctive therapy during acute flare-ups."

A nurse is teaching a group of nursing students why glucocorticoids are preferred over nonsteroidal anti-inflammatory drugs In the treatment of inflammation. Which statement by a student indicates a need for further teaching? A) "Giucocorticoids act by multiple mechanisms and have more anti-inflammatory effects than NSAIDs." B) "Giucocorticoids have fewer side effects than nonsteroidal anti-inflammatory drugs." C) "Giucocorticoids help avert damage to tissues from lysosomal enzymes." D) "Giucocorticoids reduce the immune component of inflammation."

B) "Giucocorticoids have fewer side effects than nonsteroidal anti-inflammatory drugs."

A 12-year-old female patient is admitted to the hospital before sinus surgery. The nurse preparing to care for this patient notes that the admission hemoglobin is 10.2 gm/dL, and .the hematocrit is 32%. The nurse will ask the child's parents which question about their daughter? A) "Does she eat green, leafy vegetables?" B) "Has she begun menstruating?" C) "Is she a vegetarian?" D) "Is there a chance she might be pregnant?"

B) "Has she begun menstruating?"

A patient with chronic congestive heart failure has repeated hospitalizations in spite of ongoing treatment with hydrochlorothiazide (HydroDIURIL) and digoxin. The prescriber has ordered spironolactone (Aidactone) to be added to this patient's drug regimen, and the nurse provides education about this medication. Which statement by the patient indicates understanding of the teaching? A) "I can expect improvement within a few hours after taking this drug." B) "I need to stop taking potassium supplements." C) "I should use salt substitutes to prevent toxic side effects." D) "I should watch closely for dehydration."

B) "I need to stop taking potassium supplements."

A nurse provides teaching to a patient with allergic rhinitis who will begin using an Intranasal glucocorticoid. Which statement by the patient indicates understanding of the teaching? A) "If the glucocorticoid causes burning or itching, Ishould use it every other day." B) "I should clear secretions from my nose before using the glucocorticoid." C) "I should use the glucocorticoid whenever Ihave symptoms." D) "I will probably develop systemic effects from the topical glucocorticoid."

B) "I should clear secretions from my nose before using the glucocorticoid."

A nurse is providing medication teaching for a patient who will begin taking diclofenac (Voltaren) gel for osteoarthritis in both knees and elbows. Which statement by the patient indicates understanding of the teaching? A) "Because this is a topical drug, liver toxicity will not occur." B) "I should cover areas where the gel is applied to protect them from sunlight." C) "I will apply equal amounts of gel to all affected areas." D) "The topical formulation has the same toxicity as the oral formulation."

B) "I should cover areas where the gel is applied to protect them from sunlight."

A nurse is teaching a group of nursing students about influenza prevention. Which statement by a student indicates understanding of the teaching? A) "I may develop a mild case of influenza if Ireceive the vaccine by injection." B) "I should receive the vaccine every year in October or November." C) "If Ihave a cold Ishould postpone getting the vaccine." D) "The antiviral medications are as effective as the flu vaccine for preventing the flu."

B) "I should receive the vaccine every year in October or November."

A nurse is teaching a patient who is about to undergo allograft transplantation of the liver. Which statement by the patient indicates understanding of the post-transplant medications? A) "Immunosuppressants help reduce the risk of postoperative infection." B) "I will need to have periodic laboratory work to assess for toxicity." C) "I will need to take immunosuppressants until all signs of organ rejection are gone." D) "These drugs will prevent organ rejection."

B) "I will need to have periodic laboratory work to assess for toxicity."

A 7-year-old child has a cough, runny nose, congestion, and fever, and the parents ask the nurse to recommend an over-the-counter product. Which response by the nurse is correct? A) "Any product will be effective when combined with vitamin C and zinc." B) "It is best to use single-agent medications to treat individual symptoms." C) "The fever indicates that your child may need an antibiotic; you should call your provider." D) "You should ask your provider to prescribe a combination product that will treat multiple symptoms."

B) "It is best to use single-agent medications to treat individual symptoms."

The nurse is providing education to a patient with ulcerative colitis who is being treated with sulfasalazine (Azulfidine). What statement by the patient best demonstrates understanding of the action of sulfasalazine? A) "It treats the infection that triggers the condition." B) "It reduces the inflammation." C) "It enhances the immune response." D) "It increases the reabsorption of fluid."

B) "It reduces the inflammation."

A child who has perennial allergic rhinitis has been using an intranasal glucocorticoid. The provider has ordered montelukast (Singulair) to replace the glucocorticoid, because the child has frequent nosebleeds. When teaching this child's parents about montelukast, the nurse will include which statement? A) "Montelukast is also effective for treating infectious rhinitis." B) "Montelukast may cause behavior changes in your child." C) "Montelukast will treat both congestion and rhinitis." D) "Montelukast works best when combined with a topical decongestant."

B) "Montelukast may cause behavior changes in your child."

A nurse fs providing teaching to a nursing student about to care for a woman with irritable bowel syndrome with diarrhea (IBS-D) who is receiving alosetron (Lotronex). Which statement by the student indicates a need for further teaching? A) "I should evaluate the patient's abdomen for distension and bowel sounds." B) "Patients with diverticulitis and IBS-C may take this drug." C) "There are no known serious drug interactions with alosetron." D) "This drug is given only to women with severe IBS-D."

B) "Patients with diverticulitis and IBS-C may take this drug."

A nurse is reviewing the phenomenon of reflex tachycardia with a group of nursing students. Which statement by a student indicates understanding of this phenomenon? A) "Baroreceptors in the aortic arch stimulate the heart to beat faster." B) "Reflex tachycardia can negate the desired effects of vasodilators." C) "Reflex tachycardia is more likely to occur when beta blockers are given." D) "Venous dilation must occur for reflex tachycardia to occur."

B) "Reflex tachycardia can negate the desired effects of vasodilators."

The nurse prepares a patient with Graves' disease for radioactive iodine (131 I) therapy. Which statement made by the patient best demonstrates understanding of 131 I therapy? A) "I will have to isolate myself from my family for 1week so that Idon't expose them to radiation." B) "This drug will be taken up by the thyroid gland and will destroy the cells to reduce my hyperthyroidism." C) "This drug will help reduce my cold intolerance and weight gain." D) "I will need to take this drug on a daily basis for at least 1 year."

B) "This drug will be taken up by the thyroid gland and will destroy the cells to reduce my hyperthyroidism."

A patient has begun taking an HMG-COA reductase inhibitor. Which statement about this class of drugs made by the nurse during patient education would be inappropriate? A) "Statins reduce the risk of stroke." B) "You should come into the clinic for liver enzymes in 1month." C) "Statins reduce the risk of coronary events in people with normal LDL levels." D) ''You should maintain a healthy lifestyle and avoid high-fat foods."

B) "You should come into the clinic for liver enzymes in 1month."

A patient tells a nurse that she is thinking about getting pregnant and asks about nutritional supplements. What will the nurse recommend? A) A balanced diet high in green vegetables and grains B) 400 to 800 mg of folic acid per day C) A multivitamin with iron D) Vitamin B12 supplements

B) 400 to 800 mg of folic acid per day

What is ipratropium bromide (Atrovent)? A) A cholinergic agent used for perennial rhinitis B) An anticholinergic used for allergic rhinitis and colds C) A medication that is used only in patients with asthma D) A drug that is inappropriate for use in patients with allergic rhinitis

B) An anticholinergic used for allergic rhinitis and colds

A child has received amoxicillin (Amoxil) for three previous ear infections, but a current otitis media episode is not responding to treatment. The nurse caring for this child suspects that resistance to the bacterial agent has occurred by which microbial mechanism? A) Alteration of drug target molecules B) Antagonist production C) Drug inactivation D) Reduction of drug concentration at the site of action

B) Antagonist production

A nurse is educating the staff nurses about ketoacidosis. To evaluate the group's understanding, the nurse asks, "Which sign and symptoms would not be consistent with ketoacidosis?" The group gives which correct answer? A) Blood glucose level of 600 mg/dL B) Blood glucose level of 60 mg/dL C) Acidosis D) Ketones in the urine

B) Blood glucose level of 60 mg/dL

A 60-year-old female patient is about to begin long-term therapy with a glucocorticoid. Which of the following will be important for minimizing the risk of osteoporosis? A) Baseline vitamin D level B) Calcium and vitamin D supplements C) Estrogen therapy D) Skeletal x-rays before treatment

B) Calcium and vitamin D supplements

A patient admitted to the hospital has a history of peptic ulcer disease. The patient takes ranitidine (Zantac) and sucralfate (Carafate). The patient tells the nurse that discomfort is usually controlled but that symptoms occasionally flare up. What will the nurse do? A) Ask the provider about ordering an endoscopic examination. B) Contact the provider to discuss serologic testing and an antibiotic. C) Contact the provider to discuss switching to a proton pump inhibitor. D) Counsel the patient to avoid beverages containing caffeine.

B) Contact the provider to discuss serologic testing and an antibiotic.

The nurse knows that which two immunosuppressants are the most effective? A) Azathioprine (Imuran) and everolimus (Zortress) B) Cyclosporine (Sandimmune) and tacrolimus (Prograf) C) Methotrexate (Rheumatrex) and muromonab-CD3 (Orthoclone OKT3) D) Sirolimus (Rapamune) and methylprednisolone

B) Cyclosporine (Sandimmune) and tacrolimus (Prograf)

A patient has lamlvudine-sensitive hepatitis Band is taking entecavir (Baraclude) 0.5 mg per day. The nurse reviews the patient's laboratory values and notes a creatinine clearance of 40 ml/min. What will the nurse discuss with the patient's provider? A) Continuing the entecavir at the same dose B) Decreasing the entecavir dose to 0.25 mg per day C) Discontinuing the entecavir and considering dialysis D) Increasing the entecavir dose to 1mg per day

B) Decreasing the entecavir dose to 0.25 mg per day

A patient with arthritis asks a nurse which nonsteroidal anti-inflammatory medication is best to take. The nurse learns that this patient has a family history of cardiovascular disease. The nurse will recommend which NSAID? A) Celecoxib (Celebrex) B) Diclofenac (Voltaren) C) Ketorolac intranasal (Sprix) D) Naproxen (AIeve)

B) Diclofenac (Voltaren)

A patient receiving a cephalosporin develops a secondary intestinal infection caused by Clostridium difficile. What Is an appropriate treatment for this patient? A) Adding an antibiotic, such as vancomycin (Vancocin), to the patient's regimen B) Discontinuing the cephalosporin and beginning metronidazole (Flagyl) C) Discontinuing all antibiotics and providing fluid replacement D) Increasing the dose of the cephalosporin and providing isolation measures

B) Discontinuing the cephalosporin and beginning metronidazole (Flagyl)

An 88-year-old patient with heart failure has progressed to Stage D and is hospitalized for the third time in a month. The nurse will expect to discuss which topic with the patient's family? A) Antidysrhythmic medications B) End-of-life care C) Heart transplantation D) Implantable mechanical assist devices

B) End-of-life care

A patient is taking cyclosporine (Sandimmune) and prednisone to prevent organ rejection after right renal transplantation, The patient is febrile and complains of right-sided flank pain. The nurse reviews the patient's chart and finds that the patient's BUN and serum creatinine are elevated. The cyclosporine trough is 150 ng/mL. What will the nurse do? A) Be concerned that the left kidney is failing. B) Expect the provider to order intravenous methylprednisolone. C) Request an order for a urine culture. D) Suspect nephrotoxicity secondary to an elevated cyclosporine level.

B) Expect the provider to order intravenous methylprednisolone.

A female patient who begins taking spironolactone (Aidactone) as an adjunct to furosemide (Lasix) complains that her voice is deepening. What will the nurse do? A) Contact the provider to obtain an order for a complete blood count (CBC) and liver function tests. B) Explain that this drug binds with receptors for steroid hormones, causing this effect. C) Report this side effect to the provider and request another medication for this patient. D) Teach the patient to report any associated cough, which may indicate a more severe side effect.

B) Explain that this drug binds with receptors for steroid hormones, causing this effect.

A patient has seasonal allergies and needs an antihistamine to control symptoms. The patient likes to have wine with dinner several times a week and wants to know which antihistamine will be the safest to take. The nurse will tell the patient to discuss which medication with the provider? A) Cetlrizine (Zyrtec) B) Fexofenadine (Allegra) C) Levocetirizine (Xyzal) D) Loratadine (Ciaritin)

B) Fexofenadine (Allegra)

A patient asks the nurse what type of medications would be most effective for treating seasonal and perennial rhinitis. Which response by the nurse is correct? A) Pseudoephedrine (Sudafed) B) Fluticasone propionate (Fiuticasone) C) Loratadine (Ciaritin) D) Intranasal cromolyn sodium (Atrovent)

B) Fluticasone propionate (Fiuticasone)

A nurse is preparing to administer digoxin (Lanoxin) to a patient. The patient's heart rate is 62 beats per minute, and the blood pressure is 120/60 mm Hg. The last serum electrolyte value showed a potassium level of 5.2 mEq/L. What will the nurse do? A) Contact the provider to request an increased dose of digoxin. B) Give the dose of digoxin and notify the provider of the potassium level. C) Request an order for a diuretic. D) Withhold the dose and notify the provider of the heart rate.

B) Give the dose of digoxin and notify the provider of the potassium level.

A nurse caring for a patient notes that the patient has a temperature of 104° F arid a heart rate of 110 beats per minute. The patient's skin is warm and moist, and the patient complains that the room is too warm. The patient appears nervous and has protuberant eyes. The nurse will contact the provider to discuss: A) cretinism. B) Graves' disease. C) myxedema. D) Plummer's disease.

B) Graves' disease.

The nurse is caring for a pregnant patient who is in labor. The woman reports having had mild preeclampsia with a previous pregnancy. The nurse notes that the woman has a blood pressure of 168/102 mm Hg. The nurse will contact the provider to request an order for which drug? A) Angiotensin-converting enzyme (ACE) inhibitor B) Hydralazine (Apresoline) C) Magnesium sulfate D) Sodium nitroprusside

B) Hydralazine (Apresoline)

A man asks a nurse why he cannot use digoxin (Lanoxin) for his heart failure, because both of his parents used it for HF. The nurse will explain that digoxin is not first-line therapy for which reason? A) It causes tachycardia and increases the cardiac workload. B) It does not correct the underlying pathology of heart failure. C) It has a wide therapeutic range that makes dosing difficult. D) It may actually shorten the patient's life expectancy.

B) It does not correct the underlying pathology of heart failure.

A patient who is taking acetaminophen for pain wants to know why it does not cause gastrointestinal upset, as do other over-the-counter pain medications. The nurse will explain that this is most likely because of which property of acetaminophen? A) It does not inhibit cyelooxygenase. B) It has minimal effects at peripheral sites. C) It is more similar to opioids than to nonsteroidal anti-inflammatory drugs (NSAIDs). D) It is selective for cyclooxygenase- 2.

B) It has minimal effects at peripheral sites.

A patient will begin taking dabigatran etexilate (Pradaxa) to prevent stroke. The nurse will include which statement when teaching this patient? A) Dabigatran should be taken on an empty stomach to improve absorption. B) It is important not to crush, chew, or open capsules of dabigatran. C) The risk of bleeding with dabigatran is less than that with warfarin (Coumadin). D) To remember to take dabigatran twice dally, a pill organizer can be useful.

B) It is important not to crush, chew, or open capsules of dabigatran.

The nurse is caring for a pregnant patient recently diagnosed with hypothyrdfdism. The patient tells the nurse she does not want to take medications while she is pregnant. What will the nurse explain to this patient? A) Hypothyroidism is a normal effect of pregnancy and usually is of no consequence. B) Neuropsychologic deficits in the fetus can occur if the condition is not treated. C) No danger to the fetus exists until the third trimester. D) Treatment is required only if the patient is experiencing symptoms.

B) Neuropsychologic deficits in the fetus can occur if the condition is not treated.

A young adult woman will begin using an inhaled glucocorticoid to treat asthma. The nurse will teach this patient about the importance of which action? A) Lowering her calcium intake and Increasing her vitamin D intake B) Participating in weight-bearing exercises on a regular basis C) Taking oral glucocorticoids during times of acute stress D) Using two reliable forms of birth control to prevent pregnancy

B) Participating in weight-bearing exercises on a regular basis

A patient with angina who is taking ranolazine (Ranexa) has developed a respiratory infection and a dysrhythmia. The provider has ordered azithromycin (Zithromax) for the infection and amlodlpine for the dysrhythmia. A nursing student caring for this patient tells the nurse that the patient's heart rate is 70 beats per minute, and the blood pressure is 128/80 mm Hg. The nurse asks the student to discuss the plan for this patient's care. Which action is correct? A) Observe the patient closely for signs of respiratory toxicity. B) Question the order for azithromycin (Zithromax). C) Report the patient's increase in blood pressure to the provider. D) Request an order for a different calcium channel blocker.

B) Question the order for azithromycin (Zithromax).

A patient is taking bismuth subsalicylate (Pepto-Bismol) to prevent diarrhea. The nurse performing an assessment notes that the patient's tongue is black. What will the nurse do? A) Assess further for signs of gastrointestinal (GI) bleeding. B) Reassure the patient that this is an expected side effect of this drug. C) Request an order for liver function tests to evaluate for hepatotoxicity. D) Withhold the drug, because this is a sign of bismuth overdose.

B) Reassure the patient that this is an expected side effect of this drug.

A patient admitted to the hospital has been using phenylephrine nasal spray (Neo-Synephrine), 2 sprays every 4 hours, for a week. The patient complains that the medication is not working, because the nasal congestion has increased. What will the nurse do? A) Request an order for an oral decongestant to replace the intranasal phenylephrine. B) Request an order for an intranasal glucocorticoid to be used while the phenylephrine is withdrawn. C) Tell the patient to increase the dose of phenylephrine to 4 sprays every 4 hours. D) Tell the patient to stop using the phenylephrine and begin using an intranasal antihistamine.

B) Request an order for an intranasal glucocorticoid to be used while the phenylephrine is withdrawn.

A patient will undergo a colonoscopy, and the provider has ordered sodium phosphate as a bowel cleanser before the procedure. The nurse reviews the patient's chart and notes that the patient's creatinine clearance and blood urea nitrogen are both elevated. What will the nurse do? A) Reduce the amount of fluid given with the laxative to prevent fluid retention. B) Request an order to give polyethylene glycol and electrolytes (PEG-ELS) instead. C) Suggest that the patient reduce the dietary sodium intake. D) Suggest using a suppository laxative Instead.

B) Request an order to give polyethylene glycol and electrolytes (PEG-ELS) instead.

A child with seasonal rhinitis has used budesonide (Rhinocort Aqua) for several years. The parents are concerned that the child's rate of growth has slowed. What will the nurse do? A) Reassure the parents that this is an expected side effect. B) Suggest that the parents discuss using fluticasone (Fionase) with the provider. C) Tell the parents to administer the drug only when symptoms are severe. D) Tell the parents that antihistamines work as well as intranasal glucocorticoids.

B) Suggest that the parents discuss using fluticasone (Fionase) with the provider.

A pregnant patient who is taking ondansetron (Zofran) for morning sickness tells the nurse she is experiencing headache and dizziness. What will the nurse tell her? A) It is not safe to take this drug during pregnancy. B) These are common side effects of ondansetron. C) She should stop taking the ondansetron immediately. D) She should report these adverse effects to her provider.

B) These are common side effects of ondansetron.

A patient with hepatitis B begins treatment with adefovir (Hepsera) and asks the nurse how long the drug therapy will last. The nurse will tell the patient that the medication will need to be taken for: A) a lifetime. B) an indefinite, prolonged period of time. C) forty-eight weeks. D) until nephrotoxicity occurs.

B) an indefinite, prolonged period of time.

A patient tells a nurse that antihistamines help relieve cold symptoms and wants to know why they are not recommended or prescribed for this purpose. The nurse tells the patient that antihistamines provide relief from some cold symptoms by: A) blocking H1 receptors in nasal passages. B) blocking muscarinic receptors in the nose. C) reducing secretions at H2 receptor sites. D) having sedative effects, which help patients rest and sleep.

B) blocking muscarinic receptors in the nose.

A nurse caring for a patient receiving heparin therapy notes that the patient has a heart rate of 98 beats per minute and a blood pressure of 110/72 mm Hg. The patient's fingertips are purplish in color. A stat CBC shows a platelet count of less than 100,000 mm3• The nurse will: A) administer oxygen and notify the provider. B) discontinue the heparin and notify the provider. C) request an order for protamine sulfate. D) request an order for vitamin K (phytonadione).

B) discontinue the heparin and notify the provider.

A nurse assisting a nursing student with medications asks the student to describe how penicillins (PCNs) work to treat bacterial infections. The student is correct in responding that penicillins: A) disinhibit transpeptidases. B) disrupt bacterial cell wall synthesis. C) inhibit autolysins. D) inhibit host cell wall function.

B) disrupt bacterial cell wall synthesis.

A patient is started on immunosuppressant drugs after kidney transplantation and will be taking azathioprine (Imuran) as part of the drug regimen. The patient asks the nurse why it is necessary to have a specimen for a complete blood count drawn at the beginning of therapy and then periodically thereafter. The nurse explains that azathioprine can alter blood cells and tells the patient to report: A) alopecia. B) easy bruising. C) fatigue. D) gastrointestinal (GI) upset.

B) easy bruising.

A patient complains of burning on urination and increased frequency. The patient has a history of frequent urinary tract infections (UTis) and is going out of town in 2 days. To treat the infection quickly, the nurse would expect the healthcare provider to order: A) aztreonam (Azactam). B) fosfomycin (Monurol). C) trimethoprim/sulfamethoxazole (Bactrim). D) vancomycin (Vancocin).

B) fosfomycin (Monurol).

A patient Is admitted to the emergency department with chest pain. An electrocardiogram shows changes consistent with an evolving myocardial infarction. The patient's cardiac enzymes are pending. The nurse caring for this patient will expect to: A) administer aspirin when cardiac enzymes are completed. B) give alteplase (Activase) within 2 hours. C) give tenecteplase (TNKase) immediately. D) obtain an order for an INR.

B) give alteplase (Activase) within 2 hours.

A patient with HIV contracts herpes simplex virus (HSV), and the prescriber orders acyclovir (Zovirax) 400 mg PO BID for 10 days. After 7 days of therapy, the patient reports having an increased number of lesions. The nurse will expect the provider to: A) extend this patient's drug therapy to BID for 12 months. B) give intravenous foscarnet every 8 hours for 2 to 3 weeks. C) increase the acyclovir dose to 800 mg PO 5 times daily. D) order intravenous valacyclovir (Valtrex) 1gm PO BID for 10 days.

B) give intravenous foscarnet every 8 hours for 2 to 3 weeks.

A patient who experiences motion sickness is about to go on a cruise. The prescriber orders transdermal scopolamine (Transderm Scop). The patient asks the nurse why an oral agent is not ordered. The nurse will explain that the transdermal preparation: A) can be applied as needed at the first sign of nausea. B) has less intense anticholinergic effects than the oral form. C) is less sedating than the oral preparation. D) provides direct effects, because it is placed close to the vestibular apparatus of the ear.

B) has less intense anticholinergic effects than the oral form.

A patient with a cough has been advised to use guaifenesin. The patient asks the nurse to explain the purpose of the drug. The nurse will explain that guaifenesin: A) dries secretions to help suppress coughing so patients can rest. B) helps stimulate the flow of secretions to increase cough productivity. C) helps to relieve chest pain associated with a cough. D) stimulates the body's natural immune responses.

B) helps stimulate the flow of secretions to increase cough productivity.

A child has juvenile idiopathic arthritis and will begin a course of etanercept (Enbrei). The nurse will conduct a thorough health history with special attention to: A) bone growth development. B) immunization status. C) pulmonary function. D) seasonal allergies.

B) immunization status.

A patient who has taken warfarin (Coumadin) for a year begins taking carbamazepine. The nurse will anticipate an order to: A) decrease the dose of carbamazepine. B) increase the dose of warfarin. C) perform more frequent aPTT monitoring. D) provide extra dietary vitamin K.

B) increase the dose of warfarin.

A patient with new-onset exertional angina takes a nitroglycerin sublingual tablet, but the pain intensifies. The nurse notes that the patient has a heart rate of 76 beats per minute and a blood pressure of 120/82 mm Hg. The electrocardiogram is normal. The patient's lips and nail beds are pink, and there is no respiratory distress. The nurse will anticipate providing: A) an angiotensin-converting enzyme (ACE) inhibitor. B) intravenous nitroglycerin and a beta blocker. C) ranolazine (Ranexa) and quinidine. D) supplemental oxygen and intravenous morphine.

B) intravenous nitroglycerin and a beta blocker.

The nurse is taking a medication history on a newly admitted patient. The patient reports taking folic acid and vitamin B12. The nurse notifies the provider because of the concern that folic acid can: A) cause fetal malformation. B) mask the signs of vitamin Bn deficiency. C) negatively affect potassium levels. D) worsen megaloblastic anemia.

B) mask the signs of vitamin Bn deficiency.

A patient has a positive test for influenza type A and tells the nurse that symptoms began 5 days before being tested. The prescriber has ordered oseltamivir (Tamiflu). The nurse will tell the patient that oseltamivir: A) may decrease symptom duration by 2 or 3 days. B) may not be effective because of the delay in starting treatment. C) may reduce the severity but not the duration of symptoms. D) will alleviate symptoms within 24 hours of the start of therapy.

B) may not be effective because of the delay in starting treatment.

A patient has a positive test for hepatitis C and is admitted to the hospital. The admission laboratory tests reveal a normal ALT, and a liver biopsy is negative for hepatic fibrosis and inflammation. The nurse will prepare this patient for: A) dual therapy with pegylated interferon alfa and ribavirin. B) no medication therapy at this time. C) pegylated interferon alfa only until ALT levels are elevated. D) triple drug therapy with pegylated interferon alfa, ribavirin, and boceprevir.

B) no medication therapy at this time.

A postoperative. patient will begin anticoagulant therapy with rivaroxaban (Xarelta) after knee replacement surgery. The nurse performs a history and learns that the patient is taking erythromycin. The patient's creatinine clearance is 50 ml/min. The nurse will: A) administer the first dose of rivaroxaban as ordered. B) notify the provider to discuss changing the patient's antibiotic. C) request an order for a different anticoagulant medication. D) request an order to increase the dose of rivaroxaban.

B) notify the provider to discuss changing the patient's antibiotic.

A patient has 2+ pitting edema of the tower extremities bilaterally. Auscultation of the lungs reveals crackles bilaterally, and the serum potassium level is 6 mEqjL. Which diuretic agent ordered by the prescriber should the nurse question? A) Bumetanide (Bumex) B) Furosemide (Lasix) C) Spironolactone (Aidactone) D) Hydrochlorothiazide (HydroDIURIL)

C) Spironolactone (Aidactone)

An older adult patient with chronic obstructive pulmonary disease (COPD) develops bronchitis. The patient has a temperature of 39.5° C. The nurse will expect the provider to: A) obtain a sputum culture and wait for the results before prescribing an antibiotic. B) order empiric antibiotics while waiting for sputum culture results. C) treat symptomatically, because antibiotics are usually ineffective against bronchitis. D) treat the patient with more than one antibiotic without obtaining cultures.

B) order empiric antibiotics while waiting for sputum culture results.

The nurse is caring for a patient who is receiving vancomycin (Vancocin). The nurse notes that the patient is experiencing flushing, rash, pruritus, and urticaria. The patient's heart rate is 120 beats per minute, and the blood pressure is 92/57 mm Hg. The nurse understands that these findings are consistent with: A) allergic reaction. B) red man syndrome. C) rhabdomyolysis. D) Stevens-Johnson syndrome.

B) red man syndrome.

A patient who takes theophylline (Theochron) twice daily for chronic stable asthma develops an infection and will take ciprofloxacin. The nurse will contact the provider to discuss: A) changing to a different antibiotic. B) reducing the theophylline dose. C) giving theophylline once daily. D) switching from theophylline to a LABA.

B) reducing the theophylline dose.

A patient is admitted to the hospital and will begin taking levothyroxine (Synthroid). The nurse learns that the patient also takes warfarin (Coumadin). The nurse will notify the provider to discuss ____ the ____ dose. A) reducing; levothyroxine B) reducing; warfarin C) increasing; levothyroxine D) increasing; warfarin

B) reducing; warfarin

A nurse is caring for a patient after hip replacement surgery. The patient has been receiving iron replacement therapy for 2 days. The nurse notes that the patient's stools appear black. The patient is pale and complains of feeling tired. The patient's heart rate is 98 beats per minute, respirations are 20 breaths per minute, and the blood pressure is 100/50 mm Hg. The nurse will contact the provider to: A) report possible gastrointestinal hemorrhage. B) request a hemoglobin and hematocrit (H&H). C) request an order for a stool guaiac. D) suggest giving a hypertonic fluid bolus.

B) request a hemoglobin and hematocrit (H&H).

A patient who has been diagnosed with rheumatoid arthritis for 1month and has generalized symptoms is taking high-dose NSAIDs and an oral glucocorticoid. The provider has ordered methotrexate (Rheumatrex). The patient asks the nurse why methotrexate is necessary since pain and swelling have been well controlled with the other medications. The nurse will tell the patient that: A) a methotrexate regimen can reduce overall costs and side effects of treatment. B) starting methotrexate early can help delay joint degeneration. C) starting methotrexate now will help increase life expectancy. D) with methotrexate, doses of NSAIDs can be reduced to less toxic levels.

B) starting methotrexate early can help delay joint degeneration.

A patient reports taking an oral bisacodyl laxative (Dulcolax) for several years. The provider has suggested discontinuing the laxative, but the patient is unsure how to do this. The nurse will tell the patient to: A) stop taking the oral laxative and use a suppository until normal motility resumes. B) stop taking the laxative immediately and expect no stool for several days. C) switch to a bulk-forming laxative, such as methylcellulose (Metamucil). D) withdraw from the laxative slowly to avoid a rebound constipation effect.

B) stop taking the laxative immediately and expect no stool for several days.

A patient who does not consume alcohol or nicotine products reports a strong family history of hypertension and cardiovascular disease. The patient has a blood pressure of 126/82 and a normal weight and body mass index for height and age. The nurse wiJJ expect to teach this patient about: A) ACE inhibitors and calcium channel blocker medications. B) the DASH diet, sodium restriction, and exercise. C) Increased calcium and potassium supplements. D) thiazide diuretics and lifestyle changes.

B) the DASH diet, sodium restriction, and exercise.

A patient will begin taking fexofenadine (Allegra) for hay fever. The nurse teaching this patient will tell the patient that: A) fexofenadine should be taken with food to prevent gastrointestinal (GI) symptoms. B) the medication may be taken once or twice daily. C) tolerance to sedation will occur in a few weeks. D) with renal impairment, this drug should be taken every other day.

B) the medication may be taken once or twice daily.

A patient who takes daily doses of aspirin is scheduled for surgery next week. The nurse should advise the patient to: A) continue to use aspirin as scheduled. B) reduce the aspirin dosage by half until after surgery. C) stop using aspirin immediately. D) stop using aspirin 3 days before surgery.

C) stop using aspirin immediately.

How beta blockers decrease blood pressure?

Beta blockers lower the heart rate. ARB's work by blocking the binding of angiotensin at the receptor; the end result is a decrease in blood pressure.

A patient who is taking furosemide (Lasix) and digoxin will begin taking captopril (Capoten). The nurse is providing information about the drug. Which statement by the patient indicates a need for further teaching? A) "I can use acetaminophen for analgesia if needed." B) "I should stop taking the Lasix about a week before starting the Capoten." C) "I should take this medication on a full stomach." D) "I will need to have blood tests done every 2 weeks for a few months."

C) "I should take this medication on a full stomach."

A parent asks a nurse to recommend an intranasal decongestant for a 6-year-old child. Which response by the nurse is correct? A) "Decongestants are too sedating for children and should not be used." B) "Decongestants should not be given to children under 7 years old." C) "Decongestant drops are recommended instead of decongestant sprays." D) "Decongestant sprays should be used no longer than 5 to 10 days."

C) "Decongestant drops are recommended instead of decongestant sprays."

A patient has been receiving heparin while in the hospital to treat deep vein thromboses and will be discharged home with a prescription for enoxaparin (Lovenox). The nurse provides teaching for the nursing student who asks about the advantages of enoxaparin over heparin. Which statement by the student indicates a need for further teaching? A) "Enoxaparin does not require coagulation monitoring." B) "Enoxaparin has greater bioavailability than heparin." C) "Enoxaparin is more cost-effective than heparin." D) "Enoxaparin may be given using a fixed dosage."

C) "Enoxaparin is more cost-effective than heparin."

A nurse is discussing fibrinolytic therapy for the acute phase of STEMI management with a group of nursing students. Which statement by a student indicates understanding of this therapy? A) "Fibrinolytics are effective when the first dose is given up to 24 hours after symptom onset." B) "Fibrinolytics should be given once cardiac troponins reveal the presence of STEMI." C) "Fibrinolytics should be used with caution in patients with a history of cerebrovascular accident." D) "Patients should receive either an anticoagulant or an antiplatelet agent with a fibrinolytic drug."

C) "Fibrinolytics should be used with caution in patients with a history of cerebrovascular accident."

A 6-year-old child has frequent constipation. The nurse provides teaching after the parent asks the nurse why the provider recommended using laxatives only when needed. Which statement by the parent indicates a need for further teaching? A) "Children who take laxatives regularly can become dehydrated." B) "Chronic laxative use can cause electrolyte imbalances." C) "Frequent use of laxatives can cause diverticulitis." D) "The normal reflex to defecate can be inhibited with overuse of laxatives."

C) "Frequent use of laxatives can cause diverticulitis."

A nurse is providing discharge teaching instructions for a patient taking cholestyramine (Questran). Which statement made by the patient demonstrates a need for further teaching? A) "I will take warfarin (Coumadin) 1hour before my medicine." B) "I will increase the fluids and fiber in my diet." C) "I can take cholestyramine with my hydrochlorothiazide." D) "I will take digoxin 4 hours after taking the cholestyramine."

C) "I can take cholestyramine with my hydrochlorothiazide."

A patient's provider has recommended a bulk-forming laxative for occasional constipation. Which statement by the patient indicates understanding of the teaching about this agent? A) "I can take this medication long term." B) "I should not take this drug if Ihave diverticulitis." C) "I should take each dose with a full glass of water." D) "This drug can cause severe diarrhea."

C) "I should take each dose with a full glass of water."

A nurse is preparing to assist a nursing student in administering intravenous verapamil to a patient who also receives a beta blocker. The nurse asks the nursing student to discuss the plan of care for this patient. Which statement by the student indicates a need for further teaching? A) "I will check to see when the last dose of the beta blocker was given." B) "I will monitor vital signs closely to assess for hypotension." C) "I will monitor the heart rate frequently to assess for reflex tachycardia." D) "I will prepare to administer intravenous norepinephrine if necessary."

C) "I will monitor the heart rate frequently to assess for reflex tachycardia."

A patient who has been taking verapamil (Calan) for hypertension complains of constipation. The patient will begin taking amlodipine (Norvasc) to prevent this side effect. The nurse provides teaching about the difference between the two drugs. Which statement by the patient indicates that further teaching is needed? A) "I can expect dizziness and facial flushing with nifedipine." B) "I should notify the provider if Ihave swelling of my hands and feet." C) "I will need to take a beta blocker to prevent reflex tachycardia." D) "I will need to take this drug once a day."

C) "I will need to take a beta blocker to prevent reflex tachycardia."

A nurse provides teaching to a patient who has undergone kidney transplantation and will begin taking cyclosporine (Sandimmune), a glucocorticoid, and sirolimus (Rapamune). Which statement by the patient indicates understanding of the teaching? A) "I should take sirolimus at the same time as the cyclosporine." B) "I will need to have my blood sugar checked regularly." C) "I will need to take an antibiotic to prevent lung infections." D) "Taking this combination of drugs lowers my risk of kidney damage."

C) "I will need to take an antibiotic to prevent lung infections."

A patient has been taking chlorthalidone to treat hypertension. The patient's prescriber has just ordered the addition of spironolactone to the patient's drug regimen. Which statement by the patient indicates a need for further teaching? A) "I should continue following the DASH diet when adding this drug." B) "I should not take an ACE inhibitor when adding this drug." C) "I will need to take potassium supplements when adding this drug." D) "I will not experience a significant increase in diuresis when adding this drug."

C) "I will need to take potassium supplements when adding this drug."

During discharge instruction, a nurse wants to assess a patient's understanding of treatment with cholestyramine (Questran). Which statement made by the patient best demonstrates a need for additional teaching? A) "I will take cholestyramine (Questran) 1hour before my other medications." B) "I will increase fluids and fiber in my diet." C) "I will weigh myself weekly." D) "I will have my blood pressure checked weekly."

C) "I will weigh myself weekly."

A nurse is providing teaching for a patient with stable angina who will begin taking nitroglycerin. Which statement by the patient indicates understanding of the teaching? · A) "I should not participate in aerobic exercise while taking this drug." B) "I should take aspirin daily to reduce my need for nitroglycerin." C) "If Itake nitroglycerin before exertion, Ican reduce the chance of an anginal attack." D) "I take nitroglycerin to increase the amount of oxygen to my heart."

C) "If Itake nitroglycerin before exertion, Ican reduce the chance of an anginal attack."

A nurse tells a nursing student that the glucocorticoids given for rheumatoid arthritis are nearly identical to substances produced naturally by the body. The student remarks that the drug must be very safe. Which response by the nurse is correct? A) "As long as the drug is taken as prescribed, side effects usually do not occur." B) "By interrupting the Inflammatory process, these drugs inhibit side effects." C) "Side effects can occur and are dependent on the dose and duration of treatment." D) "The negative feedback loop prevents side effects."

C) "Side effects can occur and are dependent on the dose and duration of treatment."

The nurse is teaching a nursing student about the mechanism by which antimicrobial agents achieve selective toxicity. Which statement by the student indicates a need for further teaching? A) "Some agents disrupt the bacterial cell wall;" B) "Some agents act to block the conversion of para-aminobenzoic acid (PABA) to folic acid." C) "Some agents cause phagocytosis of bacterial cells." D) "Some agents weaken the cell wall, causing cell wall lysis."

C) "Some agents cause phagocytosis of bacterial cells."

A prescriber orders ramipril (Altace) for an obese patient with type 2 diabetes mellitus who has developed hypertension. The nurse provides teaching before dismissing the patient home. Which statement by the patient indicates understanding of the teaching? A) "I am less likely to develop diabetic nephropathy when taking this medication." B) "I should check my blood sugar more often, because hyperglycemia is a side effect of this drug." C) "Taking this medication helps reduce my risk of stroke and heart attack." D) "This medication will probably prevent the development of diabetic retinopathy."

C) "Taking this medication helps reduce my risk of stroke and heart attack."

A nurse provides teaching for a patient with cytomegalovirus (CMV) retinitis who will receive the ganciclovir ocular implant (Vitrasert). Which statement by the patient indicates a need for further teaching? A) "My vision may be blurred for 2 to 4 weeks after receiving the implant." B) "Surgical placement of the implant Is an outpatient procedure." . C) "The implant will remain in place permanently." D) "The implant will slow progression of CMV retinitis."

C) "The implant will remain in place permanently."

A nursing student who Is preparing to care for a postoperative patient with deep vein thrombosis asks the nurse why the patient must take heparin rather than warfarin. Which response by the nurse is correct? A) "Heparin has a longer half-life." B) "Heparin has fewer adverse effects." . C) "The onset of warfarin is delayed." D) "Warfarin prevents platelet aggregation."

C) "The onset of warfarin is delayed."

A patient with Crohn's disease will begin receiving an initial infusion of infliximab (Remicade). The nurse explains how this drug works to treat this disease. Which statement by the patient indicates a need for further teaching? A) "I may have an increased risk of infections, such as tuberculosis, when taking infliximab." B) "I should report chills, fever, itching, and shortness of breath while receiving the infusion." C) "This drug sometimes provides a complete cure of inflammatory bowel disease." D) "I will take the second dose in 2 weeks, the third dose in 6 weeks, and then a dose every 8 weeks thereafter."

C) "This drug sometimes provides a complete cure of inflammatory bowel disease."

A nurse is teaching a nursing student who wants to know how aspirin and nonaspirin first-generation NSAIDs differ. Which statement by the student indicates a need for further teaching? A) "Unlike aspirin, first-generation NSAIDs cause reversible inhibition of cyclooxygenase." B) "NSAIDs do not increase the risk of MI and stroke; however, unlike ASA, they do not provide protective benefits against those conditions." C) "Unlike aspirin, first-generation NSAIDs do not carry a risk of hypersensitivity reactions." D) "Unlike aspirin, first-generation NSAIDs cause little or no suppression of platelet aggregation."

C) "Unlike aspirin, first-generation NSAIDs do not carry a risk of hypersensitivity reactions."

A nurse is discussing heart failure with a group of nursing students. Which statement by a student reflects an understanding of how compensatory mechanisms can compound existing problems in patients with heart failure? A) "An increase in arteriolar tone to improve tissue perfusion can decrease resistance." B) "An increase in contractility to increase cardiac output can cause pulmonary edema." C) "When the heart rate increases to increase cardiac output, it can prevent adequate filling of the ventricles." D) "When venous tone increases to increase ventricular filling, an increase in arterial pressure occurs."

C) "When the heart rate increases to increase cardiac output, it can prevent adequate filling of the ventricles."

A 30-year-old male patient reports having two to four urinary tract infections a year. What will the nurse expect to teach this patient? A) "Make sure you void after intercourse and drink extra fluids to stay well hydrated." B) "We will treat each infection as a separate infection and treat with short-course therapy." C) "You will need to take a low dose of medication for 6 months to prevent infections." D) "You will need to take antibiotics for 4 to 6 weeks each time you have an infection."

C) "You will need to take a low dose of medication for 6 months to prevent infections."

A patient who takes nonsteroidal anti-inflammatory drugs (NSAIDs) for arthritis asks a nurse what can be done to prevent ulcers. The nurse will recommend asking the provider about using which medication? A) Antacids B) Antibiotics C) Antisecretory agents D) Mucosal protectants

C) Antisecretory agents

A nurse preparing to administer morning medications notes that a patient with a history of hypertension has been prescribed the angiotensin-converting enzyme (ACE) inhibitor captopril (Capoten) concurrently with spironolactone (Aidactone). Morning laboratory results reveal a serum sodium level of 144 mg/dL, a serum potassium level of 5.1 mEq/L, and a blood glucose level of 128 mgjdL. Which intervention is appropriate? A) Administer the medications as ordered. B) Ask the patient about the use of salt substitutes. C) Contact the provider to report the laboratory values. D) Request an order for furosemide (Lasix).

C) Contact the provider to report the laboratory values.

A patient who stops taking an ACE inhibitor because of its side effects will begin taking an angiotensin II receptor blocker (ARB) medication. Which side effect of ACE inhibitors will not occur with an ARB medication? A) Angioedema B) Cancer C) Cough D) Renal failure

C) Cough

The nurse is caring for a patient after recent renal transplantation. The patient is taking sirolimus (Rapamune) to prevent transplant rejection. What other medications would the nurse expect the patient to be taking? A) Rifampin and ketoconazole B) Carbamazepine and phenobarbital C) Cyclosporlne and glucocorticoids D) Amphotericin Band erythromycin

C) Cyclosporlne and glucocorticoids

A 30-year-old male patient will begin a three-drug regimen to treat peptic ulcer disease. The regimen will consist of bismuth subsalicylate, tetracycline, and cimetidine (Tagamet). The nurse will include which information when teaching this patient about this drug regimen? A) Black discoloration of the tongue and stools should be reported immediately. B) Central nervous system depression and confusion are likely to occur. C) Decreased libido, impotence, and gynecomastia are reversible side effects. D) Staining of the teeth may occur and is an indication for discontinuation of these drugs.

C) Decreased libido, impotence, and gynecomastia are reversible side effects.

A patient is taking clonidine for hypertension and reports having dry mouth and drowsiness. What will the nurse tell the patient? A) Beta blockers can reverse these side effects. B) Discontinue the medication immediately and notify the provider. C) Drink extra fluids and avoid driving when drowsy. D) Notify the provider if symptoms persist after several weeks.

C) Drink extra fluids and avoid driving when drowsy.

A provider has ordered intravenous promethazine (Phenergan) for an adult patient postoperatively to prevent nausea and vomiting (N/V). What will the nurse do? A) Administer the drug at a rate of 25 mg/min or more to achieve maximum effects. B) Contact 'the provider to suggest using dimenhydrinate (Dramamine) instead. C) Ensure that the IV is infusing freely through a large-bore needle. D) Monitor the patient closely for cardiac dysrhythmias.

C) Ensure that the IV is infusing freely through a large-bore needle.

A nurse is reviewing a patient's medications before administration. Which drug-to-drug interactions should most concern the nurse in a patient with a history of heart failure and a potassium level of 5.5 mEq/L? A) Furosemide (Lasix) and enalapril (Vasotec) B) Amlodipine (Norvasc) and spironolactone (Aidactone) C) Eplerenone (Inspra) and spironolactone (Aidactone) D) Metoprolol (Lopressor) and furosemide (Lasix)

C) Eplerenone (Inspra) and spironolactone (Aidactone)

A patient with chronic hypertension is admitted to the hospital. During the admission assessment, the nurse notes a heart rate of 96 beats per minute, a blood pressure of 150/90 mm Hg, bibasilar crackles, 2+ pitting edema of the ankles, and distension of the jugular veins. The nurse will contact the provider to request an order for which medication? A) ACE inhibitor B) Digoxin (Lanoxin) C) Furosemide (Lasix) D) Spironolactone (Aldactone)

C) Furosemide (Lasix)

A hospitalized patient complains of acute chest pain. The nurse administers a 0.3 mg sublingual nitroglycerin tablet, but the patient continues to complain of pain. Vital signs remain stable. What is the nurse's next step? A) Apply a nitroglycerin transdermal patch. B) Continue dosing at 10-minute intervals. C) Give a second dose of nitroglycerin in 5 minutes. D) Request an order for intravenous nitroglycerin.

C) Give a second dose of nitroglycerin in 5 minutes.

A nurse obtaining an admission history on an adult patient notes that the patient has a heart rate of 62 beats per minute, a blood pressure of 105/62 mm' Hg, and a temperature of 96.2° F. The patient appears pale and complains of always feeling cold and tired. The nurse will contact the provider to discuss tests for which condition? A) Cretinism B) Graves' disease C) Hypothyroidism D) Plummer's disease

C) Hypothyroidism

A nurse is administering a vasodilator that dilates resistance vessels. The nurse understands that this drug will have which effect on the patient? A) Decreased cardiac preload B) Decreased cardiac output C) Increased tissue perfusion D) Increased ventricular contraction

C) Increased tissue perfusion

The nurse Is preparing to administer antibodies to a patient to prevent acute rejection. By which route will the nurse administer the antibodies? A) Oral B) Intramuscular C) Intravenous D) Transdermal

C) Intravenous

A patient with asthma is admitted to an emergency department with a respiratory rate of 22 breaths per minute, a prolonged expiratory phase, tight wheezes, and an oxygen saturation of 90% on room air. The patient reports using fluticasone (Fiovent HFA) 110 mcg twice daily and has used 2 puffs of albuterol (Proventil HFA), 90 mcg/puff, every 4 hours for 2 days. The nurse will expect to administer which drug? A) Four puffs of albuterol, oxygen, and intravenous theophylline B) Intramuscular glucocorticoids and salmeterol by metered-dose inhaler C) Intravenous glucocorticoids, nebulized albuterol and ipratropium, and oxygen D) Intravenous theophylline, oxygen, and fluticasone (Fiovent HFA) 220 mcg

C) Intravenous glucocorticoids, nebulized albuterol and ipratropium, and oxygen

A patient with hypothyroidism begins taking PO levothyroxine (Synthroid). The nurse assesses the patient at the beginning of the shift and notes a heart rate of 62 beats per minute and a temperature of 97.2° F. The patient is lethargic and difficult to arouse. The nurse will contact the provider to request an order for which drug? A) Beta blocker B) Increased dose of PO levothyroxine C) Intravenous levothyroxine D) Methimazole (Tapazole)

C) Intravenous levothyroxine

A patient begins taking an ACE inhibitor and complains of a dry cough. What does the nurse correctly tell the patient about this symptom? A) It indicates that a serious side effect has occurred. B) It is a common side effect that occurs in almost all patients taking the drug. C) It may be uncomfortable enough that the drug will need to be discontinued. D) It occurs frequently in patients taking the drug but will subside over time.

C) It may be uncomfortable enough that the drug will need to be discontinued.

A patient has undergone a primary percutaneous coronary intervention with a sirolimus-eluting stent. The provider has ordered a daily dose of 243 mg of aspirin. What will the nurse tell this patient about the dose of aspirin? A) It will be necessary indefinitely. B) It will decrease to 81 mg per day in 6 months. C) It will decrease to 162 mg per day in 3 months. D) It will increase to 325 mg per day in 1month.

C) It will decrease to 162 mg per day in 3 months.

The nurse is administering medications to a patient who is receiving cyclosporine (Sandimmune). Which medication, when administered concurrently with cyclosporine, would warrant a reduction in the dosage of cyclosporine? A) Phenytoin (Dilantin) B) Prednisone C) Ketoconazole (Nizoral) D) Trimethoprim/sulfamethoxazole (Bactrim)

C) Ketoconazole (Nizoral)

An older adult patient with severe gastroesophageal reflux disease (GERD) has had only minimal relief using a histamine2-receptor antagonist (H2RA). The patient is to begin taking omeprazole (Prilosec). What will the nurse teach this patient? A) A complete cure is expected with this medication. B) lifestyle changes can be as effective as medication therapy. C) Long-term therapy may be needed. D) The medication will be used until surgery can be performed.

C) Long-term therapy may be needed.

A patient with hypertension will begin taking an alpha1 blocker. What will the nurse teach this patient? A) A persistent cough is a known side effect of this drug. B) Eat foods rich in potassium while taking this drug. C) Move slowly from sitting to standing when taking this drug. D) Report shortness of breath while taking this drug.

C) Move slowly from sitting to standing when taking this drug.

A patient is receiving intravenous promethazine (Phenergan) 25 mg for postoperative nausea and vomiting. What is an important nursing action when giving this drug? A) Giving the dose as an IV push over 3 to 5 minutes B) Infusing the dose with microbore tubing and an infusion pump C) Observing the IV insertion site frequently for patency D) Telling the patient to report dry mouth and sedation

C) Observing the IV insertion site frequently for patency

Insulin glargine is prescribed for a hospitalized patient who is diabetic. When will the nurse administer this drug? A) Approximately 15 to 30 minutes before each meal B) Inthe morning and at 4 PM C) Once daily at bedtime D) After meals and at bedtime

C) Once daily at bedtime

A patient who is taking digoxin is admitted to the hospital for treatment of congestive heart failure. The prescriber has ordered furosemide (Lasix). The nurse notes an irregular heart rate of 86 beats per minute, a respiratory rate of 22 breaths per minute, and a blood pressure of 130/82 mm Hg. The nurse auscultates crackles in both lungs. Which laboratory value causes the nurse the most concern? A) Blood glucose level of 120 mg/dL B) Oxygen saturation of 90% C) Potassium level of 3.5 mEq/L D) Sodium level of 140 mEq/L

C) Potassium level of 3.5 mEq/L

A patient has an infection caused by Streptococcus pyogenes. The prescriber has ordered dicloxacillin PO. What will the nurse do? A) Administer the medication as ordered. B) Contact the provider to suggest giving the drug IV. C) Question the need for a penicillinase-resistant penicillin. D) Suggest ordering vancomycin to treat this infection.

C) Question the need for a penicillinase-resistant penicillin.

An 18-month-old child develops an urticarial reaction after a transfusion. The prescriber orders intravenous promethazine (Phenergan). What will the nurse do? A) Give the medication as ordered. B) Monitor the child for bronchoconstriction. C) Question the order. D) Request an order to give the drug orally.

C) Question the order.

A patient is diagnosed with Zollinger- Ellison syndrome. Which medication does the nurse expect the provider to order for this patient? A) Cimetidine (Tagamet) B) Esomeprazole (Nexium) C) Ranitidine (Zantac) D) Sucralfate (Carafate)

C) Ranitidine (Zantac)

A patient is diagnosed with moderate vitamin B12 deficiency. The nurse reviews the laboratory work and notes that the plasma B12 is low; also, a Schilling test reveals B12 malabsorption. The provider orders oral cyanocobalamin 500 meg per day. The nurse will contact the provider to: A) discuss IM dosing. B) request an order for folic acid. C) suggest an increased dose. D) suggest platelet transfusion therapy.

C) suggest an increased dose.

A nurse checks a patient's vital signs in the hospital and notes a blood pressure of 146/98 mm Hg. What will the nurse do? A) Instruct the patient to consume a Jaw-sodium diet. B) Prepare the patient for an electrocardiogram and blood tests. C) Recheck the patient's blood pressure in the other arm. D) Request an order for a thiazide diuretic.

C) Recheck the patient's blood pressure in the other arm.

A patient is about to receive penicillin G for an infection that is highly sensitive to this drug. While obtaining the patient's medication history, the nurse learns that the patient experienced a rash when given amoxicillin (Amoxil) as a child 20 years earlier. What will the nurse do? A) Ask the provider to order a cephalosporin. B) Reassure the patient that allergic responses diminish over time. C) Request an order for a skin test to assess the current risk. D) Suggest using a desensitization schedule to administer the drug.

C) Request an order for a skin test to assess the current risk.

A patient has undergone a PCI, and the provider orders clopidogrel to be given for 12 months, along with an ACE inhibitor and heparin. What will the nurse do? A) Question the need for heparin. B) Request an order for a beta blocker. C) Request an order for aspirin. D) Suggest ordering clopidogrel for 14 days.

C) Request an order for aspirin.

A patient is receiving intravenous potassium penicillin G, 2 million units to be administered over 1hour. At 1900, the nurse notes that the dose hung at 1830 has infused completely. What will the nurse do? A) Assess the skin at the infusion site for signs of tissue necrosis. B) Observe the patient closely for confusion and other neurotoxic effects. C) Request an order for serum electrolytes and cardiac monitoring. D) Watch the patient's actions and report any bizarre behaviors.

C) Request an order for serum electrolytes and cardiac monitoring.

A nurse caring for a patient who is undergoing a third round of chemotherapy is preparing to administer ondansetron (Zofran) 30 minutes before initiation of the chemotherapy. The patient tells the nurse that the ondansetron did not work as well the last time as It had the first time. What will the nurse do? A) Administer the ondansetron at the same time as the chemotherapy. B) Contact the provider to suggest using high-dose intravenous dolasetron (Anzemet). C) Request an order to administer dexamethasone with the ondansetron. D) Suggest to the provider that loperamide (Lomotil) be given with the ondansetron.

C) Request an order to administer dexamethasone with the ondansetron.

A prescriber has ordered rosuvastatin (Crestor) for a patient with non-alcoholic-related cirrhosis. Which intervention would be most appropriate for the nurse before administration of this drug? A) Question the order, because rosuvastatin is contraindicated in patients with liver disease. B) No intervention is necessary; just administer the drug as ordered. C) Review the baseline liver function test results. D) Assess the patient for liver disease.

C) Review the baseline liver function test results.

A patient is taking gentamicin (Garamycin) and furosemide (Lasix). The nurse should counsel this patient to report which symptom? A) Frequent nocturia B) Headaches C) Ringing in the ears D) Urinary retention

C) Ringing in the ears

A patient who is receiving a final dose of intravenous (IV) cephalosporin begins to complain of pain and irritation at the infusion site. The nurse observes signs of redness at the IV insertion site and along the vein. What is the nurse's priority action? A) Apply warm packs to the arm, and infuse the medication at a slower rate. B) Continue the infusion while elevating the arm. C) Select an alternate intravenous site and administer the infusion more slowly. D) Request central venous access.

C) Select an alternate intravenous site and administer the infusion more slowly.

A patient with type 1 diabetes recently became pregnant. The nurse plans a blood glucose testing schedule for her. What is the recommended monitoring schedule? A) Before each meal and before bed B) In the morning for a fasting level and at 4 PM for the peak level C) Six or seven times a day D) Three times a day, along with urine glucose testing

C) Six or seven times a day

A nursing student asks a nurse to clarify the differences between the mechanisms of spontaneous mutation and conjugation in acquired resistance of microbes. What will the nurse say? A) Conjugation results in a gradual increase in resistance. B) Conjugation results in random changes in the microbe's DNA. C) Spontaneous mutation leads to resistance to only one antimicrobial agent. D) Spontaneous mutation can transfer DNA from one organism to another.

C) Spontaneous mutation leads to resistance to only one antimicrobial agent.

A patient is being treated for chemotherapy-induced nausea and vomiting (CINV) with ondansetron (Zofran) and dexamethasone. The patient reports getting relief during and immediatefy after chemotherapy but has significant nausea and vomiting several days after each chemotherapy treatment. What will the nurse do? A) Contact the provider to discuss increasing the dose of ondansetron. B) Suggest giving prolonged doses of dexamethasone. C) Suggest adding aprepitant (Emend) to the medication regimen. D) Tell the patient to ask the provider about changing the ondansetron to aprepitant.

C) Suggest adding aprepitant (Emend) to the medication regimen.

A nurse is evaluating a patient admitted to the emergency department with an evolving STEMI for possible administration of thrombolytic therapy. Which information, identified during history taking, would contraindicate this type of therapy? A) The patient just completed her last menstrual cycle. B) The patient states that the chest pain started 1hour ago. C) The patient has a history of a small cerebral aneurysm. D) The patient has hypertension that is well controlled by diuretic therapy.

C) The patient has a history of a small cerebral aneurysm.

A prescriber orders hydroxyzine (Vistaril) for a patient with acute urticaria. The nurse will include which information when teaching the patient about this drug? A) The drug will reduce redness and itching but not edema. B) This antihistamine is not likely to cause sedation. C) The patient should avoid drinking alcohol while taking the drug. D) The patient should report shortness of breath while taking the drug.

C) The patient should avoid drinking alcohol while taking the drug.

A patient in her twenties with Graves' disease who takes methimazole (Tapazole) tells a nurse that she is trying to conceive and asks about disease management during pregnancy. What will the nurse tell her? A) Methimazole is safe to take throughout pregnancy. B) Propylthiouracil should be taken throughout her pregnancy. C) The patient should discuss changing to propylthiouracil from now until her second trimester with her provider. D) The patient should discuss therapy with iodine-131 instead of medications with her provider.

C) The patient should discuss changing to propylthiouracil from now until her second trimester with her provider.

A patient tells the nurse that she takes aspirin for menstrual cramps, but she does not feel that it works well. What will the nurse suggest? A) The patient should avoid any type of COX inhibitor because of the risk of Reye's syndrome. B) The patient should increase the dose to a level that suppresses inflammation. C) The patient should use a first-generation nonsteroidal anti-inflammatory medication instead. D) The patient should use acetaminophen because of its selective effects on·uterlne smooth muscle.

C) The patient should use a first-generation nonsteroidal anti-inflammatory medication instead.

A patient with rheumatoid arthritis is taking leflunomide (Arava) and an oral contraceptive. She tells the nurse she would like to get pregnant. What will the nurse tell her? A) That leflunomide is not dangerous during the first trimester of pregnancy B) That plasma levels of leflunomide will drop rapidly when she stops taking it C) To ask her provider about an 11-day course of cholestyramine D) To stop taking leflunomide when she stops using contraception

C) To ask her provider about an 11-day course of cholestyramine

A patient has had blood pressures of 150/95 mm Hg and 148/90 mm Hg on two separate office visits. The patient reports a blood pressure of 145/92 mm Hg taken in an ambulatory setting. The patient's diagnostic tests are all normal. The nurse will expect this pptient's provider to order: A) a beta blocker. B) a loop diuretic and spironolactone. C) a thiazide diuretic. D) counseling on lifestyle changes.

C) a thiazide diuretic.

A nurse is reviewing a patient's medications and realizes that gemfibrozil (Lopid) and warfarin (Coumadin) are to be administered concomitantly. This finding concerns the nurse, who is aware that the ____ will be ____. A) level of gemfibrozil; increased B) level of gemfibrozil; reduced C) anticoagulant effects; increased D) anticoagulant effects; reduced

C) anticoagulant effects; increased

A patient who is taking clopidogrel (Piavix) calls the nurse to report black, tarry stools and coffee-ground emesis. The nurse will tell the patient to: A) ask the provider about using aspirin instead of clopidogrel. B) consume a diet high in vitamin K. C) continue taking the clopidogrel until talking to the provider. D) stop taking the clopidogrel immediately.

C) continue taking the clopidogrel until talking to the provider.

A patient is taking a thiazide diuretic for hypertension and quinidine to treat a dysrhythmia. The prescriber orders digoxin 0.125 mg to improve this patient's cardiac output. The nurse should contact the provider to request: A) adding spironolactone (Aidactone). B) reducing the dose of digoxin. C) discontinuing the quinidine. D) giving potassium supplements.

C) discontinuing the quinidine.

A 5-year-old child is brought to the emergency department after ingesting diphenhydramine (Benadryl). The child is uncoordinated and agitated. The nurse observes that the child's face is flushed and that the heart rate is 110 beats per minute. The nurse will expect to: A) administer atropine to reverse the adverse effects. B) give diazepam to prevent seizures. C) give activated charcoal to absorb the drug. D) prepare to provide mechanical ventilation.

C) give activated charcoal to absorb the drug.

A patient is taking enalapril (Vasotec). The nurse understands that patients taking this type of drug for heart failure need to be monitored carefully for: A) hypernatremia. B) hypertension. C) hyperkalemia. D) hypokalemia.

C) hyperkalemia.

A clinic patient who has been taking a glucocorticoid for arthritis for several months remarks to the nurse, "It's a good thing my symptoms are better, because my mother has been quite ill, and Ihave to take care of her." The patient's blood pressure is 100/60 mm Hg. The nurse will report this to the provider and ask about: A) reducing the patient's dose. B) using every other day dosing. C) increasing the patient's dose. D) tapering the dose.

C) increasing the patient's dose.

A nursing student asks a nurse why pegylated interferon alfa is used instead of regular interferon for a patient with hepatitis C. The nurse will tell the student that pegylated interferon: A) decreases the need for additional medications. B) has fewer adverse effects than interferon. C) is administered less frequently than interferon. D) may be given orally to increase ease of use.

C) is administered less frequently than interferon.

A patient will begin taking etanercept (Enbrei) for severe rheumatoid arthritis. The patient has been taking methotrexate (Rheumatrex). The patient asks ifthe etanercept is stronger than the methotrexate. The nurse will tell the patient that etanercept ____ methotrexate. A) has synergistic effects with B) helps reduce adverse effects associated with C) is better at delaying progression of joint damage than D) has fewer adverse effects than

C) is better at delaying progression of joint damage than

A patient with heart failure who takes a thiazide diuretic and digoxin (Lanoxin) is admitted for shortness of breath. The patient's heart rate Is 66 beats per minute, and the blood pressure is 130/88 mm Hg. The serum potassium level is 3.8 mEq/l, and the digoxin level is 0.8 ng/mL. The nurse admitting this patient understands that the patient: A) has digoxin toxicity. B) is showing signs of renal failure. C) is experiencing worsening of the disease. D) needs a potassium-sparing diuretic.

C) is experiencing worsening of the disease.

A nurse is admitting a patient to the hospital who reports having recurrent, crampy abdominal pain followed by diarrhea. The patient tells the nurse that the diarrhea usually relieves the pain and that these symptoms have occurred daily for the past 6 months. The patient undergoes a colonoscopy, for which the findings are normal. The nurse will plan to teach this patient to: A) use antispasmodic.medications. B) avoid food containing lactose and gluten. C) keep a food, stress, and symptom diary. D) use antidiarrheal drugs to manage symptoms

C) keep a food, stress, and symptom diary.

A pregnant female patient with bacteriuria, suprapubic pain, urinary urgency and frequency, and a low grade fever is allergic to sulfa, ciprofloxacin, and amoxicillin. The nurse knows that the best alternative for treating this urinary tract infection is with: A) cephalexin (Keflex). B) fosfomycin (Monural). C) methenamine (Hiprex). D) nitrofurantoin (Macrodantin).

C) methenamine (Hiprex).

A patient with malaise has been taking daptomycin (Cubicin) for an infection and is concurrently taking simvastatin (Zocor). The nurse should be concerned if the patient complains of: A) nausea. B) tiredness. C) muscle pain. D) headache.

C) muscle pain.

A patient presents to the clinic with complaints of muscle aches, muscle pain, and weakness. Upon review of the individual's medications, the nurse notes that the patient is concurrently taking gemfibrozil (Lopid) and atorvastatin (Lipitor). The nurse should assess the patient for the development of: A) migraines. B) hypothyroidism. C) myopathy. D) heart failure.

C) myopathy.

A patient who is pregnant has a history of recurrent genital herpesvirus (HSV). The patient asks the nurse what will be done to suppress an outbreak when she is near term. The nurse will tell the patient that: A) antiviral medications are not safe during pregnancy. B) intravenous antiviral agents will be used if an outbreak occurs. C) oral acyclovir (Zovirax) may be used during pregnancy. D) topical acyclovir (Zovirax) must be used to control outbreaks.

C) oral acyclovir (Zovirax) may be used during pregnancy.

A patient is admitted with lower abdominal pain and nausea. The nurse performing the initial assessment notes that the patient's abdomen is distended and firm, and hypoactive bowel sounds are present. The patient has not had a stool for 3 days. The nurse will contact the provider, who will: A) order a bulk-forming laxative. B) order extra fluids and fiber. C) perform diagnostic tests. D) prescribe a cathartic laxative.

C) perform diagnostic tests.

A patient with renal disease is scheduled for a colonoscopy. Before the procedure, the nurse will anticipate administering: A) glycerin suppository. B) magnesium hydroxide (MOM). C) polyethylene glycol and electrolytes. D) sodium phosphate.

C) polyethylene glycol and electrolytes.

A patient presents to the emergency department with complaints of chills, severe flank pain, dysuria, and urinary frequency. The vital signs are temperature of 102.9° F, pulse of 92 beats per minute, respirations of 24 breaths per minute, and blood pressure of 119/58 mm Hg. The nurse would be correct to suspect that the patient shows signs and symptoms of: A) acute cystitis. B) urinary tract infection. C) pyelonephritis. D) prostatitis.

C) pyelonephritis.

A nursing student asks a nurse why a patient in hypertensive crisis is receiving both intravenous sodium nitroprusside (Nitropress) and oral hydralazine (Apresoline). The nurse will explain that this is done to prevent: A) cyanide poisoning. B) fluid retention. C) rebound hypertension. D) reflex tachycardia.

C) rebound hypertension.

A patient with an infection caused by Pseudomonas aeruginosa is being treated with piperacillin. The nurse providing care reviews the patient's laboratory reports and notes that the patient's blood urea nitrogen and serum creatinine levels are elevated. The nurse will contact the provider to discuss: A) adding an aminoglycoside. B) changing to penicillin G. C) reducing the dose of piperacillin. D) ordering nafcillin.

C) reducing the dose of piperacillin.

175. Many medications list side effects that include dry mouth, constipation, and urinary retention. What kind of effects are these? a. Alpha adrenergic b. Anticholinergic c. Beta adrenergic d. Sympathetic

Cholinergic stimulation by ACh causes increased salivation, increased gastrointestinal (GI) motility, and relaxation of the bladder trigone and sphincter. Anticholinergic medications inhibit these responses, causing dry mouth, constipation, and urinary retention. Alpha-adrenergic agents effect processes in the sympathetic nervous system, causing peripheral vasoconstriction and pupil dilation, among other effects. The effects of beta-adrenergic medications are seen mostly in the heart and blood vessels. The sympathetic nervous system has both cholinergic and adrenergic transmitters.

What is the most reliable measure for assessing diabetes control over the preceding 3-month period? A) Self-monitoring blood glucose (SMBG) graph report B) Patient's report C) Fasting blood glucose level D) Glycosylated hemoglobin level

D) Glycosylated hemoglobin level

A patient who uses a fentanyl (Duragesic) patch for chronic cancer pain complains to the nurse of the rapid onset of pain at a level 9 (of a 0-10 scale) and requests "something for pain that will work quickly." The best way for the nurse to document this information is as

Correct Answer: A, Breakthrough pain Rationale: Pain that occurs beyond the chronic pain already being treated by appropriate analgesics is termed breakthrough pain. Neuropathic pain is caused by damage to peripheral nerves or the central nervous system (CNS). Somatic pain is localized and arises from bone, joint, muscle, skin, or connective tissue. Referred pain is pain that is localized in uninjured tissue.

A patient diagnosed with STEMI is about to undergo a primary percutaneous coronary intervention (PCI). Which combination of pharmacotherapeutic agents will be given to augment this procedure? A) Beta blocker and nitroglycerin B) Abciximab and a fibrinolytic drug C) Angiotensin-converting enzyme (ACE) inhibitor and aspirin D) Heparin, aspirin, and clopidogrel

D) Heparin, aspirin, and clopidogrel

What is cytotec and what is the benefit of taking them with NSAID's?

Cytotec has been shown to reduce the incidence of gastric ulcers in patients taking NSAID's. Cytotec is thought to inhibit gastrica acid secretion. They are believed to protect the gastric mucos from injury.

A parent asks a nurse about giving diphenhydramine (Benadryl) to a child to relieve cold symptoms. Which response by the nurse is correct? A) "Benadryl must be given in higher doses to provide relief for cold symptoms." B) "Intranasal glucocorticoids are more effective for treating cold symptoms." C) "Nasal antihistamines are more effective for treating cold symptoms." D) "Because histamine doesn't cause cold symptoms, Benadryl would not be effective."

D) "Because histamine doesn't cause cold symptoms, Benadryl would not be effective."

A nurse is providing patient education about colesevelam (Welchel), a bile acid sequestrant. Which statement made by the patient demonstrates a need for further teaching? A) "Colesevelam will reduce my levels of low-density lipoprotein." B) "Colesevelam will augment my statin drug therapy." C) "I will not have to worry about having as many drug interactions as Idid when Itook cholestyramine." D) "I cannot take digoxin while on this medication."

D) "I cannot take digoxin while on this medication."

A female patient who is not taking any other medications is prescribed aliskiren (Tekturna), a direct renin inhibitor (DRI). The nurse reviews medication information with the patient. Which statement by the patient indicates understanding of the teaching? A) "If Iget pregnant, Ishould stop taking this drug by the second trimester." B) "If Itake this drug with a high-fat meal, it will be more effective." C) "I should restrict my potassium intake while taking this drug." D) "I should take this medication 1hour before sitting down to a meal."

D) "I should take this medication 1hour before sitting down to a meal."

A patient has been taking warfarin (Coumadln) for atrial fibrillation. The provider has ordered dabigatran etexilate (Pradaxa) to replace the warfarin. The nurse teaches the patient about the change in drug regimen. Which statement by the patient indicates understanding of the teaching? A) "I may need to adjust the dose of dabigatran after weaning off the warfarin." B) "I should continue to take the warfarin after beginning the dabigatran until my INR is greater than 3." C) "I should stop taking the warfarin 3 days before starting the dabigatran." D) "I will stop taking the warfarin and will start taking the dabigatran when my INR is less than 2."

D) "I will stop taking the warfarin and will start taking the dabigatran when my INR is less than 2."

A nurse is teaching a patient who has been diagnosed with hypothyroidism about levothyroxine (Synthroid). Which statement by the patient indicates a need for further teaching? A) "I should not take heartburn medication without consulting my provider." B) "I should report insomnia, tremors, and an increased heart rate to my provider." C) "If Itake a multivitamin with iron, Ishould take it 4 hours after the Synthroid." D) "If Itake calcium supplements, Imay need to decrease my dose of Synthroid."

D) "If Itake calcium supplements, Imay need to decrease my dose of Synthroid."

A patient in the emergency department has severe chest pain. The nurse administers morphine intravenously. The patient asks the nurse why morphine is given. Whichresponse by the nurse is correct? A) "Morphine helps by reducing anxiety and relieving pain." B) "Morphine helps by reducing pain and dissolving clots." C) "Morphine helps by relieving pain and lowering blood pressure." D) "Morphine helps by relieving pain and reducing the cardiac oxygen demand."

D) "Morphine helps by relieving pain and reducing the cardiac oxygen demand."

A nurse provides dietary counseling for a patient newly diagnosed with type 1 diabetes. Which instruction should be included? A) "You may eat any foods you want and cover the glucose increase with sliding scale, regular insulin." B) "Most of the calories you eat should be in the form of protein to promote fat breakdown and preserve muscle mass." C) "Your total caloric intake should not exceed 1800 calories in a 24-hour period." D) "Most of your calories should be in the form of carbohydrates and monounsaturated fats."

D) "Most of your calories should be in the form of carbohydrates and monounsaturated fats."

A nurse is teaching a group of nursing students about the different formulations of beta2-adrenergic agonist medications. Which statement by a student indicates understanding of the teaching? A) "Beta2-adrenergic agonists provide quick relief via any formulation." B) "Long-acting betaz agonists may be used alone to prevent attacks." C) "Short-acting betaz agonists are usually given by nebulizer." D) "Oral beta2 agonists are not useful for short-term treatment."

D) "Oral beta2 agonists are not useful for short-term treatment."

A nursing student is caring for a patient who is taking sucralfate (Carafate) and ciprofloxacin (Cipro) to treat peptic ulcer disease. The student asks the nurse about the pharmacokinetics of sucralfate. Which statement by the student indicates a need for further teaching? A) "Sucralfate adheres to the ulcer and blocks the back-diffusion of hydrogen ions." B) "Sucralfate and ciprofloxacin should be administered 1hour apart." C) "Sucralfate does not cause systemic side effects." D) "Sucralfate has a moderate acid-neutralizing capacity."

D) "Sucralfate has a moderate acid-neutralizing capacity."

A child with otitis media has had three ear infections in the past year. The child has just completed a 10-day course of amoxicillin (Amoxil) with no improvement. The parent asks the nurse why this drug is not working, because it has worked in the past. What will the nurse tell the patient? A) "Amoxicillin is too narrow in spectrum." B) "The bacteria have developed a three-layer cell envelope." C) "The bacteria have developed penicillin-binding proteins (PBPs) that have a low affinity for penicillins." D) "The bacteria have synthesized penicillinase."

D) "The bacteria have synthesized penicillinase."

A patient newly diagnosed with asthma receives prescriptions for an inhaled glucocorticoid and an inhaled beta,-adrenergic agonist. Which statement by the patient indicates understanding of this medication regimen? A) "I should use the glucocorticoid as needed when symptoms flare." B) "I will need to use the beta2 adrenergic agonist drug daily." C) "The beta2-adrenergic agonist suppresses the synthesis of inflammatory mediators." D) "The glucocorticoid Is used as prophylaxis to prevent exacerbations."

D) "The glucocorticoid Is used as prophylaxis to prevent exacerbations."

A nurse is caring for a patient with hypertension and diabetes. The patient's prescriber recently ordered an angiotensin-converting enzyme (ACE) inhibitor. Which statement by the patient indicates understanding of the nurse's teaching about this drug? A) "ACE inhibitors only dilate veins, so Imay have more edema in my ankles." B) "ACE inhibitors prevent the nervous system from causing increased blood pressure." C) "This drug can also cause shortness of breath because of bronchoconstriction." D) "This drug will help lower my blood pressure and will benefit my kidneys."

D) "This drug will help lower my blood pressure and will benefit my kidneys."

A nursing student asks the nurse why multi-drug therapy is often used to treat hypertension. Which statement by the student indicates a need for further teaching? A) "Multi-drug therapy often means that drugs may be given in lower doses." B) "Some agents are used to offset adverse effects of other agents." C) "Treatment of hypertension via different mechanisms increases success." D) "Two or more drugs will lower blood pressure more quickly."

D) "Two or more drugs will lower blood pressure more quickly."

A nurse is discussing the difference between stable and variant angina with a group of nursing students. Which statement by a student indicates the need for further teaching? A) "Beta blockers are effective in stable angina but not in variant angina." B) "In both types of angina, prophylactic treatment is possible." C) "Variant angina is primarily treated with vasodilators to increase oxygen supply." D) "Variant angina is the result of increased oxygen demand by the heart."

D) "Variant angina is the result of increased oxygen demand by the heart."

A patient has a free T4 level of 0.6 ng/dL and a free T3 of 220 pg/dL. The patient asks the nurse what these laboratory values mean. How will the nurse respond? A) "These laboratory values indicate that you may have Graves' disease." B) "These results suggest you may have hyperthyroidism." C) "We will need to obtain a total T4 and a total T3 to tell for sure." D) "We will need to obtain a TSH level to better evaluate your diagnosis."

D) "We will need to obtain a TSH level to better evaluate your diagnosis."

A nurse has provided education for a patient newly diagnosed with hypertension who is just beginning therapy with antihypertensive medications. Which statement by the patient indicates a need for further teaching? A) "I may experience serious long-term problems even if Iam not having symptoms." B) "I should report side effects to the provider since other drugs may be substituted." C) "I will need to take medications on a long-term basis." D) "When my symptoms subside, Imay discontinue the medications."

D) "When my symptoms subside, Imay discontinue the medications."

A nurse is instructing a patient receiving a cholesterol-lowering agent. Which information should the nurse include in the patient education? A) "This medication will replace other interventions you have been trying." B) "It is important for you to double your dose if you miss one to maintain therapeutic blood levels." C) "Stop taking the medication if you experience constipation." D) "You should continue your exercise program to increase your HDL serum levels."

D) "You should continue your exercise program to increase your HDL serum levels."

A patient who has seasonal allergies in the spring and fall asks the nurse about oral antihistamines. Which response by the nurse is correct? A) "Anticholinergic effects are more common with second-generation antihistamines." B) "First-generation antihistamines, such as diphenhydramine (Benadryl), are more effective." C) "Make sure you take antihistamines only when you have symptoms to minimize side effects." D) "You should take oral antihistamines daily during each allergy season to get maximum effects."

D) "You should take oral antihistamines daily during each allergy season to get maximum effects."

A patient with gastroesophageal reflux disease (GERD) iS to begin taking oral metoclopramide (Reglan). The patient asks the nurse about the medication. Which response by the nurse is correct? A) "After 3 months, if the drug is not effective, you may need to increase the dose." B) "Metoclopramide may cause hiccups, especially after meals:' C) "Serious side effects may occur but will stop when the drug is discontinued." D) "You should take the drug 30 minutes before each meal and at bedtime."

D) "You should take the drug 30 minutes before each meal and at bedtime."

A parent asks a nurse if the provider will prescribe an antibiotic for a child who attends school with several children who have strep throat. The child is complaining of a sore throat and has a fever. What will be the nurse's response? A) "Because strep throat is likely, your child should be treated empirically." B) "With good hand washing, your child should not get strep throat." C) "Your child probably has strep throat, so your provider will order an antibiotic." D) "Your child should come to the clinic to have a throat culture done today."

D) "Your child should come to the clinic to have a throat culture done today."

A young, nonpregnant female patient with a history of a previous urinary tract infection is experiencing dysuria, urinary urgency and frequency, and suprapubic pain of 3 days' duration. She is afebrile. A urine culture is positive for more than 100,000/mL of urine. The nurse caring for this patient knows that which treatment is most effective? A) A 14-day course of amoxicillin with clavulanic acid (Augmentin) B) A 7-day course of ciprofloxacin (Cipro) C) A single dose of fosfomycin (Monurai) D) A 3-day course of trimethoprim/sulfamethoxazole (Bactrim)

D) A 3-day course of trimethoprim/sulfamethoxazole (Bactrim)

The parent of an infant with otitis media asks the nurse why the prescriber has ordered amoxiciliin (Amoxil) and not ampicillin (Unasyn). What will the nurse tell the parent? A) Amoxicillin is a broader spectrum antibiotic than ampicillin. B) Amoxiciliin is not inactivated by beta-lactamases. C) Ampicillin is associated with more allergic reactions. D) Ampicillin is not as acid stable as amoxicillin.

D) Ampicillin is not as acid stable as amoxicillin.

Which patient with a urinary tract infection will require hospitalization and intravenous antibiotics? A) A 5-year-old child with a fever of 100.5° F, dysuria, and bacteriuria B) A pregnant woman with bacteriuria, suprapubic pain, and fever C) A young man with dysuria, flank pain, and a previous urinary tract infection D) An older adult man with a low-grade fever, flank pain, and an indwelling catheter

D) An older adult man with a low-grade fever, flank pain, and an indwelling catheter

Which cephalosporin may be used to treat meningitis? A) Cefaclor B) Cefazolln C) Cefoxitin D) Cefotaxime

D) Cefotaxime

A nurse prepares to administer a scheduled dose of digoxin. The nurse finds a new laboratory report showing a plasma digoxin level of 0.7 ng/ml. What action should the nurse take? A) Withhold the drug for an hour and reassess the level. B) Withhold the drug and notify the prescriber immediately. C) Administer Digibind to counteract the toxicity. D) Check the patient's apical pulse, and if it is within a safe range, administer the digoxin.

D) Check the patient's apical pulse, and if it is within a safe range, administer the digoxin.

A patient takes an ACE inhibitor to treat hypertension and tells the nurse that she wants to become pregnant. She asks whether she should continue taking the medication while she is pregnant. What will the nurse tell her? A) Controlling her blood pressure will decrease her risk of preeclampsia. B) Ask the provider about changing to an ARB during pregnancy. C) Continue taking the ACE Inhibitor during her pregnancy. D) Discuss using methyldopa instead while she is pregnant.

D) Discuss using methyldopa instead while she is pregnant.

A patient has been taking senna (Senokot) for several days, and the nurse notes that the urine is yellowish-brown. What does the nurse know about this symptom? A) It indicates that renal failure has occurred. B) It is caused by dehydration, which is a laxative side effect. C) It is a sign of toxicity, indicating immediate withdrawal of the drug. D) It is an expected, harmless effect of senna.

D) It is an expected, harmless effect of senna.

A patient with persistent, frequent asthma exacerbations asks a nurse about a long-acting beta2-agonist medication. What will the nurse tell this patient? A) LABAs are safer than short-acting betaz agonists. B) LABAs can be used on an as-needed basis to treat symptoms. C) LABAs reduce the risk of asthma-related deaths. D) LABAs should be combined with an inhaled glucocorticoid.

D) LABAs should be combined with an inhaled glucocorticoid.

A patient with type 1 diabetes is eating breakfast at 7:30AM. Blood sugars are on a sliding scale and are ordered before a meal and at bedtime. The patient's blood sugar level is 317 mg/dL. Which formulation of insulin should the nurse prepare to administer? A) No insulin should be administered. B) NPH C) 70/30 mix D) Lispro (Humalog)

D) Lispro (Humalog)

A patient with a liver transplant has been receiving cyclosporine (Sandimmune) for 6 months. The nurse reviews this patient's laboratory results and notes a sharp increase in the blood urea nitrogen (BUN) and serum creatinine. Vital signs are normal, and the patient reports no discomfort. What does the nurse suspect? A) Hepatotoxicity B) Infection C) Organ rejection D) Nephrotoxicity

D) Nephrotoxicity

The nurse is caring for a patient on a medical-surgical unit who has a fever of unknown origin. The prescriber has ordered a broad-spectrum antibiotic. Which intervention is the priority? A) Administering the antibiotic immediately B) Administering antipyretics as soon as possible C) Delaying administration of the antibiotic until the culture results are available D) Obtaining all cultures before the antibiotic is administered

D) Obtaining all cultures before the antibiotic is administered

A patient has been taking psyllium (Metamucil) 2 to 3 times daily for several days. The patient complains of stomach pain but has not had a stool. What will the nurse do? A) Ask the patient to drink a full glass of water. B) Give another dose of the psyllium. C) Request an order for a bisacodyl (Dulcolax) suppository. D) Palpate the patient's abdomen and auscultate for bowel sounds.

D) Palpate the patient's abdomen and auscultate for bowel sounds.

The parent of a child with cerebral palsy reports that the child has pebble-like stools most of the time and seems uncomfortable if several days have passed between stools. The nurse will suggest that the parent discuss which medication with the child's provider? A) Bisacodyl (Dulcolax) suppositories B) Magnesium citrate C) Methylcellulose (Citrucel) D) Polyethylene glycol (MiraLax)

D) Polyethylene glycol (MiraLax)

A patient is preparing for travel and reports having frequent motion sickness. The nurse will tell the patient to ask the provider about which antihistamine? A) Desloratadine (Ciarinex) B) Diphenhydramine (Phenadoz) C) Hydroxyzine (Vistaril) D) Promethazine (Phenergan)

D) Promethazine (Phenergan)

A patient with asthma comes to a clinic for treatment of an asthma exacerbation. The patient uses an inhaled glucocorticoid, montelukast (Singulair), and a SABA via MDI. The nurse assesses the patient and notes a respiratory rate of 18 breaths per minute, a heart rate of 96 beats per minute, and a·n oxygen saturation of 95%. The nurse auscultates mild expiratory wheezes and equal breath sounds bilaterally. What will the nurse do? A) Contact the provider to request a systemic glucocorticoid. B) Contact the provider to suggest using a long-acting betaz agonist. C) Evaluate the need for teaching about MDI use. D) Question the patient about how much albuterol has been used.

D) Question the patient about how much albuterol has been used.

An Asian patient comes to the clinic. Upon assessment, the nurse notes a sllght yellow tint to the skin and sclera, edema, and hepatomegaly. The drug history reveals that the patient has been taking rosuvastatin (Crestor) for 6 months. The nurse is concerned, because rosuvastatin (Crestor) has been shown to do what? A) Cause renal and pancreatic toxicity specifically in Asian patients B) Cause renal failure C) Create myoglobin in the urine D) Reach levels twice as high in Asians as in Caucasians, resulting in a greater chance of hepatotoxicity

D) Reach levels twice as high in Asians as in Caucasians, resulting in a greater chance of hepatotoxicity

A child is receiving a combination albuterol/ipratropium (DuoNeb) inhalation treatment. The patient complains of a dry mouth and sore throat. What will the nurse do? A) Contact the provider to report systemic anticholinergic side effects. B) Discontinue the aerosol treatment immediately. C) Notify the provider of a possible allergic reaction. D) Reassure the patient that these are expected side effects.

D) Reassure the patient that these are expected side effects.

A patient with hypertension is prescribed an angiotensin-converting enzyme (ACE) inhibitor. The nurse reviewing this patient's chart before administering the medication will be most concerned about which other disease process? A) Bronchial asthma B) Coronary artery disease C) Diabetes mellitus D) Renal artery stenosis

D) Renal artery stenosis

Which medication should be used for asthma patients as part of step 1 management? A) Combination inhaled glucocorticoids/long-acting beta2 agonists B) Inhaled low-dose glucocorticoids C) Long-acting beta2 agonists D) Short-acting betaz agonists

D) Short-acting betaz agonists

A patient is admitted with severe hypertensive crisis. The nurse will anticipate administering which medication? A) Captopril PO B) Hydralazine (Apresoline) 25 mg PO C) Minoxidil 20 mg PO D) Sodium nitroprusside (Nitropress) IV

D) Sodium nitroprusside (Nitropress) IV

A patient with hypertension is taking furosemide {Lasix) for congestive heart failure. The prescriber orders digoxin to help increase cardiac output. What other medication will the nurse expect to be ordered for this patient? A) Bumetanide (Bumex) B) Chlorothiazide (Diuril) C) Hydrochlorothiazide (HydroDIURjL) D) Spironolactone (Aidactone)

D) Spironolactone (Aidactone)

An older adult patient Is diagnosed with hypothyroidism. The initial free T4 level is 0.5 mg/dl, and the TSH level is 8 microunits/mL. The prescriber orders levothyroxine (Levothroid) 100 mcg/day PO. What will the nurse do? A) Administer the medication as ordered. B) Contact the provider to discuss giving the levothyroxine IV. C) Request an order to give desiccated thyroid (Armour Thyroid). D) Suggest that the provider lower the dose.

D) Suggest that the provider lower the dose.

The nurse has been caring for a patient who has been taking antibiotics for 3 weeks. Upon assessing the patient, the nurse notices the individual has developed oral thrush. What describes the etibltlgy of the thrush? A) Antibiotic resistance B) Community-acquired infection C) Nosocomial Infection D) Suprainfection

D) Suprainfection

A patient is about to receive prednisone for tendonitis. The nurse reviewing the chart would be concerned about which of the following in the patient's medical history? A) Asthma and allergic rhinitis B) Gouty arthritis C) Seborrheic dermatitis D) Systemic fungal infection

D) Systemic fungal infection

A patient with recurrent bacterial pneumonia is treated with an antibiotic that has worked previously but is not working to reduce symptoms in a current infection. The nurse caring for this patient understands that this is likely for which reason? A) The antibiotic altered the genetic makeup of the bacterial strain causing this infection. B) The antibiotic caused host cells to change and become more susceptible to bacterial effects. C) The antibiotic caused a mutation of the organism leading to reduced drug sensitivity. D) The antibiotic destroyed competing organisms that secrete substances toxic to the pathogen.

D) The antibiotic destroyed competing organisms that secrete substances toxic to the pathogen.

A patient with type 1 diabetes who takes insulin reports taking propranolol for hypertension. Why is the nurse concerned? A) The beta blocker can cause insulin resistance. B) Using the two agents together increases the risk of ketoacidosis. C) Propranolol increases insulin requirements because of receptor blocking. D) The beta blocker can mask the symptoms of hypoglycemia.

D) The beta blocker can mask the symptoms of hypoglycemia.

A female patient taking an ACE inhibitor learns that she is pregnant. What will the nurse tell this patient? A) The fetus most likely will have serious congenital defects. B) The fetus must be monitored closely while the patient is taking this drug. C) The patient's prescriber probably will change her medication to an ARB. D) The patient should stop taking the medication and contact her provider immediately.

D) The patient should stop taking the medication and contact her provider immediately.

A patient taking gemfibrozil (Lopid) and rosuvastatin (Crestor) concurrently begins to complain of muscle aches, fatigue, and weakness. What should the nurse monitor? A) For tendon tenderness B) For a lupuslike syndrome C) The patient's LFT results D) The patient's creatinine kinase levels

D) The patient's creatinine kinase levels

A 55-year-old patient asks a nurse about taking aspirin to prevent heart disease. The patient does not have a history of myocardial infarction. Her cholesterol and blood pressure are normal, and she does not smoke. What will the nurse tell the patient? A) Aspirin is useful only for preventing a second myocardial infarction. B) She should ask her provider about using a P2Y12 ADP receptor antagonist. C) She should take one 81 mg tablet per day to prevent myocardial infarction. D) There is most likely no protective benefit for patients her age.

D) There is most likely no protective benefit for patients her age.

Which statement is accurate about the long-term complications of diabetes? A) Long-term complications are almost always the result of hypoglycemia and ketoacidosis. B) The complication rates for patients with tightly controlled type 2 diabetes are the same as for those whose disease is not tightly controlled. C) Tightly controlling type 1diabetes produces excessive episodes of life-threatening hypoglycemia. D) Tightly controlling both types of diabetes reduces the risk of eye, kidney, and nerve damage.

D) Tightly controlling both types of diabetes reduces the risk of eye, kidney, and nerve damage.

A patient who is taking nitrofurantoin calls the nurse to report several side effects. Which side effect of this drug causes the most concern and would require discontinuation of the medication? A) Anorexia, nausea, and vomiting B) Brown-colored urine C) Drowsiness D) Tingling of the fingers

D) Tingling of the fingers

A patient is preparing to travel to perform missionary work in a region with poor drinking water. The provider gives the patient a prescription for ciprofioxacin (Cipro) to take on the trip. What will the nurse instruct this patient to do? A) Combine the antibiotic with an antidiarrheal medication, such as loperamide. B) Start taking the ciprofloxacin 1week before traveling. C) Take 1tablet of ciproftoxacin with each meal for best results. D) Use the drug If symptoms are severe or do not improve in a few days.

D) Use the drug If symptoms are severe or do not improve in a few days.

A patient is admitted to the hospital. The patient's initial laboratory results reveal megaloblastic anemia. The patient complains of tingling of the hands and appears confused. The nurse suspects what in this patient? A) Celiac disease B) Folic acid deficiency C) Iron deficiency anemia D) Vitamin B12 deficiency

D) Vitamin B12 deficiency

A nurse is caring for a patient with cancer who has been undergoing chemotherapy. The patient has oral mucositis as a result of the chemotherapy, and the provider has ordered palifermin (Kepivance). Which is an appropriate nursing action when giving this drug? A) Administering the drug as a slow IV infusion B) Flushing the IV line with heparin before infusing the drug C) Giving the drug within 6 hours of the chemotherapy D) Warning the patient about the potential for distortion of taste

D) Warning the patient about the potential for distortion of taste

A patient was given a 30-day supply of Feosol and has been taking the drug for 4 weeks for iron deficiency anemia. The patient's initial hemoglobin was 8.9 gm/dL. The nurse notes that the hemoglobin has risen to 9.7 gm/dL. What will the nurse ask the patient about? A) Dietary iron intake B) Gastrointestinal (GI) upset C) Whether stools have been tarry or black D) Whether the prescription needs to be refilled

D) Whether the prescription needs to be refilled

A patient who takes aspirin for rheumatoid arthritis is admitted to the hospital complaining of headache and ringing in the ears. The plasma salicylate level is 300 mcg/mL, and the urine pH is 6.0. What will the nurse do? A) Increase the aspirin dose to treat the patient's headache. B) Notify the provider of possible renal toxicity. C) Prepare to provide respiratory support, because the patient shows signs of overdose. D) Withhold the aspirin until the patient's symptoms have subsided.

D) Withhold the aspirin until the patient's symptoms have subsided.

A patient who takes warfarin (Coumadin) is brought to the emergency department after accidentally taking too much warfarin. The patient's heart rate is 78 beats per minute and the blood pressure is 120/80 mm Hg. A dipstick urinalysis is normal. The patient does not have any obvious hematoma or petechiae and does not complain of pain. The nurse will anticipate an order for: A) vitamin K (phytonadione). B) protamine sulfate. C) a PTT. D) a PT and an INR.

D) a PT and an INR.

A nursing student asks a nurse how beta blockers increase the oxygen supply to the heart in the treatment of anginal pain. The nurse tells the student that beta blockers: A) dilate arterioles to improve myocardial circulation. B) improve cardiac contractility, which makes the heart more efficient. C) increase arterial pressure to improve cardiac afterload. D) increase the time the heart is in diastole.

D) increase the time the heart is in diastole.

A patient has a localized skin infection, which is most likely caused by a gram-positive cocci. Until the culture and sensitivity results are available, the nurse will expect the provider to order a ____ spectrum _____ agent. A) broad; systemic B) broad; topical C) narrow; systemic D) narrow; topical

D) narrow; topical

A patient begins taking nifedipine (Procardia), along with a beta blocker, to treat hypertension. The nurse understands that the beta blocker is used to: A) reduce flushing. B) minimize gingival hyperplasia. C) prevent constipation. D) prevent reflex tachycardia.

D) prevent reflex tachycardia.

A patient asks a nurse how nitroglycerin works to relieve anginal pain. The nurse correctly states, "Nitroglycerin: A) dilates coronary arteries to increase blood flow to the heart." B) increases the oxygen supply to the cardiac muscle." C) increases ventricular filling to improve cardiac output." D) promotes vasodilation, which reduces preload and oxygen demand."

D) promotes vasodilation, which reduces preload and oxygen demand."

A patient arrives in the emergency department with a heart rate of 128 beats per minute and a temperature of 105° F. The patient's skin feels hot and moist. The free T4 level is 4 ng/dL, the free T3 level is 685 pg/dL, and the TSH level is 0.1 microunits/mL. The nurse caring for this patient will expect to administer: A) intravenous levothyroxine. B) iodine-131 (131 I). C) methimazole (Tapazole). D) propylthiouracil (PTU).

D) propylthiouracil (PTU).

An older male patient comes to the clinic with complaints of chills, malaise, myalgia, localized pain, dysuria, nocturia, and urinary retention. The nurse would most likely suspect that the patient has: A) acute cystitis. B) urinary tract infection. C) pyelonephritis. D) prostatitis.

D) prostatitis.

A patient is receiving heparin postoperatively to prevent deep vein thrombosis. The nurse notes that the patient has a blood pressure of 90/50 mm Hg and a heart rate of 98 beats per minute. The patient's most recent aPTT is greater than 90 seconds. The patient reports lumbar pain. The nurse will request an order for: A) a repeat aPTT to be drawn immediately. B) analgesic medication. C) changing heparin to aspirin. D) protamine sulfate.

D) protamine sulfate.

A patient who takes warfarin for atrial fibrillation undergoes hip replacement surgery. On the second postoperative day, the nurse assesses the patient and notes an oxygen saturation of 83%, pleuritic chest pain, shortness of breath, and hemoptysis. The nurse will contact the provider to report possible _____ and request an order for _____. A) congestive heart failure; furosemide (Lasix) B) hemorrhage; vitamin K (phytonadione) C) myocardial infarction: tissue plasminogen activator (tPA) D) pulmonary embolism; heparin

D) pulmonary embolism; heparin

A patient with heart failure who has been taking an ACE inhibitor, a thiazide diuretic, and a beta blocker for several months comes to the clinic for evaluation. As part of the ongoing assessment of this patient, the nurse will expect the provider to evaluate: A) complete blood count. B) ejection fraction. C) maximal exercise capacity. D) serum electrolyte levels.

D) serum electrolyte levels.

A female patient who has hepatitis C is being treated with pegylated interferon alfa and ribavirin (Ribasphere). It will be important for the nurse to teach this patient that: A) if she gets pregnant, she should use the inhaled form of ribavirin (Virazole). B) if she is taking oral contraceptives, she should also take a protease inhibitor. C) she should use a hormonal contraceptive to avoid pregnancy. D) she will need a monthly pregnancy test during her treatment.

D) she will need a monthly pregnancy test during her treatment.

A patient with asthma will be using a metered-dose inhaler (MDI) for delivery of an inhaled medication. The provider has ordered 2 puffs to be given twice daily. It is important for the nurse to teach this patient that: A) a chlorofluorocarbon (CFC) propellant is superior to a hydrofluoroalkane (HFA) propellant. B) the patient should activate the device and then inhale. C) the patient should store the MDI in the refrigerator between doses. D) the patient should wait 1 minute between puffs.

D) the patient should wait 1 minute between puffs.

A patient stops taking a proton pump inhibitor (PPI) after 6 weeks of therapy for treatment of peptic ulcer disease. The patient reports symptoms of dyspepsia to the nurse. The nurse will tell this patient to: A) come to the clinic to be tested for Clostridium difficile. B) resume taking the PPI, because long-term therapy is necessary. C) resume taking the PPI until symptoms resolve completely. D) try an antacid to see whether it relieves these symptoms.

D) try an antacid to see whether it relieves these symptoms.

A patient about to begin therapy with etanercept has a positive tuberculin skin test. A chest radiograph is negative. The nurse will expect this patient to: A) begin taking antituberculosis drugs at the beginning of treatment with etanercept. B) have periodic chest radiographs during treatment with etanercept. C) have regular monitoring of symptoms to detect active tuberculosis. D) undergo tuberculosis treatment prior to beginning etanercept treatment.

D) undergo tuberculosis treatment prior to beginning etanercept treatment.

A patient who has begun using transdermal nitroglycerin for angina reports occasional periods of tachycardia. The nurse will expect the prescriber to order: A) digoxin (Lanoxin) to slow the heart rate. B) immediate discontinuation of the nitroglycerin. C) periods of rest when the heart rate increases. D) verapamil as an adjunct to nitroglycerin therapy.

D) verapamil as an adjunct to nitroglycerin therapy.

An older male patient with an increased risk of MI is taking furosemide (Lasix) and low-dose aspirin. The patient is admitted to the hospital, and the nurse notes an initial blood pressure of 140/80 mm Hg. The patient has had a 10-pound weight gain since a previous admission 3 months earlier. The patient has voided only a small amount of concentrated urine. The serum creatinine and blood urea nitrogen (BUN) levels are elevated. The nurse will contact the provider to discuss: A) adding an antihypertensive medication. B) obtaining serum electrolytes. C) ordering a potassium-sparing diuretic. D) withdrawing the aspirin.

D) withdrawing the aspirin.

The nurse should instruct patients about a possible systemic effect that may occur if excessive amounts of topically applied adrenergic nasal decongestants are used. Which systemic effect may occur?

D. Nervousness

While assessing a patient who is receiving intravenous digitalis, the nurse recognizes that the drug has a negative chronotropic effect. How would this drug effect be evident in the patient? Choose one answer.

Decreased heart rate

What are the effects of phenobarbital?

Drowsiness, lethargy, dizziness, hangover, and paradoxial restlessness. Barbiturates deprive people of REM sleep.

What is the most reliable measure for assessing diabetes control over a 3 - month period?

Fasting plasma glucose A1C levels prescribers order for insulin must be assessed, so that the correct drug, route, type of insulin and dosage are implemented correctly.

Describe how evaluation relates to the assessment, diagnosis, planning and implementation phases of the nursing process

If a goal is unmet, you must re-examine all steps of the nursing process (possibly adding to or altering any of them based on your findings)

Can Gabapentin be stopped all at once?

If discontinuation of the drug is indicated, dosage should be tapered over at least 1 week to avoid rebound seizures.

What is the disadvantage of self - treatment for heartburn?

Rebound Hyperacidity milk-alkali syndrome changes in systemic PH

What are the therapeutic effects of Sibutramine?

Is used to treat obesity, effects are often minimal without accompanying behavioral modifications involving diet and exercise. Appetite control and weight loss for the treatment of obesity.

How long does donepezil (Aricept) take to see improvement when taking this medication?

It could take up to 6 weeks for therapeutic results.

Why and when should glycol be used?

It is a laxative and can be used before surgery or any other medical procedure. it rids the colon of fecal matter.

During the night shift, a patient's total parenteral nutrition (TPN) infusion ran out, and there was no TPN solution on hand to continue the infusion. The nurse will have to implement measures to prevent what consequences of abruptly discontinuing TPN infusions?

Rebound hypoglycemia

Lasix

Lasix is a loop diuretic, it is used in the management of pulmonary edema and the edema associated with heart failure, liver disease, nephrotci syndrome, and ascites. It has also been used in the treatment of hypertension. Adverse effects, Hypokalemia

A patient is using a metered-dose inhaler containing albuterol for asthma. The medication label instructs the patient to administer "2 puffs every 4 hours as needed for coughing or wheezing." The patient reports feeling jittery sometimes when taking the medication, and she doesn't feel that the medication is always effective. Which is not an appropriate nursing intervention for this patient?

Suggesting that the patient use one puff to reduce side effects It is not within the nurse's scope of practice to change the dose of a medication without an order from a prescriber. Asking the patient to demonstrate inhaler use helps the nurse to evaluate the patient's ability to administer the medication properly and is part of the nurse's evaluation. Assessing tobacco smoke exposure helps the nurse determine whether nondrug therapies, such a smoke avoidance, can be used as an adjunct to drug therapy. Performing a physical assessment helps the nurse evaluate the patient's response to the medication.

Why is Mannitol (Osmitrol) used for?

Mannitol is a osmotic diuretic, used in the treatment of patients in the early, oliguric phase of acute renal failure. However, enough renal blood flow and glomerular filtration must still remain to enable the drug to reach the renal tubules. It can also be used to promote the excretion of toxic substances, reduce intracranial pressure, and treat cerebral edema. in addition, it can be used as a genitourinary irrigant in the preparation of patients for transurethral surgical procedures and as a supportive treatment in patients with edema induced by other conditions. Mannitol is not indicated for patients with peripheral edema because it does not promote sufficient sodium excretion.

What are the recommendations for a patient taking Warfarin (Coumadin) and wants to take a pain reliever for a headache

Take acetaminophen (Tylenol) avoid NSAID's

What to avoid while taking MAOI's

Opioid analgesics and St. Johns Wort and tyramine containing foods such as smoked meats, aged cheese, soy sauce, caffeinated drinks, chocolate and fruit.

Prior to providing education on a new medication to a patient and family, what is the priority step of the nursing process?

Outcome planning

Patient teaching on nasal sprays

Patient should be instructed to report excessive dizziness, heart palpitations, weakness, sedation, and / or excessive irritability to the prescriber.

What is the most important to observe from a patient that has just started SSRI's for depression?

Patients should be monitored closely for serotonin withdrawal. Should not be given to patients with coronary heart disease.

What age is a DTap is given

Pediatric only - age of first injection 6wk-7yr; give second and third doses at 4-8 wk intervals. ages are 2,4,6, and 15 yrs.

A woman has been receiving both radiation and chemotherapy for her cancer. Lately, she has developed anorexia caused by the treatments, so she needs short-term nutrition supplementation. The nurse anticipates that the physician will initiate which therapy?

Peripheral parenteral nutrition (PPN)

162. A prescriber has ordered pilocarpine (Pilocar). A nurse understands that the drug stimulates muscarinic receptors and would expect the drug to have which action? a. Reduce excessive secretions in a postoperative patient b. Lower intraocular pressure in patients with glaucoma c. Inhibit muscular activity in the bladder d. Prevent hypertensive crisis

Pilocarpine is a muscarinic agonist used mainly for topical therapy of glaucoma to reduce intraocular pressure. Pilocarpine is not indicated for the treatment of excessive secretions and mucus; in fact, pilocarpine is used to treat dry mouth. Pilocarpine does not inhibit muscular activity in the bladder. Pilocarpine is not used to prevent hypertensive crisis.

What are the advantages for a patient to take captopril (Capoten) with severe liver disease?

Pro-drugs are inactive in their administered form and must be metabolized in the liver to an active form so as to be effective. Captopril and lisinopril can be used if a patient has liver dysfunction, unlike other ACE inhibitors that are pro-drugs

Nurse prepares teaching plan regarding administration of eardrops for parents of 2 yr. old child. Which would be included?

Pull earlobe down and back before instilling eardrops. R: When administering eardrops to child who is less than 3, ear s/be pulled down and back. For children who are more than 3, ear is pulled up and back. Hand washing needs to be performed before and after procedure. Child s/be in a side-lying position w/the affected ear facing upward to facilitate flow of medication down ear canal by gravity.

Why would a patient be taking multi -drug therapy for hypertension?

Resistant Hypertension; two drugs work better to lower blood pressure.

Patient teaching on Omeprazole (Prilosec)

This is an antisecretory drug, works best when taken 30 to 60 minutes before meals. Must know list of current medications. Omeprazole should be taken whole and not crushed, opened, or chewed.

What is the purpose of the planning phase of the nursing process?

Setting priorities and developing patient goals and outcomes* To design a plan of care for and with the patient that, once implemented , results in the prevention, reduction, or resolution of patient health problems and the attainment of the patient's health expectations as identified in the patient outcomes

Why would a patient have to receive digoxin immune Fab (Digibind)?

Should toxicity occur and digoxin rise to a life-threatening level, the antidote, digoxin immune fab, should be administered. It is given parenterally over 30 minutes, and in some scenarios it is given as an intravenous bolus. Digoxin levels are 0.5 -2

A patient with an intestinal infection that is positive for the Giardia lamblia organism will be taking an antiprotozoal drug. Which of the following should the nurse include in the teaching plan for this patient?

Taking the medications with food reduces gastrointestinal upset.

patient has prescriptions for two inhalers. One inhaler is a bronchodilator, and the other is a corticosteroid. Which instruction regarding these inhalers should the nurse give to the patient? Choose one answer.

The bronchodilator should be taken first."

Why would a patient take 81mg of aspirin a day?

To keep blood thin prevent thrombosis formation stroke prevention used as a prophylaxis against transient ischemic attacks. used to treat angina pain

Why would a patient be receiving allopurinol (Zyloprim) when the patient is already receiving cyclosporine (Neoral)?

To prevent uric acid production

What are the principles of neurmuscular blocking drugs such as pancuronium?

These drugs are given primarily with general anesthetics to facilitate endotracheal intubation and to relax the skeletal muscles during surgery. In addition to their use in the OR, they are given in the ICU to paralyze mechanically ventilated patients.

What is an advantage of taking catechol - O- methyltransferase inhibitors while treating parkinsons disease?

They have been shown to have a greater efficacy in patients with advanced forms of Parkinsons disease. After treatment using the various dosage forms of levodopa or carbidopa - levodopa, a COMT inhibitor may then be added to the therapeutic regimen, and onset of therapeutic effects is rapid.

Patient teaching for a patient that has been ordered atenolol (Tenormin)

This bet blocker is commonly used to prevent future heart attacks in patients who have had one. it is also used in the treatment of hypertension and angina. Apical pulse should be counted for one full minute and both supine and standing blood pressures should be measured and documented. Patients should weight themselves daily, avoid sudden changes in position, and increase fluids and fiber. Alcohol should be avoided.

What does the "cheese effect" result in ? (Selegiline)

This drug interacts with tyramine - containing foods such as (cheese, red wine, beer, and yogurt) because of their inhibitory activity against MAO-A. And one hazardous result can be hypertension.

Levodopa

This drug is a dopamine replacement drug. Dopamine must be administered orally as levodopa, because exogenously administered dopamine cannot pass through the blood brain barrier. Levodopa is given in combination with carbidopa because very large oral doses of levodopa must be given to obtain adequate dopamine replacement in the brain. When given in combination with levodopa, carbidopa inhibits the breakdown of levodopa in the periphery and thus allows smaller doses of levodopa to be used.

How would a nurse manage break through pain?

This is administered between doses of pain medications.

Nurses instructs a patient who is ordered ocular cyclosporine (Restasis) and an artificial tears product.

Use the Restasis first,then use the artificial tears

Patient teachings on Allegra

When Allegra is given with erythromycin, increased fexofenadine concentrations can result. Its an antihistamine that is given to relieve the symptoms of the common cold, sneezing, and runny nose, and treatment for allergies. Reduces salivary, gastric, lacrimal, and bronchial secretions. They are used for motion sickness, Parkinsons disease, and vertigo.and sometimes used as a sleep aid. Allegra is not recommended for children under the age of 6 yrs or those with renal impairment. Caution should be addressed in patient with impaired liver function or renal insufficiency as well as lactating mothers.

When would a patient be likely to see a change in his serum cholesterol when taking statin drugs?

When these drugs are taken, lipid levels may not be lowered to their maximum extent until 6 to 8 weeks after the start of therapy.

241. A patient is given a prescription for azithromycin (Zithromax) and asks the nurse why the dose on the first day is twice the amount of the dose on the next 4 days. Which reply by the nurse is correct? a. "A large initial dose helps to get the drug to optimal levels in the body faster." b. "The first dose is larger to minimize the first pass effect of the liver." c. "The four smaller doses help the body taper the amount of drug more gradually." d. "Tubular reabsorption is faster with initial doses, so more is needed at first."

a. "A large initial dose helps to get the drug to optimal levels in the body faster." A large initial dose is often used as a loading dose to help get serum drug levels to plateau levels more quickly. Larger doses do not prevent first pass effects in drugs susceptible to this type of metabolism. Tapering of doses sometimes is used to prevent rebound or withdrawal effects and is done by stepping down the amount of drug with each dose. Tubular reabsorption is a process that allows drugs to be reabsorbed from the urine into the blood.

141. A nursing student asks why albuterol, which is selective for beta2 receptors, causes an increased heart rate in some patients. How does the nurse respond? a. "Adrenergic agonists can lose their selectivity when given at higher doses." b. "Bronchodilation lowers blood pressure, which causes a reflex tachycardia." c. "Some patients metabolize the drug differently and have unusual side effects." d. "Systemic effects are intensified with inhaled doses."

a. "Adrenergic agonists can lose their selectivity when given at higher doses." Although albuterol affects the beta2 receptors, at higher doses it may also activate beta1 receptors, causing an increase in the heart rate. Bronchodilation does not lower the blood pressure, and activation of beta1 receptors actually may increase it. Tachycardia is a common side effect. The inhaled route reduces the likelihood of systemic side effects.

137. A nurse is teaching parents how to use an Epi-Pen for their child, who has a peanut allergy. Which statement by the parents indicates understanding of the teaching? a. "After using the Epi-Pen, we must go to the emergency department." b. "The Epi-Pen should be stored in the refrigerator, because epinephrine is sensitive to heat." c. "The teacher should call us when symptoms start so that we can bring the Epi-Pen to school." d. "We should jab the device into the thigh until it is empty of solution."

a. "After using the Epi-Pen, we must go to the emergency department." After using the Epi-Pen, it is important that the individual get medical attention as quickly as possible. The effects of epinephrine fade in 10 to 20 minutes, and the anaphylactic reaction can be biphasic and prolonged. Epinephrine is sensitive to heat, but storing the device in the refrigerator can compromise the injection mechanism; the device should be stored at room temperature in a dark place. Individuals who need an Epi-Pen must have the device with them at all times; any delay in treatment can be fatal, because anaphylaxis can occur within minutes after exposure. The Epi-Pen contains 2 mL of epinephrine, but only 0.3 mL is injected; the device will not be empty with a successful injection.

194. A nurse is teaching a group of nursing students about administering medications to older adult patients. Which statement by a student indicates a need for further teaching? a. "Alteration in hepatic function requires more frequent drug dosing." b. "Changes in GI function in older adult patients lead to lower serum drug levels." c. "Most adverse drug reactions in older adult patients are related to altered renal function." d. "Most nonadherence among older adult patients is intentional."

a. "Alteration in hepatic function requires more frequent drug dosing." Changes in hepatic function in older adult patients lead to decreased metabolism, meaning that drugs metabolized by the liver have prolonged half-lives and should be given less frequently. Altered GI function does not have much effect in this population, but most known effects from this cause are related to poor absorption and less available drug. Alterations in renal function are the cause of most adverse drug effects in the older adult. In most cases, nonadherence to drug regimens is intentional, usually because the patient doesn't believe that the drug is needed or that the dose prescribed is not necessary.

148. A nurse is reviewing nursing actions for emergency treatment of malignant hyperthermia with a group of nursing students. Which statement by a student indicates a need for further teaching? a. "Analgesics should be given to help with muscle pain." b. "Dantrolene helps to slow the metabolic activity in skeletal muscles." c. "If malignant hyperthermia occurs, the succinylcholine must be stopped immediately." d. "The patient may need an intravenous infusion of cold saline."

a. "Analgesics should be given to help with muscle pain." Malignant hyperthermia is a rare but potentially fatal adverse reaction that can be triggered by succinylcholine. Muscle pain is an expected adverse effect of succinylcholine that is thought to be caused by the muscle contractions that occur during induction; analgesics can be given, but this effect usually occurs 10 to 12 hours after the procedure and is not an emergency. Dantrolene is given because it slows metabolic activity in skeletal muscles, resulting in decreased heat production. Succinylcholine should be stopped immediately if malignant hyperthermia occurs. Cold saline, given intravenously, often is necessary to lower a patient's temperature

128. A patient will begin taking propranolol (Inderal) for hypertension. Which statement by the nurse is important when teaching this patient about the medication? a. "Check your hands and feet for swelling and report that to your provider." b. "It is safe to take this medication with a calcium channel blocker." c. "Stop taking the drug if you become short of breath." d. "Take your pulse and do not take the medication if your heart rate is fast."

a. "Check your hands and feet for swelling and report that to your provider." Patients taking propranolol can develop heart failure because of the suppression in myocardial contractility. Patients should be taught to watch for signs, which include shortness of breath, night coughs, and swelling of the extremities. Use of these agents with calcium channel blockers is contraindicated, because the effects are identical and excessive cardiosuppression can occur. Shortness of breath should be reported to the provider, but abrupt cessation of the drug can cause rebound cardiac excitability. Propranolol reduces the heart rate and should not be given if the pulse is less than 60 beats per minute.

112. A nurse is teaching a patient about a new prescription for reserpine (Serpasil) for hypertension. Which statement by the patient indicates the need for further teaching? a. "Depressive side effects are common and will improve over time." b. "I should report gastrointestinal side effects to the provider." c. "I should stand up slowly when getting up and lie down if I feel dizzy." d. "Therapeutic effects may not be optimal for a couple of weeks."

a. "Depressive side effects are common and will improve over time." Reserpine can produce severe depression, which may persist for months during drug therapy and after the drug is withdrawn. Reserpine can stimulate the secretion of gastric acid, which can cause ulcers; it also can increase the tone and motility of intestinal smooth muscle, which can cause cramps and diarrhea. Reserpine can cause orthostatic hypotension, so patients should be counseled to rise slowly when standing up and to sit or lie down if they feel dizzy. Reserpine works by depleting norepinephrine, and the processes necessary for this may take 1 to 2 weeks.

110. A patient complains to the nurse that the clonidine (Catapres) recently prescribed for hypertension is causing drowsiness. Which response by the nurse to this concern is appropriate? a. "Drowsiness is a common side effect initially, but it will lessen with time." b. "You may also experience orthostatic hypotension along with the drowsiness." c. "You may be at risk for addiction if you have central nervous system side effects." d. "You should discontinue the medication and contact your prescriber."

a. "Drowsiness is a common side effect initially, but it will lessen with time." CNS depression, evidenced in this patient by drowsiness, is common in about 35% of the population. These responses become less intense with continued drug use. Orthostatic hypotension is less likely with clonidine, because its antihypertensive effects are not posture dependent. The experience of drowsiness does not indicate abuse potential. The patient should not discontinue the medication abruptly because of the potential for rebound hypertension; the patient should contact the prescriber before stopping the medication.

180. A nurse is teaching a group of nursing students about neuropharmacology. Which statement by a student about peripheral nervous system (PNS) drugs indicates a need for further teaching? a. "Drugs affecting axonal conduction have a variety of uses." b. "Drugs that alter synaptic transmission can be highly selective." c. "Many PNS drugs act by altering synaptic transmission." d. "These drugs work by influencing receptor activity."

a. "Drugs affecting axonal conduction have a variety of uses." Local anesthetics are the only drugs shown to work by altering axonal conduction. Any drug affecting axonal conduction would be nonselective, because axonal conduction of impulses is essentially the same in all neurons. In contrast, drugs that affect synaptic transmission can be highly selective, because each transmitter has different effects on receptor sites. Most PNS drugs work on synaptic transmission processes. Through their effects on transmitters, they influence receptor activity.

129. A patient with migraines is started on a beta blocker. The nurse explains the benefits of taking the medication for migraines. Which statement by the patient indicates an understanding of the medication's effects? a. "I need to take it every day to reduce the frequency of migraines." b. "I will take it as needed to get relief from migraines." c. "I will take it to shorten the duration of my migraines." d. "I will take this drug when a migraine starts."

a. "I need to take it every day to reduce the frequency of migraines." When taken prophylactically, beta blockers can reduce the frequency of migraine attacks. Beta blockers do not provide complete relief from migraines. They do not reduce the duration of migraines. They are not effective for treating migraines once the migraine has begun.

108. A prescriber has ordered clonidine (Catapres) for a patient who has hypertension. The nurse teaches the patient about side effects of this drug. Which statement by the patient indicates understanding of the teaching? a. "I should chew sugar-free gum or drink water to reduce dry mouth." b. "I should not drive as long as I am taking this drug." c. "I should stand up slowly when taking this medication." d. "I should stop taking this drug if I feel anxious or depressed."

a. "I should chew sugar-free gum or drink water to reduce dry mouth." Xerostomia is a common side effect of clonidine and is often uncomfortable enough that patients stop using the drug. Counseling patients to chew sugar-free gum and take frequent sips of liquid can help alleviate this discomfort. Drowsiness is common, but this side effect becomes less intense over time. Patients should be counseled to avoid hazardous activities in the first weeks of therapy if they feel this effect. The hypertensive effects of clonidine are not posture dependent, as they are with the peripheral alpha-adrenergic blockers, so orthostatic hypotension is minimal with this drug. Clonidine causes euphoria, hallucinations, and sedation in high doses and can cause anxiety or depression, although the last two effects are less common. The drug should not be stopped abruptly because of the risk of rebound hypertension, so patients experiencing unpleasant central nervous system (CNS) effects should consult their provider about withdrawing the medication slowly.

171. A nurse is teaching a patient about a medication that alters sympathetic nervous system functions. To evaluate understanding, the nurse asks the patient to describe which functions the sympathetic nervous system regulates. Which answer indicates the need for further teaching? a. "The digestive functions of the body" b. "The cardiovascular system" c. "The fight-or-flight response" d. "Body temperature"

a. "The digestive functions of the body" The sympathetic nervous system does not regulate digestive functions of the body—the parasympathetic nervous system does; further education is needed. The sympathetic nervous system regulates the cardiovascular system, the fight-or-flight response, and the body temperature; no further education is needed.

230. The prescriber has ordered an antibiotic for a patient with a bacterial infection. The nurse provides patient education at discharge and instructs the patient to take the drug on an empty stomach. When should the patient take the drug? (Select all that apply.) a. 1 hour or more before a meal b. Only after an 8-hour fast c. Only after the patient has missed a meal d. At least 2 hours after a meal e. Within 12 hours with liquids but not solids

a. 1 hour or more before a meal d. At least 2 hours after a meal The absorption of some drugs can be significantly reduced by food; these drugs should be taken on an empty stomach, which is 1 hour or more before a meal or at least 2 hours after a meal. An 8-hour fast is not necessary; the patient does not need to miss a meal to take the medication; and it is not reasonable to have the patient on thin liquids for 12 hours

262. A nurse is preparing to administer medications. Which patient would the nurse consider to have the greatest predisposition to an adverse reaction? a. A 30-year-old man with kidney disease b. A 75-year-old woman with cystitis c. A 50-year-old man with an upper respiratory tract infection d. A 9-year-old boy with an ear infection

a. A 30-year-old man with kidney disease The individual with impaired kidney function would be at risk of having the drug accumulate to a toxic level because of potential excretion difficulties. Cystitis is an infection of the bladder and not usually the cause of excretion problems that might lead to an adverse reaction from a medication. A respiratory tract infection would not predispose a patient to an adverse reaction, because drugs are not metabolized or excreted by the lungs. A 9-year-old boy would not have the greatest predisposition to an adverse reaction simply because he is a child; nor does an ear infection put him at greater risk.

249. A nurse educator is conducting a continuing education class on pharmacology. To evaluate the learning of the nurses in the class, the nurse educator asks, "Which drug name is a generic drug name?" Which is the correct response? a. Acetaminophen b. Tylenol c. Cipro d. Motrin

a. Acetaminophen Acetaminophen is the generic name. Tylenol, Cipro, and Motrin are all trade names.

207. A patient is given a new medication and reports nausea within an hour after taking the drug. The nurse consults the drug information manual and learns that nausea is not an expected adverse effect of this drug. When the next dose is due, what will the nurse do? a. Administer the drug and tell the patient to report further nausea. b. Hold the drug and notify the provider of the patient's symptoms. c. Report the symptoms of nausea to the MEDWATCH program. d. Request an order for an antiemetic to counter this drug's effects.

a. Administer the drug and tell the patient to report further nausea. Not all adverse drug reactions (ADRs) can be detected during clinical trials, and nurses should be alert to any effects that may result from drug administration. Because nausea is not a serious effect and because it is not yet known whether the drug is the cause of this patient's nausea, the nurse should administer the medication and observe the patient for recurrence of the symptom. It is not necessary to hold the drug, because nausea is not a serious side effect. The MEDWATCH program should be notified when there is a greater suspicion that the drug may have caused the nausea if the nausea occurs with subsequent doses. Until there is greater suspicion that the drug actually caused this patient's nausea, giving an antiemetic is not indicated.

125. A male patient is being treated for benign prostatic hyperplasia and has stopped taking his alpha-adrenergic antagonist medication because of ejaculatory difficulties. Which medication does the nurse expect the provider to prescribe? a. Alfuzosin (Uroxatral) b. Prazosin (Minipress) c. Silodosin (Rapaflo) d. Tamsulosin (Flomax)

a. Alfuzosin (Uroxatral) Alfuzosin is used for BPH and does not interfere with ejaculation. All of the other drugs have ejaculatory side effects. Prazosin may be useful for BPH, but it is not approved for this use.

142. A patient is receiving dobutamine (Dobutrex) as a continuous infusion in the immediate postoperative period. The patient also is receiving a diuretic. What adverse drug reactions are possible in this patient? (Select all that apply.) a. Angina b. Dysrhythmias c. Hypotension d. Oliguria e. Tachycardia

a. Angina b. Dysrhythmias e. Tachycardia Angina, dysrhythmias, and tachycardia are the most common adverse effects of dopamine; general anesthetics can increase the likelihood of dysrhythmias. Dopamine elevates blood pressure by increasing cardiac output. Diuretics complement the beneficial effects of dopamine on the kidney, so urine output would be increased, not decreased.

237. Which drugs will not be affected by interpatient variability? (Select all that apply.) a. Antiseptics applied to the skin to slow bacterial growth b. Antacids to help with the discomfort of heartburn c. Broad-spectrum antibiotics that are effective against many organisms d. Chelating agents that remove metal compounds from the body e. Topical analgesics used to treat localized pain

a. Antiseptics applied to the skin to slow bacterial growth b. Antacids to help with the discomfort of heartburn d. Chelating agents that remove metal compounds from the body Antiseptics, antacids, and chelating agents are all receptorless drugs that do not depend on the body's processes for effects; these agents react with other molecules. Broad-spectrum antibiotics and topical analgesics bind with receptors to produce desired effects, and these processes can be influenced by individual patient variables.

229. A nurse is caring for a patient who is taking multiple medications. To help ensure that adverse drug reactions are prevented or minimized, the nurse will do which of the following? (Select all that apply.) a. Ask the patient about over-the-counter medications used. b. Contact the prescriber to request cytochrome P450 levels. c. Limit the patient's calcium intake. d. Obtain a thorough diet history. e. Request orders for PRN medications to treat any anticipated symptoms of drug interactions.

a. Ask the patient about over-the-counter medications used. b. Contact the prescriber to request cytochrome P450 levels. d. Obtain a thorough diet history. Over-the-counter medications add to drug interactions, and a thorough history of all medications taken by the patient is essential to minimize adverse drug reactions. Cytochrome P450 levels yield important information about a patient's ability to metabolize drugs and can help predict whether drugs will reach toxic levels or be ineffective. A diet history allows providers to anticipate significant known food-drug interactions. Limiting calcium intake is necessary only if the patient is taking drugs known to interact with calcium, such as tetracycline. Asking for PRN medications to treat drug reactions may only compound the risk, because the risk of drug interactions increases with the number of medications taken.

261. A patient tells a nurse that a medication prescribed for recurrent migraine headaches is not working. What will the nurse do? a. Ask the patient about the number and frequency of tablets taken. b. Assess the patient's headache pain on a scale from 1 to 10. c. Report the patient's complaint to the prescriber. d. Suggest biofeedback as an adjunct to drug therapy.

a. Ask the patient about the number and frequency of tablets taken. When evaluating the effectiveness of a drug, it is important to determine whether the patient is using the drug as ordered. Asking the patient to tell the nurse how many tablets are taken and how often helps the nurse determine compliance. Assessing current pain does not yield information about how well the medication is working unless the patient is currently taking it. The nurse should gather as much information about compliance, symptoms, and drug effectiveness as possible before contacting the prescriber. Biofeedback may be an effective adjunct to treatment, but it should not be recommended without complete information about drug effectiveness.

136. A patient with asthma uses albuterol (Ventolin) for wheezing. The nurse assesses the patient and notes vital signs of HR, 96 beats per minute; RR, 18 breaths per minute; and BP, 116/78 mm Hg. The patient has clear breath sounds and hand tremors. What will the nurse do? a. Ask the patient how often the inhaler is used. b. Check the patient's blood glucose level. c. Request an order for isoproterenol (Isuprel) to reduce side effects. d. Stop the medication and report the tremors to the provider.

a. Ask the patient how often the inhaler is used. Tachycardia is a response to activation of beta1 receptors. It can occur when large doses of albuterol are used and selectivity decreases. The nurse should question the patient about the number of inhalations used. Isoproterenol can cause hyperglycemia in diabetic patients. Isoproterenol has more side effects than albuterol. Tremors are an expected side effect and are not an indication for stopping the drug.

211. A nurse is preparing to give an antibiotic to a patient who reports being allergic to antibiotics. Before giving the medication, it is important for the nurse to do what? a. Ask whether the patient has taken this antibiotic for other infections. b. Question the patient about allergies to other medications. c. Obtain a history of other reactions to other drugs. d. Request an order for an antihistamine.

a. Ask whether the patient has taken this antibiotic for other infections. The nurse needs to assess whether the patient is truly allergic to this drug. Allergic reactions require previous exposure to the drug, so the nurse should ask whether the patient has taken this antibiotic before. Obtaining a history of drug allergies and of reactions to medications also is important, but not as important as finding out about possible allergic reactions to a drug about to be given. Antihistamines sometimes are given when patients must take a drug to which they are allergic.

259. A patient newly diagnosed with diabetes is to be discharged from the hospital. The nurse teaching this patient about home management should begin by doing what? a. Asking the patient to demonstrate how to measure and administer insulin b. Discussing methods of storing insulin and discarding syringes c. Giving information about how diet and exercise affect insulin requirements d. Teaching the patient about the long-term consequences of poor diabetes control

a. Asking the patient to demonstrate how to measure and administer insulin Because insulin must be given correctly to control symptoms and because an overdose can be fatal, it is most important for the patient to know how to administer it. Asking for a demonstration of technique is the best way to determine whether the patient has understood the teaching. When a patient is receiving a lot of new information, the information presented first is the most likely to be remembered. The other teaching points are important as well, but they are not as critical and can be taught later.

260. The nurse receives an order to give morphine 5 mg IV every 2 hours PRN pain. Which action is not part of the six rights of drug administration? a. Assessing the patient's pain level 15 to 30 minutes after giving the medication b. Checking the medication administration record to see when the last dose was administered c. Consulting a drug manual to determine whether the amount the prescriber ordered is appropriate d. Documenting the reason the medication was given in the patient's electronic medical record

a. Assessing the patient's pain level 15 to 30 minutes after giving the medication Assessing the patient's pain after administering the medication is an important part of the nursing process when giving medications, but it is not part of the six rights of drug administration. Checking to see when the last dose was given helps ensure that the medication is given at the right time. Consulting a drug manual helps ensure that the medication is given in the right dose. Documenting the reason for a pain medication is an important part of the right documentation—the sixth right.

164. A prescriber has ordered bethanechol (Urecholine) for a postoperative patient who has urinary retention. The nurse reviews the patient's chart before giving the drug. Which part of the patient's history would be a contraindication to using this drug? a. Asthma as a child b. Gastroesophageal reflux c. Hypertension d. Hypothyroidism

a. Asthma as a child Bethanechol is contraindicated in patients with active or latent asthma, because activation of muscarinic receptors in the lungs causes bronchoconstriction. It increases the tone and motility of the gastrointestinal (GI) tract and is not contraindicated in patients with reflux. It causes vasodilation and would actually lower blood pressure in a hypertensive patient. It causes dysrhythmias in hyperthyroid patients.

228. The nurse is administering morning medications. The nurse gives a patient multiple medications, two of which compete for plasma albumin receptor sites. As a result of this concurrent administration, the nurse can anticipate that what might occur? (Select all that apply.) a. Binding of one or both agents is reduced. b. Plasma levels of free drug will rise. c. Plasma levels of free drug will fall. d. The increase in free drug can intensify effects. e. The increase in bound drug can intensify effects.

a. Binding of one or both agents is reduced. b. Plasma levels of free drug will rise. d. The increase in free drug can intensify effects. When two drugs bind to the same site on plasma albumin, coadministration of those drugs produces competition for binding. As a result, binding of one or both agents is reduced, causing plasma levels of free drug to rise. The increase in free drug can intensify the effect, but it usually undergoes rapid elimination. The increase in plasma levels of free drug is rarely sustained.

183. A nurse learns about a drug that interferes with transmitter storage in the PNS. The transmitter affected by this drug causes an increased heart rate. What response will the nurse expect to see when this drug is administered? a. Bradycardia b. Positive inotropic effects c. Prolonged receptor activation d. Tachycardia

a. Bradycardia Drugs that interfere with transmitter storage reduce receptor activation, because disruption of storage decreases the amount of transmitter available for release. Because this transmitter increases the heart rate, the result will be a decrease in the heart rate. Inotropic effects control the force of contraction, not the rate of contraction. Decreased transmitter storage would result in decreased receptor activation. Tachycardia would occur if transmitter availability were increased.

227. The nurse is providing multiple medications to a patient whose spouse brings him grapefruit juice every morning. The nurse should be concerned about which classes of drugs? (Select all that apply.) a. Calcium channel blockers b. Selective serotonin reuptake inhibitors c. Aminoglycosides d. Beta blockers e. Penicillins

a. Calcium channel blockers b. Selective serotonin reuptake inhibitors Calcium channel blockers and selective serotonin reuptake inhibitors have been shown to reach increased and/or toxic levels when taken with grapefruit juice. Grapefruit juice is not contraindicated with aminoglycosides, beta blockers, or penicillins.

150. Small doses of tubocurarine are administered when patients are suspected of having a myasthenia gravis crisis. If the patient develops increased muscle weakness in response to the tubocurarine, the nurse should prepare to administer what kind of drug? a. Cholinesterase inhibitor b. Dopamine agonist c. Ganglionic blocker d. Neuromuscular blocking agent

a. Cholinesterase inhibitor If the patient displays increased muscle weakness, this is an adverse effect of the tubocurarine, and respiratory depression may ensue. A dopamine antagonist would not be indicated to reverse the effects of the tubocurarine. Ganglionic blockers interrupt impulse transmission through ganglia of the autonomic nervous system; they are nonselective and no longer have therapeutic uses. A neuromuscular blocking agent would increase muscle paralysis and make the condition worse.

114. A patient with hypertension has a previous history of opioid dependence. Which medication would the nurse question? a. Clonidine (Catapres) b. Guanabenz (Wytensin) c. Methyldopa d. Reserpine (Serpasil)

a. Clonidine (Catapres) Patients who abuse cocaine, opioids, and other such drugs also frequently abuse clonidine, so this agent would not be the best choice for this patient. The other drugs do no share this abuse potential.

231. A patient is receiving digoxin twice daily. When assessing the patient before giving a dose, the nurse counts a pulse of 60 beats per minute and learns that the patient is experiencing nausea. The nurse consults a drug manual and verifies that the ordered dose is correct. What will the nurse do? a. Contact the prescriber to report the symptoms. b. Delay the dose so the drug can clear from receptor sites. c. Give the medication as ordered, because the dose is correct. d. Request an antinausea medication from the prescriber.

a. Contact the prescriber to report the symptoms. The symptoms indicate toxicity, and even though the dose is safe and effective in most cases, an individual patient may have toxic effects with a standard dose. The nurse should contact the prescriber to discuss the next steps. Delaying a dose without a change in order is not within the scope of practice for a nurse. The nurse should not give a dose of a medication when toxicity is suspected, because additional drug will compound the symptoms. Antiemetics are useful for counteracting drug side effects, but they should not be used when the patient's symptoms indicate toxicity.

214. A patient is taking sertraline (Zoloft) for depression, and the provider orders azithromycin (Zithromax) to treat an infection. What will the nurse do? a. Contact the provider to discuss an alternative to azithromycin. b. Request an order for a different antidepressant medication. c. Request an order to reduce the dose of sertraline. d. Withhold the sertraline while giving the azithromycin.

a. Contact the provider to discuss an alternative to azithromycin. Both sertraline and azithromycin prolong the QT interval, and when taken together, they increase the risk of fatal dysrhythmias. Because the antibiotic is used for a short time, it is correct to consider using a different antibiotic. Reducing the dose of sertraline does not alter the combined effects of two drugs that lengthen the QT interval. Sertraline should not be stopped abruptly, so withholding it during antibiotic therapy is not indicated.

263. A nurse consults a drug manual before giving a medication to an 80-year-old patient. The manual states that elderly patients are at increased risk for hepatic side effects. Which action by the nurse is correct? a. Contact the provider to discuss an order for pretreatment laboratory work. b. Ensure that the drug is given in the correct dose at the correct time to minimize the risk of adverse effects. c. Notify the provider that this drug is contraindicated for this patient. d. Request an order to give the medication intravenously so that the drug does not pass through the liver.

a. Contact the provider to discuss an order for pretreatment laboratory work. The drug manual indicates that this drug should be given with caution to elderly patients. Getting information about liver function before giving the drug establishes baseline data that can be compared with post-treatment data to determine whether the drug is affecting the liver. Giving the correct dose at the correct interval helps to minimize risk, but without baseline information, the effects cannot be determined. The drug is not contraindicated.

116. A prescriber has ordered methyldopa for a female patient with hypertension. The nurse understands that which laboratory tests are important before beginning therapy with this drug? (Select all that apply.) a. Coombs' test b. Hemoglobin and hematocrit (H&H) c. Liver function tests d. Pregnancy test e. Urinalysis

a. Coombs' test b. Hemoglobin and hematocrit (H&H) c. Liver function tests A positive Coombs' test result occurs in 10% to 20% of patients who take methyldopa chronically. A few of these patients (5%) develop hemolytic anemia. Blood should be drawn for a Coombs' test and an H&H before treatment is started and at intervals during treatment. Because methyldopa is associated with liver disorders, liver function tests should be performed before therapy is started and periodically during treatment. Clonidine, not methyldopa, is contraindicated during pregnancy. A urinalysis is not indicated.

187. A nurse is concerned about renal function in an 84-year-old patient who is taking several medications. What should the nurse assess? a. Creatinine clearance b. Sodium levels c. Potassium levels d. Serum creatinine

a. Creatinine clearance The proper index of renal function in older adults is creatinine clearance, which indicates renal function in older patients whose organs are undergoing age-related deterioration. Sodium and potassium levels are not indicative of renal function. Serum creatinine levels do not reflect kidney function in older adults because lean muscle mass, which is the source of creatinine in serum, declines and may be low even with reduced kidney function.

197. A thin older adult woman is admitted to the hospital after several days of vomiting, diarrhea, and poor intake of foods and fluids. She has not voided since admission. In preparing to care for this patient, the nurse will look for what laboratory values to help guide medication administration? (Select all that apply.) a. Creatinine clearance b. Gastric pH c. Plasma drug levels d. Serum albumin e. Serum creatinine

a. Creatinine clearance c. Plasma drug levels d. Serum albumin Creatinine clearance is the best way to evaluate renal function in the older adult. Plasma drug levels are important for determining if the patient has toxic or subtherapeutic drug levels. Serum albumin may be decreased, especially in patients who are thin, chronically undernourished, or have been vomiting, and the decreased level may result in higher levels of drugs that normally bind to proteins. Gastric pH is not important; most GI changes result in lowered absorption and less free drug. Serum creatinine levels are related to the amount of lean muscle mass, which may be low in older adult patients, and do not reflect renal function.

218. Which are effective ways to help prevent medication errors? (Select all that apply.) a. Developing nonpunitive approaches to track errors b. Focusing on caregivers who make errors c. Helping patients to be active, informed members of the healthcare team d. Naming, blaming, and shaming those who make errors e. Using electronic medical order entry systems

a. Developing nonpunitive approaches to track errors c. Helping patients to be active, informed members of the healthcare team e. Using electronic medical order entry systems To help prevent medication errors, it is important to create an environment for tracking errors that is nonpunitive so that caregivers can learn from mistakes and work together to change systems appropriately. Helping patients be active, informed members of the healthcare team is a useful tool in this process. Using electronic order entry helps eliminate confusion from poor handwriting and allows built-in systems to warn caregivers about possible overdoses, side effects, and drug interactions; it also helps ensure the right dose at the right time to the right patient. An approach that focuses on those who make mistakes by naming, blaming, and shaming is not productive and often results in personnel who cover up mistakes instead of working to make things better.

193. An older adult patient is admitted to the hospital for treatment of an exacerbation of a chronic illness. Admission laboratory work reveals an extremely low serum drug level of the drug used to treat this condition. The patient has brought the medication to the hospital, along with other medications taken. The patient's renal and hepatic function tests are normal. What might the nurse suspect as a likely cause of this finding? a. Financial concerns b. Inability to open drug containers c. Increased tolerance to the drug's effects d. Patient's conviction that the drug is unnecessary

a. Financial concerns Older adult patients who have financial concerns about paying for medications often take less of the drug or take it less often to make the drug last longer. A patient unable to open the drug container would not get any medication and would not have a detectable serum drug level. A patient with increased tolerance to a drug's effects would require more of the drug to get effects. A patient convinced that the drug is not needed would probably not fill the prescription.

When counseling a male patient about the possible adverse effects of antihypertensive drugs, the nurse should discuss which potential problem?

a. Impotence

118. A patient with pheochromocytoma is admitted for surgery. The surgeon has ordered an alpha-blocking agent to be given preoperatively. What does the nurse understand about this agent? a. It is ordered to prevent perioperative hypertensive crisis. b. It prevents secretion of catecholamines by the adrenal tumor. c. It reduces contraction of smooth muscles in the adrenal medulla. d. It is given chronically after the surgery to prevent hypertension.

a. It is ordered to prevent perioperative hypertensive crisis. Manipulation of the adrenal tumor in patients with pheochromocytoma can cause a massive catecholamine release. Alpha-adrenergic antagonists are given to reduce the risk of acute hypertension during surgery. These agents do not prevent secretion of catecholamines; they block catecholamine receptor sites. They do not act on the tissue of the adrenal medulla. They are given chronically in patients who have inoperable tumors.

In assessing a pt before administration of cardiac glycoside, the nurse knows that which lab result can increase the toxicity of the drug?

a. K+ level of 2.8

221. In a discussion of drug-drug interactions, which would be the best example of a beneficial inhibitory interaction? a. Naloxone (Narcan) blocking morphine sulfate's actions b. Antacids blocking the action of tetracycline (Sumycin) c. Alcohol blocking the actions of opioids d. Cholestyramine blocking the actions of antihypertensive drugs

a. Naloxone (Narcan) blocking morphine sulfate's actions A respiratory rate of 8 breaths per minute indicates respiratory depression, which is a significant adverse effect indicating morphine toxicity. Naloxone blocks the actions of morphine at cell receptor sites and is given to quickly reverse the effects. This patient does not have signs of an allergic response, which would include shortness of breath, a rapid respiratory rate, and wheezing. The tachycardia might be a sign of worsening pain, but the toxic effects must be treated first. Patients who are sleeping are not always pain free.

206. Which groups of people are especially sensitive to medication effects? (Select all that apply.) a. Older adults b. Caucasians c. Infants d. Minorities e. Women

a. Older adults c. Infants Older adults and infants are the two groups most sensitive to drugs because of differences in organs that absorb, metabolize, and excrete drugs. In the older adult, organ degeneration accounts for these differences, whereas in infants the differences are related to organ immaturity. Racial and gender differences tend to be related to genetic differences and not race and gender per se. These groups are more sensitive to drug effects in some cases and less sensitive in other cases.

113. A patient who has been taking clonidine (Catapres) for several weeks complains of drowsiness and constipation. What will the nurse do? a. Recommend that the patient take most of the daily dose at bedtime. b. Suggest asking the provider for a transdermal preparation of the drug. c. Suspect that the patient is overusing the medication. d. Tell the patient to stop taking the drug and call the provider.

a. Recommend that the patient take most of the daily dose at bedtime. CNS depression is common with clonidine, but this effect lessens over time. Constipation is also a common side effect. Patients who take most of the daily amount at bedtime can minimize daytime sedation. Transdermal forms of clonidine do not alter adverse effects. Patients who are abusing clonidine often experience euphoria and hallucinations along with sedation, but they generally find these effects desirable and would not complain about them to a healthcare provider. Clonidine should not be withdrawn abruptly, because serious rebound hypertension can occur.

204. A nurse is caring for a woman with breast cancer who is receiving tamoxifen. A review of this patient's chart reveals a deficiency of the CYP2D6 gene. The nurse will contact the provider to suggest: a. a different medication. b. an increased dose. c. a reduced dose. d. serum drug levels.

a. a different medication. Women with a deficiency of the CYP2D6 gene lack the ability to convert tamoxifen to its active form, endoxifen, and will not benefit from this drug. Another drug should be used to treat this patient's breast cancer. Increasing the dose, reducing the dose, or monitoring serum drug levels will not make this drug more effective in these women.

103. A patient receives an epidural anesthetic during labor and delivery. The nurse caring for the newborn in the immediate postpartum period will observe the infant for: a. bradycardia. b. hypoglycemia. c. jitteriness. d. tachypnea.

a. bradycardia. Local anesthetics can cross the placenta, causing bradycardia and central nervous system (CNS) depression in the infant. They do not affect blood glucose. Jitteriness is a sign of CNS excitation. Increased respirations are not an adverse effect in the newborn.

235. A patient asks why albuterol causes a feeling of jitteriness when it is used to treat wheezing. The nurse knows that albuterol is a beta-adrenergic agonist that acts on beta2 receptor sites to cause smooth muscle dilation in the bronchioles of the lungs, but that it also can sometimes act on beta1 receptor sites in skeletal muscles to cause tremors. To explain this to the patient, the nurse will rely on knowledge of: a. drug selectivity. b. modified occupancy theory. c. relative potency. d. reversible effects.

a. drug selectivity. The ability of a drug to be selective for receptor sites in a patient determines the types of effects it can have on the body. This drug can bind to two different types of receptors that cause different reactions. The modified occupancy theory addresses the strength of an attraction between a drug and a receptor and the drug's ability to activate the receptor. Relative potency describes the amount of drug needed to produce a specific effect. Most drugs remain bound to receptors permanently, causing the effects to be reversible.

A patient asks why albuterol causes a feeling of jitteriness when it is used to treat wheezing. The nurse knows that albuterol is a beta-adrenergic agonist that acts on beta2 receptor sites to cause smooth muscle dilation in the bronchioles of the lungs, but that it also can sometimes act on beta1 receptor sites in skeletal muscles to cause tremors. To explain this to the patient, the nurse will rely on knowledge of:

a. drug selectivity. The ability of a drug to be selective for receptor sites in a patient determines the types of effects it can have on the body. This drug can bind to two different types of receptors that cause different reactions. The modified occupancy theory addresses the strength of an attraction between a drug and a receptor and the drug's ability to activate the receptor. Relative potency describes the amount of drug needed to produce a specific effect. Most drugs remain bound to receptors permanently, causing the effects to be reversible.

251. A nursing student asks a nurse about pharmaceutical research and wants to know the purpose of randomization in drug trials. The nurse explains that randomization is used to: a. ensure that differences in outcomes are the result of treatment and not differences in subjects. b. compare the outcome caused by the treatment to the outcome caused by no treatment. c. make sure that researchers are unaware of which subjects are in which group. d. prevent subjects from knowing which group they are in and prevent preconception bias.

a. ensure that differences in outcomes are the result of treatment and not differences in subjects. Randomization helps prevent allocation bias, which can occur when researchers place subjects with desired characteristics in the study group and other subjects in the control group so that differences in outcome are actually the result of differences in subjects and not treatment. Comparing treatment outcome to no treatment outcome is the definition of a controlled study. The last two options describe the use of blinding in studies; blinding ensures that researchers or subjects (or both) are unaware of which subjects are in which group so that preconceptions about benefits and risks cannot bias the results.

138. Because they cause vasoconstriction, alpha1-adrenergic agonists are especially useful for: a. extending the duration of local anesthetics. b. producing mydriasis to facilitate ophthalmic examinations. c. slowing the heart rate in tachycardic patients. d. treating hypotension.

a. extending the duration of local anesthetics. The vasoconstrictive effects of alpha1-adrenergic agonists, such as epinephrine, help delay absorption of a local anesthetic, prolonging the anesthetic's effects. These agents can increase blood pressure but are not the primary drugs used except in emergency situations. Alpha1 receptors in the radial muscles of the iris, not vasoconstriction, cause mydriasis. Reflex bradycardia is a side effect and not a clinical use of these agents.

170. The nurse understands that patients are given beta1 agonists to treat _____ failure. a. heart b. kidney c. respiratory d. liver

a. heart A beta1 agonist increases the patient's heart rate and blood pressure and is used in heart failure. Beta1 agonists would not be used for kidney, respiratory, or liver failure.

101. A nurse is assisting the physician during a procedure in which a local anesthetic is administered. Within a few minutes of administration of the anesthetic, the patient has a pulse of 54 beats per minute, respirations of 18 breaths per minute, and a blood pressure of 90/42 mm Hg. The nurse should monitor the patient for further signs of: a. heart block. b. anaphylaxis. c. central nervous system excitation. d. respiratory depression.

a. heart block. When absorbed in a sufficient amount, local anesthetics can affect the heart and blood vessels. These drugs suppress excitability in the myocardium and conduction system and can cause hypotension, bradycardia, heart block, and potentially cardiac arrest. Anaphylaxis would be manifested by hypotension, bronchoconstriction, and edema of the glottis. Central nervous system excitation would be manifested by hyperactivity, restlessness, and anxiety and may be followed by convulsions. No evidence indicates respiratory depression; this patient's respirations are within normal limits.

184. A patient receiving botulinum toxin injections to control muscle spasticity asks how the drug works. The nurse knows that this drug affects the transmitter acetylcholine by: a. inhibiting its release. b. interfering with its storage. c. preventing its reuptake. d. promoting its synthesis.

a. inhibiting its release Acetylcholine is a neurotransmitter that activates receptors that increase skeletal muscle contraction. Botulinum toxin inhibits the release of this transmitter. It does not interfere with storage, reuptake, or synthesis of acetylcholine.

What are the interventions for administering interferon (Actimmune) drugs?

adverse effects include: flu - like symptoms, fever, headache, malaise, myalgia, and fatigue. the major dose limiting adverse effect is fatigue Used for the treatment of viral infections, various cancers, and some autoimmune disorders. patients CBC should be documented, because long term therapy with these drugs may lead to bone marrow suppression. other levels should be checked like BUN, creatinine levels, platelet counts, and ALP and AST, and UA. baseline vital signs.

157. A nurse is helping a nursing student who is administering a medication to a patient with myasthenia gravis. Which statement by the student indicates the need for further teaching? a. "I will ask the patient to sip some water before giving the medication." b. "I will let the patient sleep after giving the medication, because rest is important." c. "I will record muscle strength assessments before and after I give the medication." d. "I will report excessive salivation to the patient's prescriber."

b. "I will let the patient sleep after giving the medication, because rest is important." An important assessment indicating the medication's effectiveness is evaluating a patient's ability to raise the eyelids. Letting the patient sleep, even though rest is important, would prevent the nurse from making this assessment. To make sure the patient can safely swallow a pill, the nurse must evaluate swallowing ability by asking the patient to take a sip of water. Muscle strength assessments before and after medication administration help the nurse evaluate the medication's effectiveness. Excessive salivation may be a sign of drug toxicity and should be reported.

252. Someone asks a nurse about a new drug that is in preclinical testing and wants to know why it cannot be used to treat a friend's illness. Which statement by the nurse is correct? a. "A drug at this stage of development can be used only in patients with serious disease." b. "At this stage of drug development, the safety and usefulness of the medication is unknown." c. "Clinical trials must be completed to make sure the drug is safe to use in humans." d. "Until postmarketing surveillance data are available, the drug cannot be used."

b. "At this stage of drug development, the safety and usefulness of the medication is unknown." Preclinical testing must be completed before drugs can be tested in humans. In this stage, drugs are evaluated for toxicities, pharmacokinetic properties, and potentially useful effects. Some drugs can be used in patients before completion of Phase III studies, but this is after preclinical testing is complete. Clinical trials proceed in stages, and each stage has guidelines defining how a new drug may be used and which patients may receive it. Postmarketing surveillance takes place after a drug is in general use.

176. A nurse is explaining activation of beta2 receptors to some student nurses during a clinical rotation at the hospital. Which statement by a student demonstrates a need for further teaching? a. "Beta2 activation results in bronchodilation." b. "Beta2 activation results in contraction of uterine muscle." c. "Beta2 activation results in glycogenolysis." d. "Beta2 activation results in vasodilation of skeletal muscles."

b. "Beta2 activation results in contraction of uterine muscle." Beta2 activation would result in relaxation of uterine smooth muscle, not contraction; this statement indicates a need for further teaching. Bronchodilation is an effect of beta2 activation; no further teaching is needed. Beta2 activation does result in glycogenolysis; no further teaching is needed. Beta2 activation does result in vasodilation of skeletal muscle; no further teaching is needed.

131. A nurse is teaching a nursing student about the two classes of adrenergic agonist drugs. Which statement by the nursing student indicates understanding of the teaching? a. "Catecholamines may be given orally." b. "Catecholamines often require continuous infusion to be effective." c. "Noncatecholamines do not cross the blood-brain barrier." d. "Noncatecholamines undergo rapid degradation

b. "Catecholamines often require continuous infusion to be effective." Catecholamines undergo rapid degradation by monoamine oxidase (MAO) and catechol-O-methyltransferase (COMT). Consequently, they have a brief duration of action, and continuous infusion often is required to maintain the drug's effects. Catecholamines cannot be used orally. Catecholamines do not cross the blood-brain barrier; noncatecholamines do. Noncatecholamines are not degraded by MAO.

223. A patient taking oral contraceptives thinks she may be pregnant. As part of this patient's history, what will the nurse ask the patient? a. "Do you drink grapefruit juice?" b. "Do you take seizure medication?" c. "Do you take your contraception with milk?" d. "Do you use laxatives regularly?"

b. "Do you take seizure medication?" Patients taking oral contraceptives along with phenobarbital, which is used to treat seizures, will have lower levels of the contraceptive, because phenobarbital is an inducing agent, which causes an increase in the metabolism of oral contraceptives. Grapefruit juice inhibits the metabolism of some drugs, leading to toxic effects. Dairy products interfere with the absorption of tetracyclines, because the calcium binds with the drug to form an insoluble complex. Laxatives reduce the absorption of some drugs by speeding up the transit time through the gut

161. A nurse is teaching a group of nursing students about irreversible cholinesterase inhibitors. Which statement by a student indicates understanding of the teaching? a. "Diazepam is given to reverse inhibition of cholinesterase when overdose occurs." b. "Irreversible cholinesterase inhibitors are rapidly absorbed by all routes." c. "These agents are often used to treat glaucoma." d. "Toxic doses of these agents produce an anticholinergic crisis."

b. "Irreversible cholinesterase inhibitors are rapidly absorbed by all routes." Almost all of these agents are highly lipid soluble, making them readily absorbed from all routes of administration. Diazepam is used during overdose but only to control seizures and not to reverse effects. The only clinical use for these agents is to treat glaucoma, but this use is limited. Toxic doses of these agents produce a cholinergic, not an anticholinergic, crisis.

156. Two nurses are discussing the major differences between physostigmine (Antilirium) and neostigmine (Prostigmin). One nurse correctly makes which statement about physostigmine (Antilirium)? a. "It is not effective for treating poisoning by muscarinic blocking drugs." b. "It can readily cross the blood-brain barrier." c. "It does not cause any side effects." d. "It can be given by all routes."

b. "It can readily cross the blood-brain barrier." The basic pharmacology of physostigmine is nearly identical to that of neostigmine, except that physostigmine readily crosses membranes, including the blood-brain barrier; neostigmine does not. Physostigmine and neostigmine are both effective for treating poisoning by muscarinic blocking drugs. Neither physostigmine nor neostigmine is devoid of side effects. Physostigmine can be given only intramuscularly (IM) or intravenously (IV); neostigmine is given orally (PO), IM, IV, and subcutaneously (subQ).

126. A patient is taking a beta-adrenergic antagonist medication for angina pectoris and asks the nurse how the drug works to relieve the discomfort associated with this condition. Which statement by the patient after the nurse's teaching indicates understanding of the drug's effects? a. "It causes bronchodilation, which increases oxygen flow." b. "It helps reduce the heart's oxygen needs." c. "It improves blood flow to the heart." d. "It increases cardiac output."

b. "It helps reduce the heart's oxygen needs." Blockade of beta1 receptors in the heart reduces cardiac work by reducing the heart rate, the force of contraction, and the velocity of impulse conduction through the AV node. Beta blockers result in bronchoconstriction, not bronchodilation. They do not increase blood flow to the heart. Cardiac output is decreased, not increased.

106. A nurse is teaching a group of nursing students about local anesthetics. Which statement by a student reflects an understanding of the teaching? a. "Local anesthetics affect large myelinated neurons first." b. "Local anesthetics affect motor and sensory nerves." c. "Local anesthetics do not block temperature perception." d. "Local anesthetics do not cause systemic effects."

b. "Local anesthetics affect motor and sensory nerves." Local anesthetics are nonselective modifiers of neuronal function. They block actions in both motor and sensory nerves. They affect small myelinated neurons first. They block temperature, pressure, and pain sensation. When absorbed into the systemic circulation, they can cause systemic effects.

155. A patient who has myasthenia gravis will be taking neostigmine (Prostigmin). What will the nurse emphasize when teaching this patient about the medication? a. "Stop taking the drug if you have diarrhea." b. "Take a supplementary dose before exercise." c. "Use atropine if you have excessive salivation." d. "Withhold the dose if ptosis occurs."

b. "Take a supplementary dose before exercise." Neostigmine doses must be adjusted continually, and patients usually need supplemental doses before exertion; therefore, patients must be taught how to modify doses as needed. Increased gastrointestinal (GI) secretions can cause loose stools; this is a known adverse effect that does not warrant stopping the drug. Atropine will help with excessive salivation but should not be used routinely, because it can mask the early signs of anticholinesterase overdose. Ptosis is one of the symptoms of myasthenia gravis and is an indication for taking neostigmine.

256. A postoperative patient is being discharged home with acetaminophen/hydrocodone (Lortab) for pain. The patient asks the nurse about using Tylenol for fever. Which statement by the nurse is correct? a. "It is not safe to take over-the-counter drugs with prescription medications." b. "Taking the two medications together poses a risk of drug toxicity." c. "There are no known drug interactions, so this will be safe." d. "Tylenol and Lortab are different drugs, so there is no risk of overdose."

b. "Taking the two medications together poses a risk of drug toxicity." Tylenol is the trade name and acetaminophen is the generic name for the same medication. It is important to teach patients to be aware of the different names for the same drug to minimize the risk of overdose. Over-the-counter (OTC) medications and prescription medications may be taken together unless significant harmful drug interactions are possible. Even though no drug interactions are at play in this case, both drugs contain acetaminophen, which could lead to toxicity.

242. A nurse is giving an enteral medication. The patient asks why this method is preferable for this drug. How will the nurse reply? a. "This route allows more rapid absorption of the drug." b. "This route is safer, less expensive, and more convenient." c. "This route is the best way to control serum drug levels." d. "This route prevents inactivation of the drug by digestive enzymes."

b. "This route is safer, less expensive, and more convenient." Parenteral routes include the intravenous, intramuscular, and subcutaneous routes. Enteral routes include oral administration, including pills and liquid suspensions. Enteral routes are safer, cheaper, and easier to use. Parenteral routes are used when rapid absorption, precise control of plasma drug levels, and prevention of digestive inactivation are important.

244. A prescriber has written an order for a medication: drug X 100 mg PO every 6 hours. The half-life for the drug is approximately 6 hours. The nurse is preparing to administer the first dose at 8:00 AM on Tuesday. On Wednesday, when will the serum drug level reach plateau? a. 2:00 AM b. 8:00 AM c. 2:00 PM d. 8:00 PM

b. 8:00 AM It takes four half-lives for a drug to reach plateau. Total body stores reach their peak at the beginning of the fifth dose of a drug if all doses are equal in amount; in this case, this will be at 8:00 AM the following day.

198. A postoperative patient who is worried about pain control will be discharged several days after surgery. The nurse providing discharge teaching tells the patient that the prescribed Lortab is not as strong as the morphine the patient was given in the immediate postoperative period. Which response is the patient likely to experience? a. A decreased likelihood of filling the prescription for the drug b. A negative placebo effect when taking the medication c. An increased compliance with the drug regimen d. Optimistic, realistic expectations about the drug

b. A negative placebo effect when taking the medication The full extent of placebo effects, if they truly occur, is not well documented or understood, although a decrease in pain as a placebo effect has been demonstrated to some extent. To foster a beneficial placebo effect, it is important for all members of the healthcare team to present an optimistic and realistic assessment of the effects of the drug the patient is taking. If the nurse tells an anxious patient that the medication being given is not as strong as what has been given, the patient is likely to have lowered expectations of the effectiveness of the drug, causing a negative placebo effect. Lowered expectations do not mean that the patient will give up on the drug entirely; in fact, the patient may actually fill the prescription and then take more drug than what is prescribed in order to get a better effect.

236. Two nurses are discussing theories of drug-receptor interaction. Which statements are true regarding the affinity of a drug and its receptor? (Select all that apply.) a. Affinity and intrinsic activity are dependent properties. b. Affinity refers to the strength of the attraction between a drug and its receptor. c. Drugs with high affinity are strongly attracted to their receptors. d. Drugs with low affinity are strongly attracted to their receptors. e. The affinity of a drug for its receptors is reflected in its potency.

b. Affinity refers to the strength of the attraction between a drug and its receptor. c. Drugs with high affinity are strongly attracted to their receptors. e. The affinity of a drug for its receptors is reflected in its potency. Affinity refers to the strength of the attraction between a drug and its receptor. Drugs with high affinity are strongly attracted to their receptors, and the affinity of a drug and its receptors is reflected in its potency. Affinity and intrinsic activity are independent properties. Drugs with low affinity are weakly attracted to their receptors.

265. A nurse is reviewing a patient's medical record before administering a medication. Which factor(s) can alter the patient's physiologic response to the drug? (Select all that apply.) a. Ability to swallow pills b. Age c. Genetic factors d. Gender e. Height

b. Age c. Genetic factors d. Gender Age, genetic factors, and gender all influence an individual patient's ability to absorb, metabolize, and excrete drugs; therefore, these factors must be assessed before a medication is administered. A patient's ability to swallow pills, although it may determine the way a drug is administered, does not affect the physiologic response. Height does not affect response; weight and the distribution of adipose tissue can affect the distribution of drugs.

226. A nurse is teaching a patient about a prescription for a monoamine oxidase (MAO) inhibitor for depression. What will the nurse teach the patient to avoid while taking this drug? a. Alcoholic beverages b. Aged cheeses c. Brussels sprouts and cabbage d. Grapefruit juice

b. Aged cheeses Aged cheeses are rich in tyramine, which interacts with MAO inhibitors to raise blood pressure to life-threatening levels. Patients taking MAO inhibitors should be taught to avoid tyramine-rich foods. Chianti wine contains tyramine, but other alcoholic beverages do not. Brussels sprouts and cabbage are foods rich in vitamin K, which can interfere with the effects of warfarin. Grapefruit juice inhibits CYP3A4 and interferes with the metabolism of many medications.

195. A nurse is obtaining a drug history from an older adult patient who is taking multiple medications prescribed by different providers. Which two medications taken together are a reason for concern? a. Acetaminophen (Tylenol) and oxycodone b. Amitriptyline (Elavil) and diphenhydramine (Benadryl) c. Fexofenadine (Allegra) and an over-the-counter laxative d. Zolpidem (Ambien) and sertraline (Zoloft)

b. Amitriptyline (Elavil) and diphenhydramine (Benadryl) Both amitriptyline and diphenhydramine are on the BEERS list, amitriptyline for anticholinergic effects and diphenhydramine because it causes blurred vision. Additionally, they both have CNS effects that can compound each other when the drugs are given together. Acetaminophen and oxycodone are both acceptable and may be given together. Fexofenadine is a second-generation antihistamine with fewer side effects, and it is not contraindicated for use with a laxative. Zolpidem is a sedative that has less risk of physical dependence and less risk of confusion, falls, and cognitive impairment; sertraline is a safer antidepressant, because it has a shorter half-life than others.

158. A patient with myasthenia gravis who is taking a cholinesterase inhibitor is being admitted to the intensive care unit and is on mechanical ventilation. The prescriber has ordered a challenge dose of edrophonium to distinguish between a myasthenic crisis and a medication overdose. The nurse will expect to do what? a. Administer neostigmine if muscle strength decreases. b. Be prepared to administer atropine if muscle weakness increases. c. Give a second dose of edrophonium if no improvement is seen. d. Give pralidoxime (Protopam) if cholinergic symptoms worsen.

b. Be prepared to administer atropine if muscle weakness increases. Edrophonium is an ultra-short-acting cholinesterase inhibitor used to distinguish between a myasthenic crisis and a cholinesterase inhibitor overdose, because weakness or paralysis can occur with both. If the symptoms are intensified and muscle weakness increases, the crisis is cholinergic, meaning that the patient is having toxic effects from the cholinesterase inhibitor and needs atropine as an antidote. Neostigmine would be given if the patient shows improved muscle strength, because that would indicate that the patient's symptoms are the result of a myasthenic crisis. A second dose of edrophonium would only worsen the symptoms. Pralidoxime is used to treat poisoning caused by irreversible cholinesterase inhibitors.

151. A patient with hepatitis requires endotracheal intubation. Which agent does the nurse expect to be administered to facilitate this procedure? a. Atropine b. Cisatracurium (Nimbex) c. Rocuronium (Zemuron) d. Vecuronium (Norcuron)

b. Cisatracurium (Nimbex) Cisatracurium is used for muscle relaxation during intubation. It is eliminated by spontaneous degradation, not by hepatic metabolism or renal excretion. Atropine is not used for muscle relaxation; it is an anticholinergic drug used to control secretions during procedures. Rocuronium is eliminated by hepatic metabolism. Vecuronium is excreted in bile and can cause prolonged paralysis in patients with liver dysfunction.

258. A postoperative patient reports pain, which the patient rates as an 8 on a scale from 1 to 10 (10 being the most extreme pain). The prescriber has ordered acetaminophen (Tylenol) 650 mg PO every 6 hours PRN pain. What will the nurse do? a. Ask the patient what medications have helped with pain in the past. b. Contact the provider to request a different analgesic medication. c. Give the pain medication and reposition the patient to promote comfort. d. Request an order to administer the medication every 4 hours.

b. Contact the provider to request a different analgesic medication. The nursing diagnosis for this patient is severe pain. Acetaminophen is given for mild to moderate pain, so the nurse should ask the prescriber to order a stronger analgesic medication. Asking the patient to tell the nurse what has helped in the past is a part of an initial assessment and should be done preoperatively and not when the patient is having severe pain. Because the patient is having severe pain, acetaminophen combined with nondrug therapies will not be sufficient. Increasing the frequency of the dose of a medication for mild pain will not be effective.

196. A nurse is making a home visit to an older adult woman who was recently discharged home from the hospital with a new prescription. The nurse notes that a serum drug level drawn the day before was subtherapeutic. What will the nurse do next? a. Ask the patient if she has difficulty swallowing pills. b. Count the pills in the prescription bottle. c. Notify the provider to request more frequent dosing. d. Request an order for renal function tests.

b. Count the pills in the prescription bottle. Intentional nonadherence is common and may occur because older adult patients are not convinced that drugs are needed or that the dose prescribed is correct. Counting the pills would be an appropriate first step as the nurse determines the cause of the low serum drug level, because it provides information about adherence. If the pill count is correct and the patient has taken the drug as prescribed, other causes may have to be investigated. If it is clear that the patient has not been taking enough of the medication, asking about her ability to swallow may be a good follow-up question. The last two options would be steps to discuss with the provider if the patient is taking the medication as prescribed.

222. A patient is taking drug X and receives a new prescription for drug Y, which is listed as an inducing agent. The nurse caring for this patient understands that this patient may require _____ doses of drug _____. a. lower; X b. lower; Y c. higher; X d. higher; Y

c. higher; X An inducing agent stimulates the synthesis of CYP isozymes, which may increase the metabolism of other drugs as much as two- to threefold, thereby lowering the level of those drugs in the body and requiring higher doses to maintain drug effectiveness.

152. A nurse is teaching a group of nursing students about neuromuscular blocking agents. For what may these agents be used? (Select all that apply.) a. Analgesia b. Electroshock therapy c. Malignant hyperthermia d. Mechanical ventilation e. Surgery

b. Electroshock therapy d. Mechanical ventilation e. Surgery Neuromuscular blocking agents are used to produce muscle paralysis in procedures including electroconvulsive therapy (ECT), intubation and ventilation, and surgery. They do not produce analgesia. They are not used to treat malignant hyperthermia.

250. The FDA Amendments Act (FDAAA) was passed in 2007 to address which aspect of drug safety? a. Allowing pharmaceutical companies to identify off-label uses of medications approved for other uses b. Evaluating drug safety information that emerges after a drug has been approved and is in use c. Expediting the approval process of the U.S. Food and Drug Administration (FDA) so that needed drugs can get to market more quickly d. Requiring manufacturers to notify patients before removing a drug from the market

b. Evaluating drug safety information that emerges after a drug has been approved and is in use The FDAAA was passed to enable the Food and Drug Administration to continue oversight of a drug after granting it approval so that changes in labeling could be made as necessary and postmarketing risks could be tracked and identified. A provision of the FDA Modernization Act (FDAMA), passed in 1997, allows drug companies to promote their products for off-label uses as long as they promise to conduct studies to support their claims. Regulations to permit accelerated approval of drugs for life-threatening diseases were adopted in 1992 by the FDA. The requirement that drug companies notify patients 6 months before removing a drug from the market is a provision of the FDAMA.

168. Bethanechol (Urecholine) is used to treat urinary retention but is being investigated for use in which other condition? a. Gastric ulcers b. Gastroesophageal reflux c. Hypotension d. Intestinal obstruction

b. Gastroesophageal reflux Bethanechol is being investigated for treatment for gastroesophageal reflux disease (GERD) because of its effects on esophageal motility and the lower esophageal sphincter. Bethanechol stimulates acid secretion and could intensify ulcer formation. Bethanechol can cause hypotension. Because bethanechol increases the motility and tone of intestinal smooth muscle, the presence of an obstruction could lead to bowel rupture.

217. Which actions occur in 90% of fatal medication errors? (Select all that apply.) a. Confusing drugs with similar packaging b. Giving a drug intravenously instead of intramuscularly c. Giving Nasarel instead of Nizoral d. Using an infusion device that malfunctions e. Writing a prescription illegibly

b. Giving a drug intravenously instead of intramuscularly c. Giving Nasarel instead of Nizoral e. Writing a prescription illegibly Ninety percent of fatal medication errors fall into three categories: human factors, communication mistakes, and name confusion. Giving a drug IV (intravenously) instead of IM (intramuscularly) is an example of a human factor; writing a prescription so that it is illegible is an example of a communication mistake; and giving a drug with a name that sounds like the name of another drug is an example of name confusion. Confusion of drugs with similar packaging and using a faulty device also can cause fatal drug errors, but these factors do not fall into the categories that account for 90% of fatal errors.

248. When administering medications to infants, it is important to remember which of the following? (Select all that apply.) a. Breast-feeding infants are more likely to develop toxicity when given lipid-soluble drugs. b. Immaturity of renal function in infancy causes infants to excrete drugs less efficiently. c. Infants have immature livers, which slows drug metabolism. d. Infants are more sensitive to medications that act on the central nervous system (CNS). e. Oral medications are contraindicated in infants, because PO administration requires a cooperative patient.

b. Immaturity of renal function in infancy causes infants to excrete drugs less efficiently. c. Infants have immature livers, which slows drug metabolism. d. Infants are more sensitive to medications that act on the central nervous system (CNS). Immature renal function causes infants to excrete drugs more slowly, and infants are at risk for toxicity until renal function is well developed. Infants' livers are not completely developed, and they are less able to metabolize drugs efficiently. Because the blood-brain barrier is not well developed in infants, caution must be used when administering CNS drugs. Lipid-soluble drugs may be excreted in breast milk if the mother is taking them, but breast-feeding does not affect medications given directly to the infant. Oral medications may be given safely to infants as long as they are awake and can swallow the drug.

132. Dopamine is administered to a patient who has been experiencing hypotensive episodes. Other than an increase in blood pressure, which indicator would the nurse use to evaluate a successful response? a. Decrease in pulse b. Increase in urine output c. Weight gain d. Improved gastric motility

b. Increase in urine output Dopamine would cause an increase in urine output, because cardiac output is increased as a result of the increase in blood pressure. The effectiveness of dopamine would not be measured by a decrease in pulse, because dopamine's primary effect is to increase blood pressure. Dopamine's effectiveness would not be evaluated by a weight gain. Dopamine's effectiveness would not be evaluated by improved gastric motility.

212. A nurse is preparing to administer a drug. Upon reading the medication guide, the nurse notes that the drug has been linked to symptoms of Parkinson's disease in some patients. What will the nurse do? a. Ask the patient to report these symptoms, which are known to be teratogenic effects. b. Observe the patient closely for such symptoms and prepare to treat them if needed. c. Request an order to evaluate the patient's genetic predisposition to this effect. d. Warn the patient about these effects and provide reassurance that this is expected.

b. Observe the patient closely for such symptoms and prepare to treat them if needed. A drug that causes diseaselike symptoms is known to be iatrogenic. Nurses should be prepared for this possibility and should be prepared to withdraw the drug if necessary and treat the symptoms. Such effects are not teratogenic. Patients with a genetic predisposition to respond differently to drugs are known to have idiosyncratic effects. Iatrogenic effects, even when known, are not typically expected side effects.

163. A patient has developed muscarinic antagonist toxicity from ingestion of an unknown chemical. The nurse should prepare to administer which medication? a. Atropine (Sal-Tropine) IV b. Physostigmine (Antilirium) c. An acetylcholinesterase activator d. Pseudoephedrine (Ephedrine)

b. Physostigmine (Antilirium) Physostigmine is indicated for muscarinic antagonist toxicity. Atropine is a drying agent and would only complicate the drying action that arises from the muscarinic antagonist. An acetylcholinesterase activator would only contribute to dryness that arises from the muscarinic antagonist. Ephedrine is not indicated for muscarinic antagonist toxicity.

269. What are the properties of an ideal drug? (Select all that apply.) a. Irreversible action b. Predictability c. Ease of administration d. Chemical stability e. A simple trade name

b. Predictability c. Ease of administration d. Chemical stability In addition to predictability, ease of administration, and chemical stability, other properties include a reversible action so that any harm the drug may cause can be undone and a simple generic name, because generic names are usually complex and difficult to remember and pronounce.

130. Which are adverse effects of alpha blockade? (Select all that apply.) a. Hypertension b. Reflex tachycardia c. Nasal congestion d. Ejaculation e. Hypernatremia

b. Reflex tachycardia c. Nasal congestion e. Hypernatremia Adverse effects of alpha blockade include reflex tachycardia, nasal congestion, and hypernatremia. Other adverse effects include orthostatic hypotension and inhibition of ejaculation. Hypertension and ejaculation are not adverse effects of alpha blockade.

172. A patient is wheezing and short of breath. The nurse assesses a heart rate of 88 beats per minute, a respiratory rate of 24 breaths per minute, and a blood pressure of 124/78 mm Hg. The prescriber orders a nonspecific beta agonist medication. Besides evaluating the patient for a reduction in respiratory distress, the nurse will monitor for which side effect? a. Hypotension b. Tachycardia c. Tachypnea d. Urinary retention

b. Tachycardia Beta agonists are used for asthma because of their beta2 effects on bronchial smooth muscle, causing dilation. Beta1 effects cause tachycardia and hypertension. Beta receptors do not exert effects on the bladder.

190. Based on changes in hepatic function in older adult patients, which adjustment should the nurse expect for oral medications that undergo extensive first pass metabolism? a. A higher dose should be used with the same time schedule. b. The interval between doses should be increased. c. No change is necessary; metabolism will not be affected. d. The interval between doses should be reduced.

b. The interval between doses should be increased. The interval between doses of the medication should be increased in older adult patients, because drugs that undergo the first pass effect may not be broken down as well as in an individual with full liver function. A higher dose of the medication is not indicated, because toxic effects could occur. A change in administration may be indicated in older adults, because their metabolism is affected. The interval between doses should not be reduced but increased.

254. A nurse is teaching nursing students about the use of nonproprietary names for drugs. The nurse tells them which fact about nonproprietary names? a. They are approved by the FDA and are easy to remember. b. They are assigned by the U.S. Adopted Names Council. c. They clearly identify the drug's pharmacological classification. d. They imply the efficacy of the drug and are less complex.

b. They are assigned by the U.S. Adopted Names Council. Nonproprietary, or generic, names are assigned by the U.S. Adopted Names Council, which ensures that each drug has only one name. Trade names, or brand names, are approved by the FDA and are easier to remember. Some nonproprietary names contain syllables that identify the classification, although not all do. Drug names are not supposed to identify the use for the drug, although some brand names do so.

107. Vasoconstrictors are combined with local anesthetics for which reasons? (Select all that apply.) a. To enhance absorption b. To reduce the risk of toxicity c. To prevent bradycardia d. To shorten the duration of action e. To prolong anesthesia

b. To reduce the risk of toxicity e. To prolong anesthesia Vasoconstrictors, when combined with local anesthetics, reduce the risk of toxicity and prolong the anesthetic effects. Vasoconstrictors, when combined with local anesthetics, do not speed up the absorption process, prevent bradycardia, or shorten the duration of action.

167. A patient received atropine intravenously before surgery. The recovery room nurse notes that the patient is delirious upon awakening, has a heart rate of 96 beats per minute, a respiratory rate of 22 breaths per minute, and a blood pressure of 110/78 mm Hg. The nurse notifies the anesthesiologist, who will order: a. activated charcoal to minimize intestinal absorption of the antimuscarinic agent. b. an acetylcholinesterase inhibitor to compete with the antimuscarinic agent at receptors. c. an antipsychotic medication to treat the patient's central nervous system symptoms. d. ipratropium bromide (Atrovent) to counter the respiratory effects of the antimuscarinic agent.

b. an acetylcholinesterase inhibitor to compete with the antimuscarinic agent at receptors. This patient is showing signs of antimuscarinic toxicity, caused by the atropine given during surgery. The most effective antidote is physostigmine, which inhibits acetylcholinesterase, allowing acetylcholine to build up at cholinergic junctions and compete with the antimuscarinic agent for receptor binding. Activated charcoal is only useful if an antimuscarinic agent has been ingested, because it impedes absorption from the GI tract. Because this patient's psychotic symptoms are caused by an antimuscarinic agent, physostigmine should be given to treat the cause; an antipsychotic medication would only treat the symptom. Ipratropium bromide is an antimuscarinic agent and would only compound the effects. This patient's respiratory rate is only mildly elevated

200. The U.S. Food and Drug Administration (FDA) recommends genetic testing of patients receiving certain medications. Genetic testing helps prescribers: a. better establish a drug's therapeutic index. b. determine whether a patient is a rapid or slow metabolizer of the drug. c. identify racial characteristics that affect psychosocial variation in drug response. d. produce a drug that is tailored to an individual patient's genetic makeup.

b. determine whether a patient is a rapid or slow metabolizer of the drug. Pharmacogenomics is the study of the ways genetic variations affect individual responses to drugs through alterations in genes that code for drug-metabolizing enzymes and drug receptors. For some drugs, the FDA requires genetic testing and for others, this testing is recommended but not required. Genetic testing does not determine a drug's therapeutic index; this is a measure of a drug's safety based on statistics of the drug's use in the general population (see Chapter 5). Any distinct physiological differences in drug response among various racial populations are related to genetic differences and do not affect psychosocial differences in drug responses. Genetic testing is recommended to identify how a patient will respond to a drug and not to design a drug specific to an individual.

124. A patient taking a beta blocker complains of shortness of breath. The patient has respirations of 28 breaths per minute, a blood pressure of 162/90 mm Hg, and a pulse of 88 beats per minute. The nurse auscultates crackles in all lung fields. The nurse understands that these assessments are consistent with: a. bronchoconstriction. b. left-sided heart failure. c. rebound cardiac excitation. d. sinus bradycardia.

b. left-sided heart failure The signs and symptoms describe left-sided heart failure, in which the blood normally handled by the left ventricle and forced out through the aorta into the body backs up into the lungs, producing respiratory signs and symptoms. The patient's signs and symptoms are not indicative of bronchoconstriction, which would cause wheezing and diminished breath sounds. Rebound cardiac excitation occurs when the beta blocker is withdrawn, not during administration of the drug. The patient's heart rate is elevated, so sinus bradycardia is not present.

205. A nurse is teaching a group of women about medications. The women want to know why so many drugs have unpredictable effects in women. The nurse will tell them that: a. drugs usually have more toxic effects in women. b. most known drug effects are based on drug trials in men. c. women have varying responses to drugs during menstrual cycles. d. women metabolize drugs more slowly.

b. most known drug effects are based on drug trials in men. Until 1997 almost all clinical drug trials were performed in men. Women may have more toxic effects with some drugs and fewer toxic effects with others. Not all drugs are influenced by hormonal changes. Women metabolize some drugs more slowly and other drugs more quickly. Unless drug trials are performed in both women and men, the effects of drugs in women will not be clear.

134. A patient brought to the emergency department requires sutures. The prescriber orders a local anesthetic with epinephrine. The nurse understands that epinephrine is ordered to: a. prevent hypertension induced by the anesthetic. b. prolong absorption of the anesthetic. c. reduce anesthetic-induced nausea. d. reduce the pain of an injection.

b. prolong absorption of the anesthetic. Epinephrine prolongs absorption of the anesthetic, because it is an alpha1 agonist. It is frequently combined with a local anesthetic for this purpose. Local anesthetics do not induce hypertension; therefore, epinephrine would not be needed to prevent it. Epinephrine does not act as an antiemetic and would not reduce anesthetic-induced nausea. Epinephrine is not used to reduce the pain of an injection.

A PATIENT IS GOING HOME WITH A NEW PRESCRIPTION FOR THE BETA-BLOCKER ATENOLOL. the NURSE SHOULD INCLUDE WHAT CONTENT WHEN TECHING THE PATIENT ABOUT THIS DRUG?

be WATCHFUL FOR THE FIRST-DOSE HYPOTENSION

117. A nurse is teaching nursing students about the use of alpha-adrenergic antagonists. Which statement by the student indicates the need for further teaching? a. "Alpha-adrenergic antagonists block alpha1 receptors on arterioles and veins." b. "Dilation of arterioles has a direct effect on arterial pressure." c. "Dilation of veins by alpha-adrenergic antagonists improves cardiac output." d. "Venous dilation by alpha-adrenergic antagonists indirectly lowers arterial pressure."

c. "Dilation of veins by alpha-adrenergic antagonists improves cardiac output." Cardiac output is decreased as a result of the venous dilation caused by alpha-adrenergic antagonists. Alpha-adrenergic antagonists block alpha1 receptors on both arterioles and veins. When alpha1 receptors on arterioles are blocked by alpha-adrenergic antagonists, a direct effect on arterial pressure occurs. When alpha1 receptors on veins are blocked by alpha-adrenergic antagonists, an indirect effect on arterial pressure occurs.

120. The nurse is discussing home management with a patient who will begin taking an alpha-adrenergic antagonist for hypertension. Which statement by the patient indicates understanding of the teaching? a. "I need to stop the medication if my heart rate increases." b. "I should not drive while taking this medication." c. "I should take the first dose at bedtime." d. "I will stop taking the medication if I feel dizzy."

c. "I should take the first dose at bedtime." Orthostatic hypotension is a common side effect of this class of drugs and is most severe with the first dose. Administering the first dose at bedtime eliminates the risk associated with this first-dose effect. Tachycardia is an expected side effect; if severe, it can be treated with other medications. Patients should not drive during the first 12 to 24 hours after taking these agents, because fainting and dizziness may occur, but they may drive after that. Dizziness is not an indication for stopping the drug; patients who experience dizziness are instructed to sit or lie down until symptoms pass.

159. A nurse is teaching a new emergency department nurse about emergency medications. When asked to describe the uses of pralidoxime (Protopam), the new nurse makes which correct statement? a. "It exerts its greatest effects at muscarinic and ganglionic sites." b. "It is used to treat poisoning by reversible cholinesterase inhibitors." c. "It may not be effective if not given immediately." d. "It reverses the effects of organophosphate insecticides in the central nervous system."

c. "It may not be effective if not given immediately." Pralidoxime must be given soon after organophosphate poisoning has occurred because of a process called aging, in which the bond between the organophosphate inhibitor and cholinesterase increases in strength, making pralidoxime ineffective. For some of these poisons, aging can occur within minutes. Pralidoxime has its greatest effects at the neuromuscular junction, not the muscarinic and ganglionic sites. It is used to treat poisoning by irreversible cholinesterase inhibitors. It does not cross the blood-brain barrier and thus does not have effects in the CNS.

111. A nurse is teaching nursing students about the pharmacology of methyldopa. Which statement by a student indicates the need for further teaching? a. "Methyldopa results in alpha2 agonist activation, but it is not itself an alpha2 agonist." b. "Methyldopa is not effective until it is converted to an active compound." c. "Methyldopa reduces blood pressure by reducing cardiac output." d. "Methyldopa's principal mechanism is vasodilation, not cardiosuppression."

c. "Methyldopa reduces blood pressure by reducing cardiac output." Methyldopa does not reduce the heart rate or cardiac output, so its hypotensive actions are not the result of cardiac depression. The drug is not, in itself, an alpha2 agonist. When taken up into brainstem neurons, it is converted into methylnorephinephrine, which is an alpha2 agonist; it is not effective until converted to this active compound. Its hypotensive effects are the result of vasodilation, not cardiosuppression.

119. A patient with type I diabetes is taking NPH insulin, 30 units every day. A nurse notes that the patient is also taking metoprolol (Lopressor). What education should the nurse provide to the patient? a. "Metoprolol has no effect on diabetes mellitus or on your insulin requirements." b. "Metoprolol interferes with the effects of insulin, so you may need to increase your insulin dose." c. "Metoprolol may mask signs of hypoglycemia, so you need to monitor your blood glucose closely." d. "Metoprolol may potentiate the effects of the insulin, so the dose should be reduced."

c. "Metoprolol may mask signs of hypoglycemia, so you need to monitor your blood glucose closely." Because metoprolol may mask the signs of hypoglycemia, the patient should monitor the blood glucose closely and report changes to the prescriber. Metoprolol does have an indirect effect on diabetes mellitus and/or insulin requirements in that it may mask the signs of hypoglycemia, causing the patient to make a healthcare decision based on the drug-to-drug interaction rather than actual physiologic factors. The patient should not increase the insulin, because metoprolol will cause a decrease in blood glucose, increasing the risk of a hypoglycemic reaction. The patient should not reduce the dose of insulin when taking metoprolol, because this might alter serum glucose levels.

123. A nurse is discussing phentolamine (OraVerse) with a nursing student. Which statement by the student indicates the need for further teaching? a. "Phentolamine can be used to block both epinephrine- and norepinephrine-mediated vasoconstriction." b. "Phentolamine can be used to treat pheochromocytoma." c. "Phentolamine is a competitive adrenergic agonist that acts selectively on alpha1 receptors." d. "Side effects of phentolamine may include tachycardia and hypotension."

c. "Phentolamine is a competitive adrenergic agonist that acts selectively on alpha1 receptors." Phentolamine has actions on both alpha1 and alpha2 receptors; it is not selective for alpha1 receptors only. It blocks both epinephrine- and norepinephrine-mediated vasoconstriction. It is used to treat pheochromocytoma. Side effects include tachycardia and hypotension.

270. Before administering a medication, what does the nurse need to know to evaluate how individual patient variability might affect the patient's response to the medication? (Select all that apply.) a. Chemical stability of the medication b. Ease of administration c. Family medical history d. Patient's age e. Patient's diagnosis

c. Family medical history d. Patient's age e. Patient's diagnosis The family medical history can indicate genetic factors that may affect a patient's response to a medication. Patients of different ages can respond differently to medications. The patient's illness can affect how drugs are metabolized. The chemical stability of the medication and the ease of administration are properties of drugs.

245. An adult male patient is 1 day postoperative from a total hip replacement. On a pain scale of 0 to 10, with 10 being the greatest pain, the patient reports a pain level of 10. Which medication would be most appropriate for the nurse to administer to this patient? a. 60 mg morphine sulfate PO b. 75 mg meperidine (Demerol) intramuscularly c. 6 mg morphine sulfate intravenously d. Fentanyl (Duragesic) patch 50 mcg transdermally

c. 6 mg morphine sulfate intravenously The intravenous route is the fastest route of absorption and the one most appropriate for a patient in extreme pain. With the oral route, the medication would take at least 45 minutes to be effective, too long for a patient in extreme pain. With the intramuscular route, the medication would take at least 15 minutes to be effective; although faster than the oral route, this is not as fast as the intravenous route. A Duragesic patch would be the most inappropriate route because of the long drug half-life. This is a more appropriate route for long-term use.

185. A patient has allergies and takes an antihistamine. The patient wants to know how the drug works. The nurse understands that antihistamines work because they are what? a. Activators b. Agonists c. Antagonists d. Antidotes

c. Antagonists Antihistamines bind to receptors to prevent activation by histamine; this makes antihistamines antagonist drugs. Antihistamines do not activate receptors. Agonist drugs activate receptors; they are not antidotes.

233. A patient has been receiving an antibiotic with a small therapeutic index for 10 days. Upon assessment, the nurse suspects that the patient may be experiencing toxicity. What would be the nurse's priority action? a. Call the prescriber and have the antibiotic changed. b. Suspect an allergic reaction and administer a PRN antihistamine. c. Ask the prescriber to order a plasma drug level test. d. Set up oxygen and obtain an order for an antagonist.

c. Ask the prescriber to order a plasma drug level test. A drug with a narrow therapeutic index indicates that a drug is relatively unsafe and should be monitored closely. The nurse should have a blood level drawn to confirm suspicions of toxicity. The nurse would not have the antibiotic changed, because there is no cause at this time. The patient is unlikely to be experiencing an allergic reaction, because the antibiotic has been in the system for 10 days. The patient shows no signs of anaphylaxis, so oxygen and an antagonist are not indicated.

177. A pregnant patient is in premature labor. Which class of drug will she be given? a. Alpha1 agonist b. Anticholinergic c. Beta2 agonist d. Beta2 antagonist

c. Beta2 agonist Beta2 agonists cause relaxation of uterine muscle, slowing or stopping the contractions that precipitate labor. An alpha1 agonist would have effects on the heart and arterioles. Anticholinergic drugs generally are given for their effects on the urinary and GI tracts and do not affect uterine muscle. A beta2 antagonist would cause increased constriction of uterine muscle.

188. A nurse is preparing to teach a forgetful older adult patient about a multiple drug regimen to follow after discharge from the hospital. To help promote adherence, what will the nurse do? a. Ask the patient to share the teaching with a neighbor or friend soon after discharge. b. Give the patient detailed written information about each drug. c. Cluster medication administration times as much as possible. d. Make sure the patient understands the actions and side effects of each drug.

c. Cluster medication administration times as much as possible. Unintentional nonadherence often is the result of confusion and forgetfulness. Grouping medications to reduce the number of medication times per day can simplify the regimen and help the patient remember medication times. Enlisting a neighbor, relative, or friend is a good idea, but this person should be included in the teaching. Asking the patient to share what is learned may not be a reasonable expectation of a forgetful patient. Detailed written information may just be more confusing; verbal and written information should be clear and concise. Making sure the patient understands the actions and side effects of medications helps when intentional nonadherence is an issue, but in this case it may just add to the patient's confusion.

135. A nurse is administering intravenous dopamine (Intropin) to a patient in the intensive care unit. Which assessment finding would cause the most concern? a. Blood pressure of 100/70 mm Hg b. Increased urine output c. Edema at the IV insertion site d. Headache

c. Edema at the IV insertion site The nurse would be concerned if the patient's peripheral IV were edematous, because this could signal infiltration of the solution into the tissues. Dopamine can cause necrosis if it extravasates. Dopamine is indicated to increase the patient's blood pressure; this blood pressure reading is acceptable. When dopamine is effective at increasing cardiac output, it also causes an increase in urine output. Headache is not a contraindication to the use of dopamine.

140. A pregnant patient with diabetes and hypertension is given isoproterenol (Isuprel) to delay preterm labor. Which side effect might the nurse expect? a. Bronchospasm b. Decreased urine output c. Hyperglycemia d. Hypotension

c. Hyperglycemia In diabetic patients, isoproterenol can cause hyperglycemia by promoting beta2-mediated glycogenolysis. It is also used to treat asthma and causes bronchodilation, not bronchospasm. It acts on beta1 receptors in the heart to increase cardiac output, which may increase urine output. It increases blood pressure.

174. A nurse is administering an agonist drug that acts on postganglionic neurons of the sympathetic nervous system. Which response will the nurse expect to see? a. Decreased sweating b. Bronchodilation c. Increased cardiac output d. Pinpoint pupils

c. Increased cardiac output Norepinephrine (NE) is the most common neurotransmitter released by all postganglionic neurons of the sympathetic nervous system except those going to sweat glands where ACh is the neurotransmitter. NE acts on alpha1, alpha2, and beta1 receptors to increase the force and rate of cardiac contraction, thus increasing cardiac output. ACh would increase sweating. Bronchodilation occurs when epinephrine activates beta2 receptors on bronchial smooth muscle. NE affects alpha1 receptors to dilate the pupils.

267. What does it mean when a drug is described as easy to administer? a. It can be stored indefinitely without need for refrigeration. b. It does not interact significantly with other medications. c. It enhances patient adherence to the drug regimen. d. It is usually relatively inexpensive to produce.

c. It enhances patient adherence to the drug regimen. A major benefit of drugs that are easy to administer is that patients taking them are more likely to comply with the drug regimen. Drugs that are easy to give may have the other attributes listed, but those properties are independent of ease of administration.

181. A nurse is teaching a group of nurses about neuropharmacology. The nurse asks which classes of drugs act by reducing axonal conduction. Which response by a participant is correct? a. Antihypertensives b. Antipsychotics c. Local anesthetics d. Antidepressants

c. Local anesthetics Local anesthetics are the only drugs shown to work by reducing axonal conduction. Antihypertensives, antipsychotics, and antidepressants do not act by reducing axonal conduction.

189. A nurse is reviewing an older adult patient's chart before giving medications. Which patient information is of most concern? a. Chronic constipation b. Increased body fat c. Low serum albumin d. Low serum creatinine

c. Low serum albumin Low serum albumin reduces protein binding of drugs and can cause levels of free drug to rise, increasing the risk of toxicity. Altered gastrointestinal (GI) absorption is not a major factor in drug sensitivity in the older adult, although delayed gastric emptying can delay drug responses. Increased body fat can alter drug distribution, causing reduced responses in lipid-soluble drugs. Low serum creatinine is a function of decreased lean muscle mass and does not reflect kidney function or drug excretion.

102. A nurse is caring for a patient in the immediate postoperative period after surgery in which a spinal anesthetic was used. The patient has not voided and complains of headache. The patient has a pulse of 62 beats per minute, a respiratory rate of 16 breaths per minute, and a blood pressure of 92/48 mm Hg. Which action by the nurse is appropriate? a. Contact the anesthetist to request an order for ephedrine. b. Have the patient sit up to relieve the headache pain. c. Lower the head of the bed to a 10- to 15-degree head-down position. d. Obtain an order for a urinary catheter for urinary retention.

c. Lower the head of the bed to a 10- to 15-degree head-down position. Spinal anesthetics have several adverse effects, but the most significant is hypotension caused by the venous dilation that occurs from blockade of sympathetic nerves. The result is decreased blood return to the heart, which causes reduced cardiac output and a drop in blood pressure. The first step in treating this is to put the patient in a 10- to 15-degree head-down position to promote venous return to the heart. Ephedrine or phenylephrine is used if the first measure fails. Spinal headaches are common; the intervention for this is to have the patient assume a supine position. Urinary retention can occur secondary to autonomic blockade; it is a concern if the patient has not voided for 8 hours after the procedure, but not in the immediate postoperative period.

240. A patient receives a drug that has a narrow therapeutic range. The nurse administering this medication will expect to do what? a. Administer the drug at intervals longer than the drug half-life. b. Administer this medication intravenously. c. Monitor plasma drug levels. d. Teach the patient that maximum drug effects will occur within a short period.

c. Monitor plasma drug levels. A drug with a narrow therapeutic range is more difficult to administer safely, because the difference between the minimum effective concentration and the toxic concentration is small. Patients taking these medications must have their plasma drug levels monitored closely to ensure that they are getting an effective dose that is not toxic. Administering medications at longer intervals only increases the time required to reach effective plasma drug levels. Drugs that have a narrow therapeutic range may be given by any route and do not differ from other medications in the amount of time it takes to take effect, which is a function of a drug's half-life and dosing frequency.

215. A patient is taking a drug that has known toxic side effects. What will the nurse do? a. Discontinue the drug at the first signs of toxicity. b. Ensure that complete blood counts are ordered periodically. c. Monitor the function of all organs potentially affected by the drug. d. Teach the patient how to treat the symptoms if they develop.

c. Monitor the function of all organs potentially affected by the drug. When a drug is administered that has known toxic side effects, the nurse is responsible for monitoring all organ systems potentially affected by the drug. Not all toxic side effects warrant discontinuation of the drug, and a nurse cannot discontinue a drug without an order from the provider. Complete blood counts are indicated only for drugs that affect the blood. Some drugs need to be discontinued, so teaching a patient to treat symptoms is not correct in all cases.

146. A patient will receive atracurium (Tracrium) during surgery for neuromuscular blockade. The nurse caring for this patient will correctly perform which of the following actions? a. Assess the patient for signs of CNS depression. b. Ensure that dantrolene is available in case of a serious adverse reaction. c. Monitor the patient's blood pressure closely. d. Review the preprocedure laboratory values related to renal and hepatic function.

c. Monitor the patient's blood pressure closely. Atracurium is used for muscle relaxation during surgery. Unlike other drugs in this class, it can cause hypotension secondary to antihistamine release. Cholinesterase inhibitors can reverse this reaction. Drugs in this class do not cause CNS depression. Dantrolene is the agent used to treat malignant hyperthermia, which is triggered by succinylcholine. Atracurium is eliminated primarily by plasma cholinesterase and not by the renal or hepatic routes.

220. A young adult postoperative patient is receiving morphine 2 to 4 mg IV every 2 hours PRN pain. The last dose was 3 mg given 2 hours ago. The patient is asleep, and the nurse notes a heart rate of 86 beats per minute and a respiratory rate of 8 breaths per minute. Which PRN medication will the nurse give this patient? a. Diphenhydramine (Benadryl) to counter morphine side effects b. Morphine 4 mg for increased pain, as indicated by tachycardia c. Naloxone (Narcan) to block the effects of the morphine d. Nothing at this time, because the patient is resting comfortably

c. Naloxone (Narcan) to block the effects of the morphine

104. A nurse is preparing a patient to go home from the emergency department after receiving sutures for a laceration on one hand. The provider used lidocaine with epinephrine as a local anesthetic. Which symptom in this patient causes the most concern? a. Difficulty moving the fingers of the affected hand b. Inability to feel pressure at the suture site c. Nervousness and tachycardia d. Sensation of pain returning to the wound

c. Nervousness and tachycardia Absorption of the vasoconstrictor can cause systemic effects, including nervousness and tachycardia. If severe, alpha- and beta-adrenergic antagonists can be given. Local anesthetics are nonselective modifiers of neuronal function and also can block motor neurons, so it is expected that patients may have difficulty with movement. The sensation of pressure also is affected and is an expected effect. As the local anesthetic wears off, the sensation of pain will return.

166. An older adult patient who lives alone and is somewhat forgetful has an overactive bladder and reports occasional constipation. Which treatment will the nurse anticipate for this patient? a. Behavioral therapy b. Oxybutynin (Ditropan XL) extended-release tablets c. Oxybutynin (Oxytrol) transdermal patch d. Percutaneous tibial nerve stimulation (PTNS)

c. Oxybutynin (Oxytrol) transdermal patch The transdermal patch is applied weekly and may be the best option for a patient who is more likely to forget to take a daily medication. The transdermal preparation has fewer side effects than the systemic dose so is less likely to increase this patient's constipation. Behavioral therapy should be tried first in patients who can participate. However, it is not the best option for a forgetful patient who lives alone, because it requires scheduled voiding and timing of fluid intake. The extended-release tablets must be given daily, and this patient may not remember to take them. PTNS is used after behavioral and drug therapies have failed.

199. A nurse administers the same medication in the same preparation in the same dose to several patients and notes that some patients have a better response to the drug than others. What is the most likely explanation for this phenomenon? a. Altered bioavailability of the drug b. Patient compliance with the therapeutic regimen c. Pharmacogenomic differences among individuals d. Placebo effects enhancing expectations of drug efficacy

c. Pharmacogenomic differences among individuals Each patient's genetic makeup can determine how that patient responds to drugs both quantitatively and qualitatively, and this is the most likely cause of individual variation when the same drug is given at the same dose. The bioavailability of a drug is determined by the drug's composition and varies across formulations of the drug. The patients in this example were given the same drug. The nurse was administering the medication to the patients, so compliance is not an issue. Nothing in this example indicates that a placebo effect was in play

154. A nurse working in the emergency department is assigned to a child who is arriving by ambulance after being involved in a spill of organophosphate insecticides. What will the nurse expect to be the initial priority for treating this child? a. Administering diphenhydramine to control secretions b. Giving diazepam to control seizures c. Providing mechanical ventilation and oxygen d. Reporting the spill to the Environmental Protection Agency

c. Providing mechanical ventilation and oxygen All of these courses of action may be part of the treatment. However, the initial concern is to prevent death from apnea caused by laryngospasm, bronchoconstriction, and paralysis leading to apnea; therefore, maintaining an airway is the most important consideration. Pralidoxime is the specific antidote and should be given next. Atropine is used to control secretions. Diazepam is given when seizures occur. Reporting the accident to the proper authorities is not a priority during a life-threatening emergency.

257. The nurse is preparing to care for a patient who will be taking an antihypertensive medication. Which action by the nurse is part of the assessment step of the nursing process? a. Asking the prescriber for an order to monitor serum drug levels b. Monitoring the patient for drug interactions after giving the medication c. Questioning the patient about over-the-counter medications d. Taking the patient's blood pressure throughout the course of treatment

c. Questioning the patient about over-the-counter medications The assessment part of the nursing process involves gathering information before beginning treatment, and this includes asking about other medications the patient may be taking. Monitoring serum drug levels, watching for drug interactions, and checking vital signs after giving the medication are all part of the evaluation phase.

What is a nurse expectations for a cancer patient that has anorexia from chemo and radiation for cancer that requires nutritional supplementation?

patient feeling nausea, vomiting, sores in mouth, no appetite, may need to TPN, difficulty swallowing

192. A nurse is caring for an older adult patient during the immediate postoperative period after a total hip replacement. The surgeon has ordered meperidine (Demerol) for severe pain. What will the nurse do? a. Administer the medication as prescribed and initiate a fall risk protocol. b. Ask for a PRN order for diphenhydramine (Benadryl) for the expected side effect of itching. c. Request an order for morphine instead of meperidine (Demerol). d. Suggest to the surgeon that the patient receive diazepam (Valium) to reduce anxiety and the need for narcotics.

c. Request an order for morphine instead of meperidine (Demerol). In older adults, meperidine is not effective at usual doses and causes more confusion than in younger patients. Morphine is recommended for severe pain. A fall risk protocol is appropriate, but the drug ordered is not. Diphenhydramine is not recommended for older adult patients, because it causes blurred vision. Both diphenhydramine and diazepam have central nervous system (CNS) sedative effects, which will compound the CNS effects of the narcotic. Diazepam also produces prolonged sedation in older adults.

127. A nurse is caring for a newborn 1 day after delivery. The infant's mother used betaxolol during pregnancy. The nurse will expect to monitor this infant for which condition? a. Hyperglycemia b. Hyperthyroidism c. Respiratory distress d. Tachycardia

c. Respiratory distress Use of beta blockers during pregnancy can affect a newborn for several days after birth. Newborns are at risk for bradycardia, respiratory distress, and hypoglycemia. Hyperglycemia, hyperthyroidism, and tachycardia are not expected residual effects.

239. A patient claims to get better effects with a tablet of Brand X of a drug than with a tablet of Brand Y of the same drug. Both brands contain the same amount of the active ingredient. What does the nurse know to be most likely? a. Advertising by pharmaceutical companies can enhance patient expectations of one brand over another, leading to a placebo effect. b. Because the drug preparations are chemically equivalent, the effects of the two brands must be identical. c. Tablets can differ in composition and can have differing rates of disintegration and dissolution, which can alter the drug's effects in the body. d. The bioavailability of a drug is determined by the amount of the drug in each dose.

c. Tablets can differ in composition and can have differing rates of disintegration and dissolution, which can alter the drug's effects in the body. Even if two brands of a drug are chemically equivalent (i.e., they have identical amounts of the same chemical compound), they can have different effects in the body if they differ in bioavailability. Tablets made by different manufacturers contain different binders and fillers, which disintegrate and dissolve at different rates and affect the bioavailability of the drug. Two brands may be chemically equivalent and still differ in bioavailability, which is not determined by the amount of drug in the dose.

208. A patient is being discharged after surgery. During the admission history, the nurse had learned that the patient normally consumes two or three glasses of wine each day. The prescriber has ordered hydrocodone with acetaminophen (Lortab) for pain. What will the nurse do? a. Request an order for acetaminophen without hydrocodone for pain. b. Suggest that the patient use ibuprofen for pain. c. Tell the patient not to drink wine while taking the Lortab. d. Tell the patient to limit his wine intake to one or two glasses per day.

c. Tell the patient not to drink wine while taking the Lortab. Combining a hepatotoxic drug with certain other drugs may increase the risk of hepatotoxicity. When even therapeutic doses of acetaminophen are taken with alcohol, the acetaminophen can cause liver damage. Patients should be cautioned not to drink alcohol; even two drinks with acetaminophen can produce this effect. Hydrocodone does not contribute to hepatotoxicity. Ibuprofen is not indicated for postoperative pain unless the pain is mild. Limiting wine to one or two glasses per day still increases the risk of hepatotoxicity

247. A patient is receiving intravenous gentamicin. A serum drug test reveals toxic levels. The dosing is correct, and this medication has been tolerated by this patient in the past. Which could be a probable cause of the test result? a. A loading dose was not given. b. The drug was not completely dissolved in the IV solution. c. The patient is taking another medication that binds to serum albumin. d. The medication is being given at a frequency that is longer than its half-life.

c. The patient is taking another medication that binds to serum albumin. Gentamicin binds to albumin, but only weakly and, in the presence of another drug that binds to albumin, can rise to toxic levels in blood serum. A loading dose increases the initial amount of a drug and is used to bring drug levels to the desired plateau more quickly. A drug that is not completely dissolved carries a risk of causing embolism. A drug given at a frequency longer than the drug half-life will likely be at subtherapeutic levels and not at toxic levels

216. A nursing student is preparing to give a medication that has a black box warning. The student asks the nurse what this means. What will the nurse explain about black box warnings? a. They indicate that a drug should not be given except in life-threatening circumstances. b. They provide detailed information about the adverse effects of the drug. c. They alert prescribers to ways to mitigate potential harm from side effects. d. They provide information about antidotes in the event toxicity occurs.

c. They alert prescribers to ways to mitigate potential harm from side effects. Black box warnings are used to alert providers to potential side effects and to ways to prevent or reduce harm from these side effects. The black box warning is placed on any drug that, although useful, has serious side effects; this is a way to keep drugs on the market while protecting patients. Many of these drugs are used in situations that are not life threatening. The black box warning provides a concise summary and not a detailed explanation of drug side effects. The black box warning does not include antidotes to toxicity

203. A patient has been taking narcotic analgesics for chronic pain for several months. The nurse caring for this patient notes that the prescribed dose is higher than the recommended dose. The patient has normal vital signs, is awake and alert, and reports mild pain. What does the nurse recognize about this patient? a. This patient exhibits a negative placebo effect with a reduced response to the drug. b. This patient has developed a reaction known as tachyphylaxis because of repeated exposure to the drug. c. This patient has developed pharmacodynamic tolerance, which has increased the minimal effective concentration (MEC) needed for analgesic effect. d. This patient produces higher than normal hepatic enzymes as a result of prolonged exposure to the drug.

c. This patient has developed pharmacodynamic tolerance, which has increased the minimal effective concentration (MEC) needed for analgesic effect. Pharmacodynamic tolerance results when a patient takes a drug over a period of time. Adaptive processes occur in response to chronic receptor occupation. The result is that the body requires increased drug, or an increased MEC, to achieve the same effect. This patient is getting adequate pain relief, so there is no negative placebo effect. Tachyphylaxis is a form of tolerance that can be defined as a reduction in drug responsiveness brought on by repeated dosing over a short time; this occurs over several months. Barbiturates induce synthesis of hepatic enzymes that cause increased metabolism of the drug, but it does not increase the MEC.

238. A patient tells the nurse that the oral drug that has been prescribed has caused a lot of stomach discomfort in the past. What will the nurse ask the prescriber? a. Whether a sublingual form of the medication can be given b. Whether the medication can be given by a parenteral route instead c. To order an enteric-coated form of the drug d. Whether the patient can receive a sustained-release preparation of the drug

c. To order an enteric-coated form of the drug Enteric-coated drugs are preparations that have been coated with a material that dissolves in the intestines, not the stomach. This coating is used either to protect the drug from stomach acid and pepsin or to protect the stomach from a drug that can cause gastric upset. Sublingual forms often are used for drugs that undergo rapid inactivation during the first pass through the hepatic circulation so that the drug can be absorbed directly into the systemic circulation. Parenteral routes are more costly and less safe than oral administration and should not be used unless necessary. A sustained-release preparation is used to release the drug into the body over a specific period to reduce the number of daily doses required to sustain therapeutic drug levels.

139. A patient is admitted to the intensive care unit for treatment of shock. The prescriber orders isoproterenol (Isuprel). The nurse expects this drug to increase tissue perfusion in this patient by activating: a. alpha1 receptors to cause vasoconstriction. b. alpha1 receptors to increase blood pressure. c. beta1 receptors to cause a positive inotropic effect. d. beta2 receptors to cause bronchodilation.

c. beta1 receptors to cause a positive inotropic effect. Isoproterenol is used for shock because of its actions on beta1 receptors in the heart. It has a positive inotropic effect, which improves cardiac output and thus increases tissue perfusion. Isoproterenol does not affect alpha1 receptors. Although it activates beta2 receptors to cause bronchodilation, this is not indicated for the treatment of shock.

232. A patient reports becoming "immune" to a medication because it no longer works to alleviate symptoms. The nurse recognizes that this decreased effectiveness is likely caused by: a. antagonists produced by the body that compete with the drug for receptor sites. b. decreased selectivity of receptor sites, resulting in a variety of effects. c. desensitization of receptor sites by continual exposure to the drug. d. synthesis of more receptor sites in response to the medication.

c. desensitization of receptor sites by continual exposure to the drug. Continual exposure to an agonist would cause the cell to become less responsive or desensitized. The body does not produce antagonists as a response to a medication. Receptor site selectivity is determined by physiologic factors and not by the substances that bind to them. Medications do not cause more receptors to be produced.

182. A nurse is administering drug X to a patient. The drug information states that the drug acts by activating receptors in the peripheral nervous system by increasing transmitter synthesis. The nurse understands that the effect of this drug is to: a. activate axonal conduction. b. enhance transmitter storage. c. increase receptor activation. d. synthesize supertransmitters.

c. increase receptor activation. Drugs that increase transmitter synthesis increase receptor activation. Other drugs that alter transmitter synthesis can decrease synthesis and would cause decreased receptor activation. Drugs that affect transmitter production do not exert their effects on axonal conduction. The amount of transmitter produced does not directly affect transmitter storage. Some drugs that alter transmitter synthesis cause the synthesis of transmitter molecules that are more effective than the transmitter itself.

149. A patient receives a neuromuscular blocking agent before a procedure. The patient's eyes close. The nurse knows this is a sign that the patient: a. has fallen asleep. b. has received a toxic dose of the medication. c. is beginning to feel the drug's effects. d. may need mechanical ventilation

c. is beginning to feel the drug's effects. Neuromuscular blocking agents relax skeletal muscle to produce flaccid paralysis; the first muscles to be affected are the levator muscle of the eyelids and the muscles of mastication. These agents do not affect the CNS. Because the drug has just taken effect, a toxic dose is not yet a concern. Mechanical ventilation is necessary when the drug affects the intercostal muscles and the diaphragm, interfering with respiration.

201. A patient asks a nurse why a friend who is taking the same drug responds differently to that drug. The nurse knows that the most common variation in drug response is due to differences in each patient's: a. drug receptor sites. b. hypersensitivity potential. c. metabolism of drugs. d. psychosocial response.

c. metabolism of drugs. The most common source of genetic variation in drug response is related to alterations in drug metabolism and is determined by genetic codes for various drug-metabolizing isozymes. There are known genetic differences in codes for drug target sites, but these are not as numerous as those for metabolic isozymes. Hypersensitivity potential is also genetically determined, but variations produce differences in adverse reactions to drugs and not in drug effectiveness. Psychosocial responses vary for many, less measurable reasons, such as individual personalities and variations in cultures.

169. A patient receives topical atropine to facilitate an eye examination. The nurse will tell the patient to remain in a darkened room or to wear sunglasses for several hours until the effects of the medication wear off. This teaching is based on the nurse's knowledge that muscarinic antagonists cause: a. elevation of intraocular pressure. b. miosis and ciliary muscle contraction c. paralysis of the iris sphincter. d. relaxation of ciliary muscles.

c. paralysis of the iris sphincter. By blocking muscarinic receptors in the eye, atropine causes paralysis of the iris sphincter, which prevents constriction of the pupil; consequently, the eye cannot adapt to bright light. This also causes an elevation in intraocular pressure, which increases the risk of glaucoma. However, it is not an indication for wearing darkened glasses. Muscarinic agonists cause miosis; atropine causes mydriasis. The effect of relaxing ciliary muscles focuses the eye for far vision, causing blurred vision.

246. A nurse is explaining drug metabolism to a nursing student who asks about glucuronidation. The nurse knows that this is a process that allows drugs to be: a. excreted in hydrolyzed form in the feces to reduce drug toxicity. b. reabsorbed from the urine into the renal circulation to minimize drug loss. c. recycled via the enterohepatic recirculation to remain in the body longer. d. transported across the renal tubules to be excreted in the urine.

c. recycled via the enterohepatic recirculation to remain in the body longer. Glucuronidation of some drugs in the liver allows drugs to enter the bile, pass into the duodenum, and then be hydrolyzed to release the free drug. This is a repeating cycle of enterohepatic recirculation, which allows drugs to remain in the body longer. Glucuronidated drugs that are more resistant to hydrolysis are excreted in the feces. Glucuronidation occurs in the enterohepatic circulation and not in the renal circulation.

What is a nurses, immediate concern for a patient that is on dobutamine (Dobutex) drip for heart failure that has been feeling better but now has tightness in the chest, palpitations, as well as a bit of anxiety?

careful titration and monitoring of vital signs and ECG are required. Patient could be in toxic overload

What is a concern with Afrin nose drops when they are stopped suddenly?

could cause damage to your nasal tissue and lead to chronic congestion. (rebound congestion in which the nasal passages become more congested as the effects of the drug wear off.)

133. A nursing student asks the nurse about receptor specificity of adrenergic agonist medications. What will the nurse say? a. "As the dosage of these medications increases, drugs in this class are more selective." b. "Dopamine is selective for dopamine receptors and has no effects on alpha or beta receptors." c. "Epinephrine is the most selective alpha-adrenergic agonist medication." d. "Specificity is relative and is dose dependent."

d. "Specificity is relative and is dose dependent." Receptor specificity is relative, not absolute, and is dose dependent. At low doses, selectivity is maximal, and selectivity declines as the dose increases. Dopamine is the only drug in this class that acts on dopamine receptors, but it also has effects on alpha and beta receptors. Epinephrine is the least selective of this class.

165. A patient will begin using a transdermal preparation of a muscarinic antagonist for overactive bladder (OAB). The nurse teaches the patient what to do if side effects occur. Which statement by the patient indicates the need for further teaching? a. "I can use sugar-free gum for dry mouth." b. "I may need laxatives for constipation." c. "I should keep the site covered to prevent other people from getting the medicine." d. "I will take Benadryl for any itching caused by a local reaction to the patch."

d. "I will take Benadryl for any itching caused by a local reaction to the patch." Benadryl is an antihistamine, and even though it is not classified as a muscarinic antagonist, it has anticholinergic effects. Giving it with a muscarinic antagonist greatly enhances these effects, so it should not be used. Muscarinic antagonists cause dry mouth, and patients should be taught to use sugar-free gum or candies to help with this. Muscarinic antagonists can cause constipation, and laxatives may be used. Medication applied topically can be transferred to others who come in contact with the skin, so the site should be covered

268. A patient tells the nurse that he was told by the prescriber that the analgesic he is taking is very effective. Which statement by the patient demonstrates an understanding of the drug's effectiveness? a. "I don't have to worry about toxicity, since it takes a large amount of this drug to cause an overdose." b. "It has no side effects and doesn't interact with other drugs." c. "I only have to take it every 12 hours." d. "It might make me sleepy, and it lessens pain for several hours at a time."

d. "It might make me sleepy, and it lessens pain for several hours at a time." A drug is effective if it produces the intended effects, even if it also produces side effects. Because no drug is completely safe, the level of toxicity does not determine effectiveness. All drugs have side effects and many react with other substances; these do not affect the drug's effectiveness. Ease of administration is independent of a drug's effectiveness.

173. A nurse is teaching a group of nursing students about neurotransmitters. Which statement by a student about acetylcholine indicates a need for further teaching? a. "It activates three cholinergic receptor subtypes." b. "It has effects in the parasympathetic, sympathetic, and somatic nervous systems." c. "It is used at most junctions of the peripheral nervous system." d. "Its transmission is terminated by reuptake into the cholinergic nerve terminal."

d. "Its transmission is terminated by reuptake into the cholinergic nerve terminal." Acetylcholine (ACh) is destroyed by acetylcholinesterase, an enzyme that degrades ACh into two inactive products: acetate and choline. Choline is taken up into the nerve terminal. ACh activates three receptor sites: nicotinicN, nicotinicM, and muscarinic. ACh has effects throughout the peripheral nervous system (PNS) and is used at most junctions in the PNS.

213. A nurse provides teaching to a patient who will begin taking a drug with a known risk of hepatotoxicity. Which statement by the patient indicates a need for further teaching? a. "I should avoid taking acetaminophen while taking this drug." b. "I will need periodic evaluation of aspartate aminotransferase and alanine aminotransferase levels." c. "If I experience nausea, vomiting, or abdominal pain, I should call my provider." d. "Routine testing and early detection of problems will prevent liver failure."

d. "Routine testing and early detection of problems will prevent liver failure." Drug-induced liver injury can progress from undetectable to advanced between routine tests; therefore, routine testing does not always prevent liver failure. Patients taking known hepatotoxic drugs should avoid other drugs, such as acetaminophen, that can cause liver damage. Aspartate aminotransferase (AST) and alanine aminotransferase (ALT) are liver enzymes that are routinely monitored when a patient is taking hepatotoxic drugs. Nausea, vomiting, and abdominal pain are all signs of liver injury and should be reported

What is the purpose of the evaluation phase of the nursing process

measure how well the patient has achieved desired outcomes during and after each intervention (involves the patient and nurse) -identify factors contributing to the patient's success or failure -modify the plan of care and make changes necessary based on why the current plan isn't effective

186. A nursing student asks about drugs that interfere with the termination of transmitter action. Which statement by the nurse is correct? a. "Drugs act on this process by altering the diffusion of the transmitter away from the synaptic gap." b. "Drugs can interfere with termination by either increasing or decreasing reuptake of the transmitter." c. "Drugs in this category lead to decreased activation by the transmitter in the synapse." d. "These drugs reduce either reuptake or degradation of the transmitter, causing an increase in receptor activation."

d. "These drugs reduce either reuptake or degradation of the transmitter, causing an increase in receptor activation." Drugs that interfere with termination of transmitter action do so by blocking transmitter reuptake or inhibiting transmitter degradation, resulting in increased receptor activation, because more of the transmitter remains available. Diffusion of the transmitter occurs naturally, but it is a slow process with little clinical significance. Drugs that alter this process cause a decrease in reuptake, not an increase. The effect of drugs that interfere with termination of transmitter action is increased activation.

243. The nurse is preparing to administer penicillin G intramuscularly to a child. The child's parents ask why the drug cannot be given in an oral liquid form. What is the nurse's reply? a. "This drug causes severe gastric upset if given orally." b. "This drug has a narrow therapeutic range, and the dose must be tightly controlled." c. "This drug is absorbed much too quickly in an oral form." d. "This drug would be inactivated by enzymes in the stomach."

d. "This drug would be inactivated by enzymes in the stomach." Penicillin G is inactivated by digestive enzymes in the stomach and cannot be given orally. It does not have a narrow therapeutic range.

109. A prescriber has ordered methyldopa for a patient with hypertension. The nurse teaches the patient about drug actions, adverse effects, and the ongoing blood tests necessary with this drug. The nurse is correct to tell the patient what? a. "If you have a positive Coombs' test result, you will need to discontinue the medication, because this means you have hemolytic anemia." b. "Methyldopa can be used for its analgesic effects and for its hypertensive effects." c. "Xerostomia and orthostatic hypotension are serious side effects and indications for withdrawing the medication." d. "You will need to contact the provider and stop taking the medication if your eyes look yellow."

d. "You will need to contact the provider and stop taking the medication if your eyes look yellow." Hepatotoxicity is a serious adverse effect of methyldopa and is an indication for withdrawal of the drug to prevent fatal hepatic necrosis. Jaundice is a sign of liver toxicity. Patients should undergo periodic liver function tests while taking the drug. Liver function usually improves when the drug is withdrawn. A positive Coombs' test result is not an indication for withdrawal of the drug in itself. About 5% of patients with a positive Coombs' test result develop hemolytic anemia; withdrawal of the drug is indicated for those patients. Methyldopa does not have analgesic effects. Xerostomia and orthostatic hypotension are known side effects of methyldopa but usually are not serious.

209. A nurse is reviewing a medication administration record before administering medications. Which order should the nurse implement? a. Furosemide (Lasix) 20 mg QD PO b. Furosemide (Lasix) 20 mg qd PO c. Furosemide (Lasix) 20 mg daily d. Furosemide (Lasix) 20 mg PO daily

d. Furosemide (Lasix) 20 mg PO daily The correct answer is a complete order; it contains the medication, dose, route, and time. QD is no longer an accepted abbreviation; it should be written out as "daily" or "every day." The order of "20 mg daily" does not specify the route to be used.

178. A patient is to receive a beta agonist. Before administration of this medication, which assessment finding would most concern the nurse? a. Pulse oximetry reading of 88% b. Blood pressure of 100/60 mm Hg c. Respiratory rate of 28 breaths per minute d. Heart rate of 110 beats per minute

d. Heart rate of 110 beats per minute A beta agonist dilates respiratory smooth muscle, but as a side effect, it can stimulate the heart. A heart rate of 110 beats per minute is a concern, because this medication may further increase the already elevated heart rate. A pulse oximetry reading of 88% is a concern, but the medication causes bronchodilation and increased oxygenation; this should increase the pulse oximetry reading. A blood pressure of 100/60 mm Hg is on the low side, but this medication may actually cause an increase in blood pressure as a side effect; this should not concern the nurse before administration of the medication. A respiratory rate of 28 breaths per minute is elevated; however, this medication should increase oxygenation by bronchodilation, and the patient's respiratory rate should decrease once oxygenation has improved. Therefore, this should not concern the nurse.

264. A patient has been receiving intravenous penicillin for pneumonia for several days and begins to complain of generalized itching. The nurse auscultates bilateral wheezing and notes a temperature of 38.5° C (101° F). Which is the correct action by the nurse? a. Administer the next dose and continue to evaluate the patient's symptoms. b. Ask the prescriber if an antihistamine can be given to relieve the itching. c. Contact the prescriber to request an order for a chest radiograph. d. Hold the next dose and notify the prescriber of the symptoms.

d. Hold the next dose and notify the prescriber of the symptoms. Pruritus and wheezing are signs of a possible allergic reaction, which can be fatal; therefore, the medication should not be given and the prescriber should be notified. When patients are having a potentially serious reaction to a medication, the nurse should not continue giving the medication. Antihistamines may help the symptoms of an allergic reaction, but the first priority is to stop the medication. Obtaining a chest radiograph is not helpful.

144. While preparing a patient for a procedure in which a neuromuscular blocking agent will be used, the nurse reviews the patient's preprocedure laboratory values. Which abnormality would cause the most concern? a. Hyponatremia b. Hypercalcemia c. Hypomagnesemia d. Hypokalemia

d. Hypokalemia Low potassium levels can enhance paralysis, whereas high potassium levels can reduce paralysis. Because electrolyte status can influence the depth of neuromuscular blockade, it is important to maintain normal electrolyte balance. There are no indications that sodium, calcium, and/or magnesium have effects similar to those of potassium in relation to neuromuscular blocking agents.

234. A patient who is taking morphine for pain asks the nurse how a pain medication can also cause constipation. What does the nurse know about morphine? a. It binds to different types of receptors in the body. b. It can cause constipation in toxic doses. c. It causes only one type of response, and the constipation is coincidental. d. It is selective to receptors that regulate more than one body process.

d. It is selective to receptors that regulate more than one body process. Morphine is a medication that is selective to receptor type that regulates more than one process. Because it is selective to receptor type, it does not bind to different types of receptors. Constipation is a normal side effect and is not significant for toxicity.

145. Which of the following areas of the body shows the effects of tubocurarine last? a. Levator muscle of the eyelids and the muscles of mastication b. Muscles in the lower extremities c. Muscles controlling the glottis d. Muscles of respiration and the diaphragm

d. Muscles of respiration and the diaphragm Although tubocurarine can paralyze all skeletal muscles, not all muscles are affected at once. The last muscles affected by tubocurarine are the muscles of respiration, including the intercostals, and the diaphragm. The first to become paralyzed are the levator muscle of the eyelids and the muscles of mastication. Paralysis occurs next in the muscles of the limbs, abdomen, and glottis.

160. A patient is experiencing toxic side effects from atropine, including delirium and hallucinations. Which medication will the nurse expect to administer? a. Donepezil (Aricept) b. Edrophonium (Reversol) c. Neostigmine (Prostigmin) d. Physostigmine

d. Physostigmine Physostigmine is the drug of choice for treating poisoning from atropine and other drugs that cause muscarinic blockade. Donepezil is used to treat Alzheimer's disease. Edrophonium is used to distinguish between a myasthenic crisis and a cholinesterase inhibitor overdose. Neostigmine does not cross the blood-brain barrier and would not effectively treat this patient's CNS symptoms.

219. The nurse is teaching a patient about taking warfarin and asks if the patient takes aspirin. This assessment by the nurse reflects a knowledge of which type of drug interaction? a. Creation of unique effects b. Increased therapeutic effects c. Inhibitory effects d. Potentiative effects

d. Potentiative effects A potentiative effect is one in which one drug intensifies the effects of another. Both warfarin and aspirin suppress blood clotting, and the combination may increase the risk of bleeding, which is an intensified adverse effect. Creation of a unique effect is a rare occurrence in which the combination of two drugs creates a response not seen with either drug when given alone. Increased therapeutic effects are a type of potentiative effect; however, in this case the combination of two drugs would increase the desired effects. An inhibitory effect is a type of pharmacodynamic effect that occurs when an antagonist drug inhibits the action of an agonist drug at the same receptor site.

105. A nurse is assisting a physician who is performing a circumcision on a newborn. The physician asks the nurse to prepare lidocaine and epinephrine for injection to provide anesthesia. What will the nurse do? a. Ask the provider why an injectable anesthetic is being used for this procedure. b. Draw up the medication as ordered and prepare the infant for the procedure. c. Make sure that seizure precautions are in place. d. Question the use of the epinephrine for this procedure.

d. Question the use of the epinephrine for this procedure. The physician is preparing to use infiltration anesthesia by injecting the local anesthetic directly into the immediate area of surgery. Epinephrine can be used in some cases but should never be used in areas supplied by end arteries, such as the penis, toes, fingers, nose, or ears, because restriction of blood flow in these areas can result in gangrene. Injectable agents are appropriate for this procedure. The nurse should not draw up the medication as requested, because the combination of agents can harm the patient. Seizure precautions are not necessary.

147. While preparing a patient for a second esophageal dilation procedure, the nurse explains that succinylcholine (Anectine) will be used for muscle relaxation. The patient is anxious and reports not being able to swallow for several hours after the previous procedure. What will the nurse do? a. Be prepared to provide mechanical ventilation after the procedure. b. Have dantrolene available, because this patient is at increased risk for side effects. c. Reassure the patient that this is expected after neuromuscular blockade. d. Request an order for a pseudocholinesterase level.

d. Request an order for a pseudocholinesterase level Succinylcholine is used for muscle relaxation during short procedures, such as esophageal dilation, because of its short duration. The effects wear off minutes after the drug is withdrawn, because plasma pseudocholinesterase quickly degrades it. Patients who have prolonged effects may have low plasma pseudocholinesterase levels and should be evaluated if this is suspected. Mechanical ventilation may be necessary if the drug is given, but the prudent nurse would rather avoid this by taking preventive measures instead of treating the adverse reaction after the fact. Dantrolene should always be available when succinylcholine is used because of the risk of malignant hyperthermia; however, nothing indicates that this particular patient is at greater risk than others. Prolonged neuromuscular blockade is not a normal reaction.

122. A patient has benign prostatic hyperplasia (BPH) and will receive alfuzosin (Uroxatral). What is an important nursing action when administering this drug? a. Administer the first dose at bedtime to minimize the first-dose effect. b. Prepare the patient for potential ejaculatory dysfunction. c. Request an order for a diuretic to counter the effects of sodium retention. d. Review the patient's chart for liver function tests (LFTs).

d. Review the patient's chart for liver function tests (LFTs). Alfuzosin is indicated for BPH only and is contraindicated in patients with moderate to severe hepatic impairment; therefore, LFTs would be an important assessment. It does not interfere with ejaculation. It does not reduce blood pressure and does not cause sodium retention or increased blood volume, so diuretics are not necessary. It does not cause orthostatic hypotension, so first-dose effect precautions are not necessary

202. A nurse is preparing to care for a patient who is receiving digoxin. To help minimize the potential for adverse effects from this drug, the nurse will review which of this patient's laboratory results? a. Albumin b. Blood urea nitrogen (BUN) and creatinine c. Hepatic enzymes d. Serum electrolytes

d. Serum electrolytes Patients with low serum potassium are at risk for fatal cardiac dysrhythmias when taking digoxin, and it is essential to know this level before this medication is administered. Knowing a patient's albumin level would be important when giving drugs that are protein bound. The BUN and creatinine levels are indicators of renal function. Hepatic enzymes are important to know when drugs are metabolized by the liver.

225. Which statement about food and drug interactions is true? a. Foods alter drug absorption and metabolism but not drug action. b. Medications are best absorbed on an empty stomach. c. Patient discomfort is the food and drug interaction of most concern. d. Some foods can inhibit CYP isozymes and alter drug metabolism.

d. Some foods can inhibit CYP isozymes and alter drug metabolism. Grapefruit juice inhibits CYP3A4, which lowers the metabolism of some drugs, leading to toxic effects of drugs affected by these isozymes. Foods can alter all pharmacokinetic and pharmacodynamic processes. Not all medications are absorbed better on an empty stomach; some require certain foods to enhance absorption. Patient comfort is a concern, but it is not as important as more severe and possibly life-threatening food and drug interactions.

255. A patient is using a metered-dose inhaler containing albuterol for asthma. The medication label instructs the patient to administer "2 puffs every 4 hours as needed for coughing or wheezing." The patient reports feeling jittery sometimes when taking the medication, and she doesn't feel that the medication is always effective. Which is not an appropriate nursing intervention for this patient? a. Asking the patient to demonstrate use of the inhaler b. Assessing the patient's exposure to tobacco smoke c. Auscultating lung sounds and obtaining vital signs d. Suggesting that the patient use one puff to reduce side effects

d. Suggesting that the patient use one puff to reduce side effects It is not within the nurse's scope of practice to change the dose of a medication without an order from a prescriber. Asking the patient to demonstrate inhaler use helps the nurse to evaluate the patient's ability to administer the medication properly and is part of the nurse's evaluation. Assessing tobacco smoke exposure helps the nurse determine whether nondrug therapies, such a smoke avoidance, can be used as an adjunct to drug therapy. Performing a physical assessment helps the nurse evaluate the patient's response to the medication.

143. A nurse is caring for an intubated patient who is receiving pancuronium for neuromuscular blockade. The patient's eyes are closed, and the patient is not moving any extremities. The heart rate is 76 beats per minute, and the blood pressure is 110/70 mm Hg. The nurse caring for this patient will do what? a. Request an order for an antihistamine to prevent a further drop in blood pressure. b. Request an order for serum electrolytes to evaluate for hyperkalemia. c. Review the patient's chart for a history of myasthenia gravis (MG). d. Talk to the patient while giving care and explain all procedures.

d. Talk to the patient while giving care and explain all procedures. Pancuronium is a competitive neuromuscular blocker and is used to paralyze skeletal muscles by neuromuscular blockade. The first muscles affected are those of the eyelids, so patients given this medication cannot open their eyes. Inability to move the extremities is an expected effect. Because these medications do not have central nervous system (CNS) effects, the patient is conscious and awake; therefore, caregivers should continue to talk to the patient while providing care. Atracurium, not pancuronium, can cause hypotension secondary to histamine release. High potassium levels would reduce paralysis. Patients with a history of MG would have increased paralysis. Nothing indicates that this patient is experiencing extreme effects from this medication.

191. A nurse is preparing to give medications to four geriatric patients who are all taking multiple medications. Which patient is most likely to have an adverse drug reaction related to increased drug effects? a. Obese patient b. Patient with decreased serum creatinine c. Patient with chronic diarrhea d. Thin patient with a chronically low appetite

d. Thin patient with a chronically low appetite The patient who is thin and has a poor appetite has an increased risk of malnutrition, with significant lowering of serum albumin. This can result in increased free drug levels of protein-bound drugs and can lead to drug toxicity. Obesity, which involves increased adipose tissue, would cause lipid-soluble drugs to deposit in adipose tissue, with a resulting reduction of drug effects. Decreased serum creatinine in an older adult patient may just be a function of a decrease in lean body mass and not of renal function. Chronic diarrhea would accelerate the passage of medications through the GI tract and reduce absorption.

115. A prescriber orders transdermal clonidine (Catapres TTS) for a patient with hypertension. What will the nurse teach this patient? a. That medication given by transdermal patch has fewer systemic side effects b. That localized skin reactions are uncommon c. To apply the patch to intact skin on the forearm or leg d. To change the patch every week

d. To change the patch every week Transdermal patches are to be changed every 7 days. Medication administered by patch has the same therapeutic effect and adverse effects as that given by other routes, except that localized skin reactions may occur and are common with clonidine patches. The patch should be applied to intact, hairless skin on the upper arm or torso.

121. A nurse prepares to administer propranolol (Inderal) to a patient recovering from acute myocardial infarction. The patient's heart rate is 52 beats per minute, and the rhythm is regular. What action should the nurse take next? a. Administer the drug as prescribed. b. Request an order for atropine. c. Withhold the dose and document the pulse rate. d. Withhold the dose and notify the prescriber.

d. Withhold the dose and notify the prescriber. A beta blocker, such as propranolol, should not be given if the pulse is lower than 60 beats per minute; therefore, the nurse should withhold the dose and notify the prescriber. Administering the dose as prescribed would not be appropriate, because the patient's pulse rate is too slow at this time. The dose should be withheld and the prescriber notified. The patient's heart rate is slow, and atropine may be necessary if the bradycardia persists, but the first step is to withhold the dose of propranolol. Withholding the dose and documenting the pulse rate is an appropriate but incomplete nursing intervention. The nurse must notify the prescriber to obtain further orders related to the medication.

266. The nurse is teaching a patient how a medication works to treat an illness. To do this, the nurse will rely on knowledge of: a. clinical pharmacology. b. drug efficacy. c. pharmacokinetics. d. pharmacotherapeutics.

d. pharmacotherapeutics. Pharmacotherapeutics is the study of the use of drugs to diagnose, treat, and prevent conditions. Clinical pharmacology is concerned with all aspects of drug-human interactions. Drug efficacy measures the extent to which a given drug causes an intended effect. Pharmacokinetics is the study of the impact of the body on a drug.

179. A patient receives morphine and shows signs of toxicity. The prescriber orders naloxone (Narcan) to reverse the effects of the morphine. The nurse understands that the naloxone works at the same receptor sites as the morphine to: a. block transmitter reuptake. b. inhibit transmitter release. c. interfere with transmitter storage. d. prevent activation of receptors.

d. prevent activation of receptors. Morphine and its antagonist, naloxone, both act directly at the same receptors. Morphine causes activation, and naloxone prevents activation. Neither morphine nor naloxone acts to alter transmitter reuptake, release, or storage.

224. A child ingests a parent's aspirin tablets, and the prescriber orders sodium bicarbonate to block the toxic effects of the aspirin. The nurse caring for this patient knows that sodium bicarbonate is effective against the aspirin because it: a. accelerates its passage through the intestine. b. alters urinary pH to enhance renal excretion. c. induces CYP isozymes to increase drug metabolism. d. raises the pH of the interstitial fluid to facilitate passage out of the cells.

d. raises the pH of the interstitial fluid to facilitate passage out of the cells. Sodium bicarbonate increases the pH of interstitial fluid and plasma, allowing the acidic aspirin ions to move outside the cells and thus removing them from the site where they have toxic effects. It does not have laxative effects and does not alter the rate of passage through the gut. It is not a CYP isozyme inducer and therefore has no effect on drug metabolism. It does not alter renal excretion of aspirin.

210. A patient is given a drug for the first time and develops shortness of breath. The patient's heart rate is 76 beats per minute, the respiratory rate is 20 breaths per minute, and the blood pressure is 120/70 mm Hg. The nurse checks a drug administration manual to make sure the correct dose was given and learns that some patients taking the drug experience shortness of breath. The nurse will contact the provider to report a(n): a. allergic reaction. b. idiosyncratic effect. c. iatrogenic response. d. side effect.

d. side effect. A side effect is a secondary drug effect produced at therapeutic doses. This patient received the correct dose of the drug and developed shortness of breath which, in this case, is a drug side effect. To experience an allergic reaction, a patient must have prior exposure to a drug and sensitization of the immune response. An idiosyncratic effect results from a genetic predisposition to an uncommon drug response. An iatrogenic response occurs when a drug causes symptoms of a disease.

253. A patient asks a nurse why drugs that have been approved by the FDA still have unknown side effects. The nurse tell the patient that: a. testing for all side effects of a medication would be prohibitively expensive. b. patients in drug trials often are biased by their preconceptions of a drug's benefits. c. researchers tend to conduct studies that will prove the benefits of their new drugs. d. subjects in drug trials do not always represent the full spectrum of possible patients.

d. subjects in drug trials do not always represent the full spectrum of possible patients. All drug trials are limited by a relatively small group of subjects who may not have all the characteristics of people who will be using the drug; therefore, some side effects go undetected until the drug is in use. Although drug trials are very expensive, this is only an indirect reason they do not detect all side effects before approval. In theory, well-designed drug trials, using blinded studies, minimize or eliminate subject bias. Designing studies to prove desired results is unethical.

153. A nurse is caring for a patient who has myasthenia gravis. The prescriber has ordered neostigmine (Prostigmin). An important initial nursing action before administration of the medication includes assessing: a. the ability to raise the eyelids. b. level of fatigue. c. skeletal muscle strength. d. swallowing ability.

d. swallowing ability. Many patients hospitalized for myasthenia gravis do not have the muscle strength to swallow well and need a parenteral form of the medication; therefore, assessing the patient's ability to swallow is an important initial safety measure. Evaluating the patient's ability to raise the eyelids, level of fatigue, and skeletal muscle strength are all important assessments both before drug administration and during drug treatment, because they indicate the effectiveness of the drug and help determine subsequent doses.

Signs and symptoms of Ketoacidosis

excessive thirst weakness frequent urination confusion, abdominal pain dry skin, mouth increased heart rate vomiting low blood pressure fruity odor on breath

Adverse Effects of Sumatriptan

feelings of tingling, flushing, and a congested feeling in the head or chest. Nausea, vomiting, cold or clammy hands and feet, muscle pain dizziness, numbness, and a vague feeling of anxiety, a bitter or foul taste in the mouth or throat, and irritation of the nose.

What are the side effects of pramipexole (Mirapex) that is less likely to occur with other dopamine agonist?

fewer dyskinesias

What is the length of time a patient will be treated for fungal infections?

fungal infections are present in the mouth, vagina, and intestinal tract. Periods of treatment can range from several weeks to as long as 1 year. Most common adverse effects of topical antifungals are local irritation, pruritus, a burning sensation, and scaling.

What are the contraindications of oral laxatives?

impaction above strictures fluid disturbances electrolyte imbalances gas formation esophageal blockage allergic reaction adverse effects of dry mouth may be helped by frequent mouth care, fluid intake, or use sugarless gum or candy.

What is the appropriate action when administering amphotericin B (Fungizone)

intravaneous amphotericin B, solutions that are cloudy or have precipitates should not be administered, IV pump is recommended, vital signs should be monitored every 15 minutes, IV site should be monitored for signs of phlebitis

Teaching plan for Retin - A

is a derivative of vitamin A that is used to treat acne and ameliorate (fine wrinkling, mottled hyperpigmentation,roughness) with photodamage. Patients should maintain adequate general hygiene avoid sunlight avoid abrasive cleaners Retin - A also treats leukemia

Gabapentin is a ?

may be used for seizure therapy, but it is also used to treat postherpetic neuralgia and neuropathic pain, and to prevent migranes. Drug interactions include; antacids and hydrocodone which can reduce gabapentin levels.

One side effect associated with pramipexole that is less likely to occur with other dopamine agonists is

sleep attacks.

What can influence the effectiveness of the cimetidine (Tagamet)

smoking should be taken 1 to 2 hours before antacids cimetidine and famotidine should be administered simultaneously with antacids. These drugs may be spaced 1 hr apart if both drugs need to be given. should be taken with meals.

A 58-year-old man has had a myocardial infarction (MI), has begun rehabilitation, and is ready for discharge. He is given a prescription for metoprolol (Lopressor), and he becomes upset. "I don't have high blood pressure-why did my doctor give me this medicine?" The nurse explains to him that Choose one answer.

studies have shown that this medication has greatly increased survival rates in patients after an MI.

Entacapone normal expectation of this med?

this is a COMT inhibitor indicated for the adjunctive treatment of Parkinsons disease. Entacapone is normally taken with levodopa. Entacapone benefits patients who are experiencing wearing - off effects. Adverse effects are GI upset, dyskinesias, and urine discoloration. Caution should be used with patients with preexisting liver disease.

What is a narrow therapeutic level?

this range is within which the drug is effective, and above that range it is rapidly toxic.

Why would pramoxine be added to an otic medication?

to help ease pain and itching common in ear infections.

Why would a patient be administered an anticholinergic medication pre - operative?

to reduce secretions

Patient education about sumatriptan (Imitrex)

treatment for migraine headaches, it is given upon the onset of headache. This drug is not a preventive migraine therapy Contraindications: drug allergy and the presence of cardiovascular disease, uncontrolled hypertension; cerebral, cardiac, or peripheral vascular disease; dysrhythmias; glaucoma; and coronary or ischemic heart disease.

Why would a patient receive medication through the parenteral route?

used for induction and / or maintenance of general anesthesia, inducion of amnesia, and as adjuncts to inhalation type anesthetics.

83. A patient has been taking methadone (Dolophine) for 5 months to overcome an opioid addiction. The nurse should monitor the patient for which of the following electrocardiographic changes? a. Prolonged QT interval b. Prolonged P-R interval c. AV block d. An elevated QRS complex

a. Prolonged QT interval Methadone prolongs the QT interval. It does not prolong the P-R interval, cause AV block, or produce an elevated QRS complex.

20. A patient complains of chronic insomnia and reports being tired of being tired all the time. The patient is reluctant to try pharmacologic remedies and asks the nurse what to do. What will the nurse suggest? a. "Eat a large meal in the evening to induce drowsiness." b. "Get out of bed for a while if you can't fall asleep." c. "Have a glass of wine at bedtime to relax." d. "Take a short nap early in the afternoon every day."

b. "Get out of bed for a while if you can't fall asleep."

34. A patient is diagnosed with major depression with severe symptoms and begins taking an antidepressant medication. Three weeks after beginning therapy, the patient tells the nurse that the drug is not working. The nurse will counsel this patient to ask the provider about: a. adding a second medication to complement this drug. b. changing the medication to one in a different drug class. c. increasing the dose of this medication. d. using nondrug therapies to augment the medication.

d. using nondrug therapies to augment the medication. Patients with severe depression benefit more from a combination of drug therapy and psychotherapy than from either component alone, so this patient should ask the provider about nondrug therapies. Once a drug has been selected for treatment, it must be used for 4 to 8 weeks before its efficacy can be assessed. Until a drug has been used at least 1 month without success, it should not be considered a failure. Adding a second medication, changing to a different medication, and increasing the dose of this medication should all be reserved until the current drug is deemed to have failed after at least 4 weeks.

54. A nurse provides teaching for a patient about to begin taking a first-generation antipsychotic drug for schizophrenia. Which statement by the patient indicates a need for further teaching about side effects of these drugs? a. "Anticholinergic effects are uncommon with this medication." b. "I may experience gynecomastia and galactorrhea." c. "I may feel lightheaded or dizzy and should sit or lie down if this occurs." d. "Sedation may occur initially, but will subside in 1 to 2 weeks."

a. "Anticholinergic effects are uncommon with this medication." Anticholinergic effects are common with FGAs, so this statement indicates a need for further teaching. Neuroendocrine effects, orthostatic hypertension, and sedation can occur with FGAs.

8. A nurse is discussing the use of benzodiazepines as sedative-hypnotic agents with a group of nursing students. A student asks about the actions of these drugs in the central nervous system. The nurse makes which correct statement? a. "Benzodiazepines affect the hippocampus and the cerebral cortex to cause anterograde amnesia." b. "Benzodiazepines depress neuronal functions by acting at a single site in the brain." c. "Benzodiazepines induce muscle relaxation by acting on sites outside the central nervous system." d. "Benzodiazepines promote sleep through effects on the limbic system."

a. "Benzodiazepines affect the hippocampus and the cerebral cortex to cause anterograde amnesia." All beneficial and most adverse effects of benzodiazepines occur from depressant actions in the central nervous system (CNS); the various effects depend on the site of action. Anterograde amnesia is the result of effects in the hippocampus and the cerebral cortex. Benzodiazepines act at multiple sites in the CNS. Muscle relaxant effects are the result of actions on supraspinal motor areas in the CNS. Benzodiazepines promote sleep through effects on cortical areas and on the sleep-wakefulness "clock."

62. A parent reports being afraid that a child may have schizophrenia because of disorganized speech and asocial behaviors. The nurse will tell this parent that which of the following must also be present to make a diagnosis? (Select all that apply.) a. A decrease in self-care, job, or school function b. A history of substance abuse c. A 1-month duration of active phase symptoms d. Continuous signs of disturbance for longer than 6 months e. The presence of manic episodes

a. A decrease in self-care, job, or school function c. A 1-month duration of active phase symptoms d. Continuous signs of disturbance for longer than 6 months Patients must have at least two symptoms with 1-month duration of active symptoms. One symptom must be delusions, hallucinations, or disordered speech. Patients must have continuous signs of disturbance for longer than 6 months. A history of substance abuse and manic episodes are not associated with schizophrenia.

24. The spouse of a patient with bipolar disorder (BPD) tells the nurse that the patient will not stay on the lithium ordered by the provider longer than 1 or 2 months at a time. The nurse understands that adherence to medication regimens in patients with BPD is problematic and will tell the spouse that: a. "During manic episodes, many patients don't see the benefit of prophylactic medications." b. "Increased gastrointestinal side effects occur over time and reduce compliance." c. "Long-term use of lithium causes memory impairment, causing patients to forget to take their medications." d. "Patients who are depressed do not want to take their medications."

a. "During manic episodes, many patients don't see the benefit of prophylactic medications." Patients experiencing manic symptoms often do not see anything wrong with their thinking and behavior and therefore do not believe they need treatment. Moreover, these symptoms are often enjoyable, and they do not want them to stop. Gastrointestinal side effects and central nervous system (CNS) effects of memory impairment subside over time and would diminish with long-term treatment. Patients are usually most uncomfortable during depressive episodes, and it is during these episodes that they often seek treatment.

26. A patient recently was diagnosed with bipolar disorder. The patient, who has a history of seasonal allergies, is an athlete who participates in track. The nurse is teaching the patient about lithium (Lithobid), which the prescriber has just ordered. Which statement by the patient indicates the need for further teaching? a. "I can continue to use ibuprofen as needed for muscle pain." b. "I should drink extra fluids before and during exercise." c. "I should not use antihistamines while taking lithium." d. "I should report muscle weakness and tremors to my provider."

a. "I can continue to use ibuprofen as needed for muscle pain." Because nonsteroidal anti-inflammatory drugs (NSAIDs) can increase lithium levels as much as 60%, they should not be used by patients taking lithium. Aspirin does not have this effect. Lithium induces polyuria in 50% to 70% of patients, so patients should be advised to drink extra fluids, especially during exercise. Antihistamines have anticholinergic effects, which cause urinary hesitancy; this can be uncomfortable when patients experience the polyuria associated with lithium use. Muscle weakness and tremors can occur with lithium; tremors can be treated with beta blockers or by altering the lithium regimen.

98. A nurse is teaching a patient who has a second-degree burn on one arm about the use of a topical anesthetic for pain. Which statement by the patient indicates understanding of the teaching? a. "I will apply a thin layer of the medication to a small area of skin." b. "I will cover the burn with a dressing after applying the medication." c. "I will make sure to apply the medication to the entire burn area." d. "I will use the medication only on the most painful, blistered areas."

a. "I will apply a thin layer of the medication to a small area of skin." Topical anesthetics can be absorbed in sufficient amounts to cause serious and even life-threatening systemic toxicity, so they should be applied in the smallest amount needed to as small an area as possible. Covering the site increases the skin's temperature, which increases absorption, so this should be avoided. Applying the medication to a large area increases systemic absorption. Applying the medication to broken skin increases systemic absorption.

17. A patient with a new-onset seizure disorder receives a prescription for phenobarbital. The patient reports being concerned about the sedative side effects of this drug. Which response by the nurse is correct? a. "Phenobarbital doses for seizures are nonsedating." b. "This is a short-acting barbiturate, so sedation wears off quickly." c. "Tolerance to the sedative effects will develop in a few weeks." d. "You may actually experience paradoxical effects of euphoria."

a. "Phenobarbital doses for seizures are nonsedating." Phenobarbital and mephobarbital are used for seizure disorders and suppress seizures at doses that are nonsedative. Phenobarbital is a long-acting barbiturate. At therapeutic doses, sedative effects do not occur. Paradoxical drug effects are associated with benzodiazepines and in older adults and debilitated patients with barbiturates.

25. A nurse is preparing to administer medications to a hospitalized patient who has been taking lithium (Lithobid) for 3 days. The patient is complaining of mild nausea and abdominal bloating. The patient's lithium level is 0.8 mEq/L. What will the nurse do? a. Administer the dose and tell the patient that the side effects are temporary. b. Contact the prescriber to request an order for serum electrolytes. c. Hold the dose and notify the prescriber of the patient's lithium level. d. Request an order for amiloride (Midamor).

a. Administer the dose and tell the patient that the side effects are temporary. This patient is experiencing side effects that are common and that occur at therapeutic levels of the drug. The lithium level is therapeutic and not toxic, so the nurse should give the dose and reassure the patient that the side effects will diminish over time. In the presence of low sodium, lithium can accumulate to toxic doses; therefore, if the lithium level were elevated, evaluating serum electrolytes would be advisable. The dose does not need to be withheld, because the patient does not have toxic levels of lithium. Amiloride is used if patients are experiencing lithium-induced polyuria, which this patient does not have.

60. Which side effects are more common in second-generation antipsychotic medications than in first-generation antipsychotic medications? (Select all that apply.) a. Agranulocytosis b. Anticholinergic effects c. Extrapyramidal symptoms d. Metabolism by CYP3A4 e. Prolactin elevation

a. Agranulocytosis b. Anticholinergic effects d. Metabolism by CYP3A4 SGAs are more likely than FGAs to cause agranulocytosis and anticholinergic effects and are metabolized by CYP3A4 enzymes. They are not more likely to cause extrapyramidal effects or prolactin elevation.

5. A patient describes feelings of anxiety and fear when speaking in front of an audience and is having difficulty at work because of an inability to present information at meetings three or four times each year. The patient is reluctant to take long-term medications. The nurse will expect the provider to order which treatment? a. Alprazolam (Xanax) as needed b. Cognitive behavioral therapy c. Paroxetine (Paxil) d. Psychotherapy

a. Alprazolam (Xanax) as needed This patient is describing social anxiety disorder; the symptoms are related to performance only and are not generalized to all social situations. Because this patient must speak in front of an audience only three or four times per year, a PRN medication can be used. Cognitive behavioral therapy is used for OCD. Paroxetine must be used continuously for at least 1 year. Psychotherapy can be used but is more effective when used in combination with drugs

46. A first-time parent of a 6-week-old infant tells the nurse that she feels anxious and sad, cries a lot, and has difficulty sleeping. What will the nurse do? a. Ask her to complete the Edinburgh Postnatal Depression Scale. b. Reassure her that the "baby blues" are normal after childbirth. c. Suggest that she get more sleep and try to increase her activity. d. Suggest that she talk to her provider about using an SSRI.

a. Ask her to complete the Edinburgh Postnatal Depression Scale. Women who experience depressive symptoms after childbirth can be screened using a quick test: the Edinburgh Postnatal Depression Scale, which may be administered 6 to 8 weeks after delivery. Reassuring her that her feelings are normal does not acknowledge her concerns; also, without objective screening, this reassurance may not be warranted. Increased sleep and activity may help, but screening is necessary to determine the severity of the depression. Until the patient is diagnosed as having postpartum depression, an SSRI is not indicated.

12. A patient takes temazepam (Restoril) for insomnia. The patient tells the nurse that a recent telephone bill lists several calls to friends that the patient does not remember making. What will the nurse do? a. Ask the patient about any alcohol consumption in conjunction with the benzodiazepine. b. Contact the prescriber to request an order for a benzodiazepine with a shorter duration. c. Reassure the patient that this is most likely caused by a paradoxical reaction to the benzodiazepine. d. Tell the patient that this is an example of anterograde amnesia, which is an expected effect of benzodiazepine

a. Ask the patient about any alcohol consumption in conjunction with the benzodiazepine. This patient is describing complex sleep-related behavior, which occurs when patients carry out complex behaviors while taking benzodiazepines but have no memory of their actions. These actions can occur with normal doses but are more likely with excessive doses or when benzodiazepines are combined with alcohol or other CNS depressants, so the nurse is correct in evaluating this possibility. The duration of the benzodiazepine does not contribute to this phenomenon. Paradoxical effects of benzodiazepines include insomnia, excitation, euphoria, anxiety, and rage. Anterograde amnesia occurs when patients have impaired recall of events that occur after dosing.

82. A nurse is preparing a patient for surgery and is teaching the patient about the use of the patient-controlled analgesia pump. The patient voices concern about becoming addicted to morphine. What will the nurse do? a. Ask the patient about any previous drug or alcohol abuse. b. Discuss possible nonopioid options for postoperative pain control. c. Suggest that the patient use the PCA sparingly. d. Tell the patient that the pump can be programmed for PRN dosing only.

a. Ask the patient about any previous drug or alcohol abuse. The nurse should remember that addiction to opioids usually occurs in patients who already have tendencies for addiction, so an assessment of previous experiences with addictive substances would be indicated. Postoperative pain should be treated appropriately with medications that are effective. Nonopioid medications are not sufficient to treat postoperative pain. Patients should be encouraged to use the PCA as needed so that pain can be controlled in a timely fashion. PRN dosing is not as effective as dosing that is continuous, so a basal dose should be given as well as a PRN dose.

81. A patient will receive buprenorphine (Butrans) as a transdermal patch for pain. What is important to teach this patient about the use of this drug? a. Avoid prolonged exposure to the sun. b. Cleanse the site with soap or alcohol. c. Remove the patch daily at bedtime. d. Remove hair by shaving before applying the patch.

a. Avoid prolonged exposure to the sun. Patients using the buprenorphine transdermal patch should be cautioned against heat, heating pads, hot baths, saunas, and prolonged sun exposure. The skin should be cleaned with water only. The patch should stay on for 7 days before a new patch is applied. Patients should remove hair by clipping, not shaving.

95. A postoperative patient complains of abdominal bloating and discomfort. The nurse caring for this patient will contact the provider to request which medication? a. Bethanechol b. Droperidol c. Promethazine d. Ondansetron

a. Bethanechol Bethanechol is a muscarinic agonist that is used to treat abdominal distention and urinary retention. The other three agents are antiemetics and would not be useful in this situation.

45. A patient who has been taking an SSRI tells the nurse that the drug has caused reduced sexual performance, weight gain, and sedation. The nurse will suggest that the patient ask the provider about using which drug? a. Bupropion (Wellbutrin) b. Imipramine (Tofranil) c. Isocarboxazid (Marplan) d. Trazodone (Oleptro)

a. Bupropion (Wellbutrin) Bupropion does not cause weight gain, sexual dysfunction, or sedation, so it may be a useful adjunct to or substitute for an SSRI when those side effects become intolerable. Imipramine causes sedation. Isocarboxazid is an MAOI and is not used unless other drugs are ineffective. Trazodone causes sedation.

47. A patient is brought to the emergency department after taking a handful of TCA pills. The nurse will expect to provide what when caring for this patient? (Select all that apply.) a. Cardiac monitoring b. Cholinesterase inhibitors c. Gastric lavage and activated charcoal d. Sedative medications e. Procainamide

a. Cardiac monitoring b. Cholinesterase inhibitors c. Gastric lavage and activated charcoal Patients who overdose with a TCA should have cardiac monitoring, because cardiac side effects can occur. Cholinesterase inhibitors are given to counteract anticholinergic side effects. Gastric lavage followed by activated charcoal can reduce absorption of the TCA. Sedative drugs would only increase the sedative effects of the TCA. Procainamide causes cardiac depression and is not recommended to treat TCA dysrhythmias.

71. Supplemental oxygen has been shown to help reduce symptoms for which type of headache? a. Cluster b. Menstrual migraine c. Migraine d. Tension-type

a. Cluster Cluster headaches can be treated with 100% oxygen inhalation. Oxygen therapy is not used to treat other types of headaches.

84. A nurse is administering morphine sulfate to a postoperative patient. Which are appropriate routine nursing actions when giving this drug? (Select all that apply.) a. Counting respirations before and after giving the medication b. Encouraging physical activity and offering increased fluids c. Monitoring the patient's blood pressure closely for hypertension d. Palpating the patient's lower abdomen every 4 to 6 hours e. Requesting an order for methylnaltrexone (Relistor) to prevent constipation

a. Counting respirations before and after giving the medication b. Encouraging physical activity and offering increased fluids d. Palpating the patient's lower abdomen every 4 to 6 hours Respiratory depression, constipation, and urinary retention are common adverse effects of opioid analgesics. It is important to count respirations before giving the drug and periodically thereafter to make sure that respiratory depression has not occurred. Increased physical activity, increased fluid intake, and increased fiber help alleviate constipation. It is important to assess the patient's abdomen and palpate the bladder to make sure that urinary retention has not occurred. Patients taking morphine often experience hypotension, not hypertension. Methylnaltrexone is given as a last resort to treat constipation, because it blocks mu receptors in the intestine.

88. A surgical patient is receiving succinylcholine (Anectine) with an inhalation anesthetic. The patient is intubated, has an indwelling urinary catheter, and has ongoing monitoring of vital signs. Which symptom during the perioperative period is cause for concern? a. Elevated temperature b. Increased urine output c. Muscle paralysis d. No response to painful stimuli

a. Elevated temperature Combining succinylcholine (a skeletal muscle relaxant) with an inhalation anesthetic increases the risk of malignant hyperthermia. The mechanism is not understood. Temperature elevation can be profound, and cooling measures must be initiated or the condition can be fatal. A decrease in urine output would be a sign of hypotension. Muscle paralysis and lack of response to pain are desired effects of anesthesia.

22. Which sedative-hypnotics are useful for both difficulty falling asleep and difficulty maintaining sleep? (Select all that apply.) a. Eszopiclone (Lunesta) b. Flurazepam c. Temazepam (Restoril) d. Triazolam (Halcion) e. Extended-release zolpidem (Ambien CR)

a. Eszopiclone (Lunesta) b. Flurazepam e. Extended-release zolpidem (Ambien CR) Eszopiclone (Lunesta), flurazepam, and extended-release zolpidem (Ambien CR) all help with difficulty falling asleep and difficulty maintaining sleep. Temazepam is useful for maintaining sleep only. Triazolam helps with difficulty falling asleep but does not maintain sleep.

73. A patient asks the nurse what can be given to alleviate severe, chronic pain of several months' duration. The patient has been taking oxycodone (OxyContin) and states that it is no longer effective. The nurse will suggest discussing which medication with the provider? a. Fentanyl (Duragesic) transdermal patch b. Hydrocodone (Vicodin) PO c. Meperidine (Demerol) PO d. Pentazocine (Talwin) PO

a. Fentanyl (Duragesic) transdermal patch Transdermal fentanyl is indicated only for persistent, severe pain in patients already opioid tolerant. Hydrocodone, a combination product, has actions similar to codeine and is not used for severe, chronic pain. Meperidine is not recommended for continued use because of the risk of harm caused by the accumulation of a toxic metabolite. Pentazocine is an agonist-antagonist opioid and is less effective for pain; moreover, when given to a patient who is already opioid tolerant, it can precipitate an acute withdrawal syndrome.

2. A nurse is admitting a patient to a hospital unit and is taking a history. The patient reports taking alprazolam (Xanax) for "nerves." The nurse knows that this patient is most likely being treated for which condition? a. Generalized anxiety disorder b. Obsessive-compulsive disorder (OCD) c. Panic disorder d. Post-traumatic stress disorder (PTSD)

a. Generalized anxiety disorder Benzodiazepines are first-choice drugs for anxiety, and alprazolam and lorazepam are prescribed most often. Selective serotonin reuptake inhibitors (SSRIs) are the first-line drugs for the treatment of OCD. Panic disorder is treated with any of the three classes of antidepressants: SSRIs, tricyclic antidepressants (TCAs), and monoamine oxidase inhibitors (MAOIs). Research has not shown any drug to be effective in the treatment of PTSD, although two SSRIs have been approved for use for this disorder.

7. Selective serotonin reuptake inhibitors are known to be effective for which disorders? (Select all that apply.) a. Generalized anxiety disorder (GAD) b. Obsessive-compulsive disorder c. Panic disorder d. Post-traumatic stress disorder e. Social anxiety disorder

a. Generalized anxiety disorder (GAD) b. Obsessive-compulsive disorder c. Panic disorder e. Social anxiety disorder SSRIs have been shown to be effective in treating GAD, OCD, panic disorder, and social anxiety disorder. They are used to treat PTSD but have not demonstrated effectiveness in clinical research.

6. During an admission history, a patient reports a frequent need to return to a room multiple times to make sure an iron or other appliance is unplugged. What does the nurse understand about this patient's behavior? a. It helps the patient reduce anxiety about causing a fire. b. It usually is treated with clomipramine (Anafranil). c. It seems perfectly normal to the patient. d. It will best respond to deep brain stimulation.

a. It helps the patient reduce anxiety about causing a fire. Patients with OCD have compulsive behaviors, such as repeatedly checking to make sure appliances have been unplugged. The compulsion is a ritualized behavior resulting from obsessive anxiety or fear that something bad will happen, such as starting a fire with an overheated appliance. Clomipramine is not a first-line drug for treating OCD. Patients usually understand that compulsive behaviors are excessive and senseless but are unable to stop. Deep brain stimulation is indicated for patients in whom other treatments have failed; its effectiveness at reducing symptoms has been shown to be about 40%.

97. A patient is given nitrous oxide, along with another inhalation anesthetic. The nurse knows that the benefits of nitrous oxide include what? (Select all that apply.) a. It is a potent analgesic. b. It has high anesthetic potency. c. It is less likely to cause nausea and vomiting. d. It is less likely to precipitate malignant hyperthermia. e. It can significantly reduce the dose of inhalation anesthetic.

a. It is a potent analgesic. d. It is less likely to precipitate malignant hyperthermia. e. It can significantly reduce the dose of inhalation anesthetic. Nitrous oxide is an inhalation agent with high analgesic potency. It is not likely to precipitate malignant hyperthermia. It can significantly reduce the dose of the primary anesthetic by as much as 50% or more. It does not have high anesthetic potency, so it cannot be used alone to induce anesthesia. It is more likely to cause postoperative nausea and vomiting.

78. A postoperative patient has received an epidural infusion of morphine sulfate. The patient's respiratory rate decreases to 8 breaths per minute, and he has a decreased level of consciousness and miosis. Which medication would the nurse anticipate administering? a. Naloxone (Narcan) b. Acetylcysteine (Mucomyst) c. Methylprednisolone (Medrol) d. Physostigmine (Antilirium)

a. Naloxone (Narcan) Narcan is an opioid antagonist and would counteract the apparent toxicity. Acetylcysteine is the antidote for acetaminophen poisoning. Methylprednisolone is an anti-inflammatory agent and would not have an effect on toxicity related to opiate overdose. Physostigmine is indicated for organophosphate poisoning.

48. Which patients are candidates for MAOIs? (Select all that apply.) a. Patients who have not responded to SSRIs and TCAs b. Patients with atypical depression c. Patients with bulimia nervosa d. Patients with hypotension e. Patients with postpartum depression

a. Patients who have not responded to SSRIs and TCAs b. Patients with atypical depression c. Patients with bulimia nervosa Patients who have not responded to SSRIs or TCAs, patients with atypical depression, and patients with bulimia nervosa are candidates for MAOIs. MAOIs contribute to hypotension and therefore are contraindicated in patients with hypotension. MAOIs are not recommended for the treatment of postpartum depression.

42. An older adult patient who is to begin taking imipramine (Tofranil) asks the nurse when the drug should be taken. The nurse will instruct the patient to: a. divide the daily dose into two equal doses 12 hours apart. b. take the entire dose at bedtime to minimize sedative effects. c. take the medication once daily in the late afternoon. d. take the medication once daily in the morning.

a. divide the daily dose into two equal doses 12 hours apart. For many patients, taking the entire dose of a TCA at bedtime is advantageous for facilitating adherence, minimizing daytime sedation, and promoting sleep. However, older adult patients are at greater risk for cardiotoxicity and may experience intolerable effects on the heart if the entire dose is taken at once; therefore, twice-daily dosing is recommended in the elderly.

39. A neonate is born to a patient who reports taking venlafaxine (Effexor XR). The nurse caring for the infant will observe the infant for: a. irritability, tremor, and respiratory distress. b. poor appetite and disturbed sleeping patterns. c. serotonin syndrome. d. sustained mydriasis.

a. irritability, tremor, and respiratory distress. Use of venlafaxine late in pregnancy can result in a neonatal withdrawal syndrome characterized by irritability, abnormal crying, tremor, respiratory distress, and possibly seizures. Poor appetite and disturbed sleep are not part of this withdrawal syndrome. Serotonin syndrome is not likely. Sustained mydriasis occurs as an adverse effect in patients taking the drug.

96. A patient will receive isoflurane (Forane) as an anesthetic for a surgical procedure. The nurse caring for this patient during the perioperative period knows that, unlike halothane, this agent will not cause: a. myocardial depression. b. muscle relaxation. c. rapid induction. d. respiratory depression.

a. myocardial depression. Isoflurane does not cause myocardial depression and does not reduce cardiac output. Isoflurane actually produces more muscle relaxation than halothane. Induction with isoflurane is rapid. Isoflurane causes respiratory depression, as do all inhalation anesthetics.

52. A nurse and a nursing student are discussing the plan of care for a patient with schizophrenia. The patient, who has been taking a high-potency FGA for 2 months, has become restless and constantly needs to be in motion. Which statement by the student indicates a need for further education? a. "Anticholinergic medications may help control these symptoms." b. "Because this may be an exacerbation of psychosis, the provider may increase the dose of the FGA." c. "The provider may try a low-potency FGA instead of the high-potency FGA." d. "This patient may need to take a benzodiazepine or a beta blocker."

b. "Because this may be an exacerbation of psychosis, the provider may increase the dose of the FGA." The patient is showing signs of akathisia, which can resemble an exacerbation of psychosis. If the two are confused and the provider orders more of the FGA, the symptoms may actually increase. Anticholinergic medications may be used, a low-potency FGA may be ordered, or a benzodiazepine or beta blocker may be prescribed.

30. A patient with bipolar disorder is admitted to the hospital. The patient has been taking lithium (Lithobid) for several years and has not been evaluated by a provider for over a year. Besides obtaining a lithium level, the nurse caring for this patient will anticipate orders for which laboratory tests? (Select all that apply.) a. Calcium level b. Complete blood count with differential c. Liver function tests d. Renal function tests e. Serum potassium f. Thyroid function tests

b. Complete blood count with differential d. Renal function tests f. Thyroid function tests Patients taking lithium can develop a mild, reversible leukocytosis, so annual CBC evaluation with differential is recommended. Chronic lithium use is associated with degenerative changes in the kidneys, so renal function should be assessed annually. Lithium can reduce the incorporation of iodine into thyroid hormone and can inhibit thyroid secretion; therefore, thyroid hormone and thyroid-stimulating hormone (TSH) levels should be measured annually. Lithium is affected by sodium levels but not by calcium or potassium levels. Because lithium is excreted by the kidneys, hepatic function tests are not indicated.

27. A patient with bipolar disorder who is taking divalproex sodium (Valproate) has just been admitted to the hospital. During the admission assessment, the patient tells the nurse about recent suicidal ideation. The nurse observes several areas of bruising over soft tissue areas and notes a weight gain of 10 pounds since the last admission 1 year ago. What will the nurse do? a. Ask the patient whether the bruises are self-inflicted. b. Contact the provider to report these findings. c. Give the patient information about weight loss. d. Request an order for an increased dose to help with depressive symptoms.

b. Contact the provider to report these findings. Divalproex sodium is used to control symptoms during manic episodes and can prevent relapse into mania. It is less effective than lithium at reducing the risk of suicide. It can cause thrombocytopenia, which results in bruising and is an indication for immediate drug withdrawal. Weight gain can be serious and chronic. All of these findings are an indication for withdrawing the drug and should be reported to the provider. Until platelet levels determine whether the bruises are drug induced, it is not appropriate to ask the patient if they are self-inflicted. Because weight gain is common and can be severe with this drug, information about weight loss is not likely to have an effect. Divalproex sodium is better than lithium at treating depression; however, in light of the other symptoms, it is probably not the best choice.

85. Which side effects of opioid analgesics can have therapeutic benefits? (Select all that apply.) a. Biliary colic b. Cough suppression c. Suppression of bowel motility d. Urinary retention e. Vasodilation

b. Cough suppression c. Suppression of bowel motility e. Vasodilation Individual effects of morphine may be beneficial, detrimental, or both. Cough suppression is usually beneficial; suppression of bowel motility and vasodilation can be either beneficial or detrimental. Biliary colic and urinary retention are always detrimental side effects.

36. A patient has been taking fluoxetine (Prozac) for 11 months and reports feeling cured of depression. The nurse learns that the patient is sleeping well, participates in usual activities, and feels upbeat and energetic most of the time. The patient's weight has returned to normal. What will the nurse tell this patient? a. Indefinite drug therapy is necessary to maintain remission. b. Discuss gradual withdrawal of the medication with the provider. c. Stop the drug while remaining alert for the return of symptoms. d. Take a drug holiday to see whether symptoms recur.

b. Discuss gradual withdrawal of the medication with the provider. Abrupt discontinuation of selective serotonin reuptake inhibitors (SSRIs) can cause a withdrawal syndrome, so these drugs should be withdrawn gradually. After symptoms are in remission, treatment should continue for 4 to 9 months to prevent relapse; these drugs are not taken indefinitely. It is incorrect to counsel the patient to stop taking the drug without tapering it. Drug holidays are used to minimize sexual dysfunction side effects, not to monitor symptoms

40. A provider has indicated that a serotonin/norepinephrine reuptake inhibitor (SNRI) will be prescribed for a patient who is experiencing major depression. When conducting a pretreatment health history, the nurse learns that the patient has a recent history of alcohol abuse. Which SNRI would be contraindicated for this patient? a. Desvenlafaxine (Pristiq) b. Duloxetine (Cymbalta) c. Escitalopram (Lexapro) d. Venlafaxine (Effexor SR)

b. Duloxetine (Cymbalta) Duloxetine can cause elevation of serum transaminases, indicating liver disease. Patients with existing liver disease or alcohol abuse should not receive this drug. This side effect is not likely with desvenlafaxine or venlafaxine. Escitalopram is an SSRI.

68. A patient is being treated for an infection with erythromycin. The nurse obtains a health history and learns that the patient has migraine headaches. The nurse will tell the patient to avoid which medication while taking the erythromycin? a. Acetaminophen b. Ergotamine (Ergomar) c. Sumatriptan (Imitrex) d. Topiramate

b. Ergotamine (Ergomar) Macrolide antibiotics, such as erythromycin, are potent inhibitors of CYP3A4; these drugs can raise ergotamine to dangerous levels, so this combination should be avoided. It is safe to take acetaminophen, sumatriptan, and topiramate with erythromycin.

50. A patient who is taking an antipsychotic drug for schizophrenia comes to the clinic for evaluation. The nurse observes that the patient has a shuffling gait and tremors and is drooling. The nurse will ask the patient's provider about which course of action? a. Administering a direct dopamine antagonist b. Giving an anticholinergic medication c. Increasing the dose of the antipsychotic drug d. Stopping the antipsychotic drug

b. Giving an anticholinergic medication The patient is showing signs of parkinsonism, an extrapyramidal effect associated with antipsychotic medications. Anticholinergic medications are indicated. A direct dopamine antagonist would counter the effects of the antipsychotic and remove any beneficial effect it has. Increasing the dose of the antipsychotic medication would only worsen the extrapyramidal symptoms. Stopping the antipsychotic medication would cause the symptoms of schizophrenia to worsen.

72. A patient on the unit complains of cluster headaches. A new graduate nurse is asked to differentiate between a migraine headache and cluster headaches. The graduate nurse is correct to state that manifestations and/or risk factors for a patient with cluster headaches include what? (Select all that apply.) a. Female gender b. Male gender c. Complaints of nausea and vomiting d. Short duration (15 minutes to 2 hours) e. Auras before the onset of headache pain f. Throbbing, sometimes piercing pain

b. Male gender d. Short duration (15 minutes to 2 hours) f. Throbbing, sometimes piercing pain Cluster headaches are more common in males, are short in duration, and present as throbbing and piercing pain. Migraine headaches are more common in females and are manifested by nausea and vomiting and the presence of an aura before the onset of headache pain.

21. A nurse recognizes that the actions of benzodiazepines include which findings? (Select all that apply.) a. Sleep deprivation b. Relief of general anxiety c. Suppression of seizures and/or seizure activity d. Development of tardive dyskinesia e. Increase in muscle spasms

b. Relief of general anxiety c. Suppression of seizures and/or seizure activity Benzodiazepines are indicated to relieve the symptoms of general anxiety and to suppress the central nervous system, thereby suppressing seizures and/or seizure activity. Benzodiazepines cause sleepiness, not sleep deprivation. Benzodiazepines do not cause tardive dyskinesia. Benzodiazepines relax muscles; they do not increase muscle spasms.

58. A patient in whom drug therapy has failed several times in the past is readmitted to a hospital to begin therapy for schizophrenia. What will the nurse do to help improve adherence? a. Encourage the patient to take responsibility for medication management. b. Teach the patient about drug side effects and how to manage them. c. Tell the patient that an abstinence syndrome will occur if the drug is stopped. d. Tell the patient that the drug may be taken as needed to control symptoms.

b. Teach the patient about drug side effects and how to manage them. One way to promote adherence to a medication regimen is to teach patients about drug side effects and how to minimize undesired responses. Family members should be encouraged to oversee medication management for outpatients, because patients themselves may fail to appreciate the need for therapy or may be unwilling to take prescribed medications. It is not true that an abstinence syndrome occurs when these drugs are withdrawn. These drugs are not used PRN; they must be given on a regular basis.

92. A preoperative patient receives atropine before induction of anesthesia. The nurse caring for this patient understands that this agent is used to prevent: a. anxiety. b. bradycardia. c. dry mouth. d. hypertension.

b. bradycardia. Atropine, an anticholinergic drug, is used as an adjunct to anesthesia to counter the effects of vagal stimulation, which is caused by surgical manipulations that trigger parasympathetic reflexes, resulting in bradycardia. Atropine is not an anxiolytic. Atropine causes dry mouth and sometimes is used to minimize bronchial secretions

75. A patient with moderate to severe chronic pain has been taking oxycodone (OxyContin) 60 mg every 6 hours PRN for several months and tells the nurse that the medication is not as effective as before. The patient asks if something stronger can be taken. The nurse will contact the provider to discuss: a. administering a combination opioid analgesic/acetaminophen preparation. b. changing the medication to a continued-release preparation. c. confronting the patient about drug-seeking behaviors. d. withdrawing the medication, because physical dependence has occurred.

b. changing the medication to a continued-release preparation. Oxycodone is useful for moderate to severe pain, and a continued-release preparation may give more continuous relief. Dosing is every 12 hours, not PRN. A combination product is not recommended with increasing pain, because the nonopioid portion of the medication cannot be increased indefinitely. This patient does not demonstrate drug-seeking behaviors. Physical dependence is not an indication for withdrawing an opioid, as long as it is still needed; it indicates a need for withdrawing the drug slowly when the drug is discontinued.

67. A patient arrives in the emergency department complaining of numbness in the extremities. The nurse notes that the patient's hands and feet are cool and pale. When conducting a health history, the nurse learns that the patient has a history of migraine headaches. The nurse recognizes this patient's symptoms as: a. ergotamine withdrawal. b. ergotism. c. severe migraine symptoms. d. sumatriptan side effects.

b. ergotism. Ergotism is a serious toxicity caused by acute or chronic overdose of ergotamine. The toxicity results in ischemia, causing the extremities to become cold, pale, and numb. Symptoms associated with ergotamine withdrawal include headache, nausea, vomiting, and restlessness. These are not symptoms of a severe migraine or side effects of sumatriptan.

23. A patient with bipolar disorder has frequent manic episodes alternating with depressive episodes. The prescriber orders risperidone (Risperdal) in addition to the lithium (Lithobid) that the patient is already taking. The patient asks the nurse why another drug is needed. The nurse will tell the patient that the risperidone is used to: a. elevate mood during depressive episodes. b. help control symptoms during manic episodes. c. manage tremors associated with lithium use. d. prevent recurrence of depressive episodes.

b. help control symptoms during manic episodes. Risperidone is an antipsychotic often used in conjunction with lithium to help manage symptoms during manic episodes, regardless of whether psychotic symptoms occur. Risperidone does not elevate mood and is not used during depressive episodes. It is not used to counter side effects associated with lithium. It does not prevent recurrence of depressive episodes.

64. A woman with moderate migraine headaches asks a nurse why the provider has ordered metoclopramide (Reglan) as an adjunct to aspirin therapy, because she does not usually experience nausea and vomiting with her migraines. The nurse will tell her that the metoclopramide is used to: a. help induce sleep. b. improve absorption of the aspirin. c. prevent gastric irritation caused by the aspirin. d. prolong the effects of the aspirin.

b. improve absorption of the aspirin. Besides reducing nausea and vomiting, metoclopramide also reverses gastric stasis and improves absorption of oral antimigraine drugs. It is not used to induce sleep. It does not prevent gastric irritation or prolong the effects of the aspirin.

19. A nurse is obtaining a health history from an older adult patient in an outpatient clinic. The patient reports chronic difficulty falling asleep and staying asleep. The nurse knows that the best treatment for this patient will be: a. alternative medications. b. improved sleep hygiene. c. short-term barbiturates. d. triazolam (Halcion).

b. improved sleep hygiene. Research has shown that cognitive behavioral therapy is superior to drug therapy for both short-term and long-term management of chronic insomnia in older adults. Alternative remedies have not been proven effective. Barbiturates may elicit paradoxical effects in elderly patients. Triazolam does not help to maintain sleep.

11. A nursing student asks a nurse what criteria are used to determine which benzodiazepine is prescribed in different situations. The nurse correctly states that selection is based on differences in the onset and duration of effects, as well as on: a. differences in sites of action in the central nervous system. b. marketing decisions of pharmaceutical companies. c. relative differences in abuse potential. d. variations in adverse effects and drug interactions.

b. marketing decisions of pharmaceutical companies. The principal factors determining the applications of a particular benzodiazepine are the pharmacokinetic properties having to do with absorption, metabolism, and excretion and the research and marketing decisions of the drug makers. All of the benzodiazepines produce a similar spectrum of responses, and all act at various sites in the CNS. All benzodiazepines have a lower abuse potential than barbiturates. Drug effects and drug interactions are similar for all benzodiazepines.

29. A patient with bipolar disorder takes lamotrigine (Lamictal). Which statement by the patient would prompt the nurse to hold the drug and notify the prescriber for further assessment? a. "I get a little dizzy sometimes." b. "I had a headache last week that lasted for about an hour." c. "I've broken out in a rash on my chest and back." d. "Last night I woke up twice with a bad dream."

c. "I've broken out in a rash on my chest and back." Evidence of a rash in a patient taking lamotrigine requires further assessment, because this may indicate the development of Stevens-Johnson syndrome. Although dizziness and headaches are side effects of lamotrigine, they are not potentially life threatening. A bad dream is not necessarily related to the lamotrigine.

55. A patient with schizophrenia shows suicidal behaviors, and the provider orders clozapine (Clozaril). The nurse teaches the family about the medication and its side effects. Which statement by a family member indicates a need for further teaching about this drug? a. "Blood counts are necessary for several weeks after discontinuation of the drug." b. "Fever, sore throat, and sores in the mouth should be reported immediately." c. "If the ANC is less than 3000, the drug will be discontinued permanently." d. "Use of this drug requires weekly evaluation of blood work."

c. "If the ANC is less than 3000, the drug will be discontinued permanently." Clozapine can cause agranulocytosis. If the absolute neutrophil count (ANC) drops below 1000/mcL, the drug must be discontinued permanently. Blood counts must be evaluated weekly, and this evaluation should be continued for several weeks after withdrawal of the drug. Fever, sore throat, and mouth ulcers are symptoms of agranulocytosis and should be reported immediately.

90. An anesthesiologist completes preoperative teaching for a patient the night before surgery. The patient asks the nurse to clarify the reason thiopental sodium will be given as an adjunct to the anesthetic. Which statement by the patient indicates understanding? a. "Pentothal allows a larger amount of inhaled anesthetic to be used without increased side effects." b. "Pentothal is given to enhance the analgesic and muscle relaxation effects of the inhaled anesthetic." c. "Pentothal is used to produce rapid unconsciousness before administration of the inhaled anesthetic." d. "Pentothal is used to reduce cardiovascular and respiratory depression caused by the inhaled anesthetic.

c. "Pentothal is used to produce rapid unconsciousness before administration of the inhaled anesthetic." Thiopental sodium, a short-acting barbiturate, is given intravenously for induction of anesthesia. It has a rapid onset and short duration and must be followed immediately by an inhalation anesthetic. It does not alter the side effects of inhalation anesthetics. Most adjuncts to inhalation anesthetics are given so that lower doses of the anesthetic may be used. Thiopental sodium has very weak analgesic and muscle relaxation effects. Thiopental sodium causes cardiovascular and respiratory depression.

41. A nurse is teaching a group of nursing students about tricyclic antidepressants (TCAs). Which statement by a student indicates a need for further teaching? a. "TCAs block receptors for histamine, acetylcholine, and norepinephrine." b. "TCAs block the uptake of norepinephrine and 5-HT." c. "TCAs have many side effects, but none of them are serious." d. "TCAs have other uses than for depression."

c. "TCAs have many side effects, but none of them are serious." TCAs can adversely affect cardiac function, and these effects can be serious in patients with pre-existing cardiac impairment. TCAs do block receptors for histamine, acetylcholine, and norepinephrine; they do block the uptake of norepinephrine and 5-HT; and they do have other uses besides depression.

99. A nurse is discussing the use of cocaine as a local anesthetic with a nursing student. Which statement by the student indicates understanding of this agent? a. "Anesthetic effects develop slowly and persist for several hours." b. "Cocaine is a local anesthetic administered by injection." c. "Vasoconstrictors should not be used as adjunct agents with this drug." d. "When abused, cocaine causes physical dependence."

c. "Vasoconstrictors should not be used as adjunct agents with this drug." Cocaine should not be combined with epinephrine or other vasoconstrictors, because it causes vasoconstriction itself, and the combination could precipitate severe hypertension. Cocaine has a rapid onset of effects, which last about 1 hour. It is used only topically for anesthesia. Although subject to widespread abuse with profound psychologic dependence, it does not cause substantial physical dependence

56. A patient with schizophrenia receives a dose of risperidone (Risperdal Consta) IM. The nurse teaching this patient about this medication will make which statement? a. "You will experience therapeutic levels of this drug in 1 to 2 weeks." b. "You will need injections of this drug every 6 weeks." c. "You will need to take an oral antipsychotic drug for 3 weeks." d. "You probably will not have extrapyramidal symptoms with this drug."

c. "You will need to take an oral antipsychotic drug for 3 weeks." Risperidone given intramuscularly is a depot preparation used for long-term therapy. Significant release of the drug does not occur until 2 to 3 weeks after injection; therefore, patients must take an oral antipsychotic medication until drug levels are raised. Therapeutic levels are reached 4 to 6 weeks after injection. Patients need injections every 2 weeks. With IM dosing, the incidence of extrapyramidal symptoms is substantial.

61. What are negative symptoms of schizophrenia? (Select all that apply.) a. Delusions b. Disordered thinking c. Poor judgment d. Poor self-care e. Poverty of speech

c. Poor judgment d. Poor self-care e. Poverty of speech Poor judgment, poor self-care, and poverty of speech are all negative symptoms of schizophrenia. Delusions and disordered thinking are positive symptoms.

59. A nurse in a mental health hospital finds a patient with schizophrenia who takes haloperidol (Haldol) lying rigid in bed with a temperature of 41.3° C. A cardiac monitor shows cardiac dysrhythmias. What will be included in the treatment of this patient? (Select all that apply.) a. Anticholinergic medications b. Beta blockers c. Dantrolene d. Intravenous fluids e. Withdrawal of haloperidol

c. Dantrolene d. Intravenous fluids e. Withdrawal of haloperidol Neuroleptic malignant syndrome is characterized by "lead pipe" rigidity, sudden high fever, and autonomic instability. Treatment requires supportive measures, drug therapy, and immediate withdrawal of the antipsychotic medication. Dantrolene is used to relax muscles and reduce heat production. Intravenous fluids are used to maintain hydration. Anticholinergic medications and beta blockers are not helpful.

37. A patient has been taking an SSRI antidepressant for major depression and reports having headaches and jaw pain. What will the nurse tell the patient? a. This represents an irreversible extrapyramidal side effect. b. Discuss discontinuing the antidepressant with the provider. c. Discuss these symptoms with a dentist. d. Try stress-relieving methods and relaxation techniques.

c. Discuss these symptoms with a dentist. Bruxism is a side effect of SSRIs and can result in headache and jaw pain. Patients who experience these signs should be evaluated for bruxism by a dentist, who can determine whether the patient may benefit from use of a mouth guard. Headache and jaw pain are not signs of extrapyramidal side effects. Discontinuing the antidepressant is not indicated, because depression may return. Stress-relieving methods and relaxation techniques are not recommended, because these symptoms occur during sleep.

15. A patient is brought to the emergency department by friends, who say that they were at a party where alcohol and a mix of barbiturates and benzodiazepines were all available. They tell the nurse that the patient was among the first to arrive at the party, which started several hours ago. The patient is nonresponsive and has pinpoint pupils and respirations of 6 breaths per minute. After oxygen has been administered, the nurse should prepare the patient for which intervention? a. A central nervous system stimulant and IV fluids b. Activated charcoal and flumazenil (Romazicon) c. Gastric lavage and possible hemodialysis d. Naloxone (Narcan) and a cathartic

c. Gastric lavage and possible hemodialysis Because time has elapsed, enough medication is present in the system to warrant elimination by hemodialysis, and any remainder in the stomach may be eliminated by gastric lavage. A central nervous system stimulant is contraindicated, and intravenous fluids do not address the overdose. Although activated charcoal may assist in absorption of medication in the gut, flumazenil will be effective only for the benzodiazepines. Naloxone, a narcotic antagonist, is not effective for barbiturates and benzodiazepines

33. A patient whose spouse has died recently reports feeling down most of each day for the past 2 months. On further questioning, the nurse learns that the patient has quit participating in church and social activities, has difficulty falling asleep, and has lost 5 pounds. The patient reports feeling tired and confused all the time but does not have suicidal thoughts. What does the nurse suspect? a. Grief and sadness b. Hypomania c. Major depression d. Situational depression

c. Major depression This patient has symptoms of major depression, which include depressed mood, loss of pleasure in usual activities, insomnia, weight loss, and feelings of fatigue. For a diagnosis of major depression, these symptoms must be present most of the day, nearly every day, for at least 2 weeks. Grief and sadness and situational depression are common responses to the death of a loved one, but this patient's symptoms go beyond this normal response. This patient does not show signs of hypomania.

A postoperative patient is reporting pain as a 7 on a scale from 1 to 10, with 10 being the worst pain. The nurse caring for the patient assesses vital signs of HR, 76; RR, 16; and BP, 110/70. The patient has vomited twice. Which postoperative medications will the nurse expect to administer? a. Atropine and morphine b. Bethanechol and ibuprofen c. Morphine and ondansetron (Zofran) d. Promethazine and clonidine (Catapres)

c. Morphine and ondansetron (Zofran) This patient is experiencing postoperative symptoms of moderate to severe pain and nausea and vomiting. Morphine is used postoperatively for this degree of pain, and ondansetron is one of the most effective antiemetics. Atropine is an anticholinergic drug and usually is given preoperatively or perioperatively to prevent bradycardia. Bethanechol is a muscarinic agonist that is used to counter postoperative abdominal distention and urinary retention. Ibuprofen can be used, but it is effective only for mild postoperative pain. Promethazine is less effective as an antiemetic and can be used, but clonidine, marketed as Catapres, is used for postoperative hypertension.

31. A patient with bipolar disorder who wants to minimize the need for drug therapy asks the nurse what else can be done to treat the disorder. The nurse will recommend which measures? (Select all that apply.) a. Electroconvulsive therapy b. Moderate use of alcohol to reduce stress c. Psychotherapy d. Regular sleep and exercise e. Using a chart to monitor mood changes

c. Psychotherapy d. Regular sleep and exercise e. Using a chart to monitor mood changes BPD should be treated with a combination of drugs and adjunctive psychotherapy, because drug therapy alone is not optimal. Other measures, such as regular sleep and exercise and recognizing early symptoms of mood change, help minimize extreme mood swings. Electroconvulsive therapy is effective, but it is not the first-choice treatment; it is reserved for patients who have not responded to other therapies. Avoidance of alcohol is recommended.

28. A patient who has recently begun taking carbamazepine (Tegretol) for bipolar disorder reports having vertigo and headaches. Which action by the nurse is appropriate? a. Ask the provider whether another medication can be used for this patient, because the patient is showing signs of toxicity. b. Contact the provider to request a complete blood count (CBC) to evaluate for other, more serious side effects. c. Reassure the patient that these effects occur early in treatment and will resolve over time. d. Review the patient's chart for cytochrome P450 enzymes to see whether an increased dose is needed.

c. Reassure the patient that these effects occur early in treatment and will resolve over time. Carbamazepine can cause several neurologic side effects early in treatment, including vertigo and headaches. These resolve with continued drug use. These side effects are not related to drug toxicity. A CBC should be obtained at baseline and periodically thereafter. Carbamazepine can cause changes in hematologic laboratory values. The side effects reported by this patient are not associated with hematologic side effects. Carbamazepine induces cytochrome P450 enzymes and can accelerate its own metabolism, which would reduce the amount of drug and decrease side effects, so an increased dose is not appropriate.

69. A young woman with migraine headaches who has recently begun taking sumatriptan (Imitrex) calls the nurse to report a sensation of chest and arm heaviness. The nurse questions the patient and determines that she feels pressure and not pain. What will the nurse do? a. Ask the patient about any history of hypertension or coronary artery disease. b. Determine whether the patient might be pregnant. c. Reassure the patient that this is a transient, reversible side effect of sumatriptan. d. Tell the patient to stop taking the medication immediately.

c. Reassure the patient that this is a transient, reversible side effect of sumatriptan. Some patients taking sumatriptan experience unpleasant chest symptoms, usually described as "heavy arms" or "chest pressure." These symptoms are transient and are not related to heart disease. Patients experiencing angina-like pain when taking sumatriptan, as a result of coronary vasospasm, should be asked about hypertension or coronary artery disease (CAD); they should not take sumatriptan if they have a history of either of these. The symptoms this patient describes are not characteristic of pregnancy. There is no need to stop taking the medication.

49. A patient with schizophrenia has been taking an antipsychotic drug for several days. The nurse enters the patient's room to administer a dose of haloperidol (Haldol) and finds the patient having facial spasms. The patient's head is thrust back, and the patient is unable to speak. What will the nurse do? a. Administer the haloperidol as ordered. b. Discuss increasing the haloperidol dose with the provider. c. Request an order to give diphenhydramine. d. Request an order to give levodopa.

c. Request an order to give diphenhydramine. An early reaction to antipsychotic drugs is acute dystonia. Initial treatment consists of an anticholinergic medication, such as diphenhydramine. Administering more antipsychotic medication would increase the symptoms and could be life threatening. Levodopa is not given for extrapyramidal symptoms, because it could counteract the beneficial effects of antipsychotic treatment.

44. A patient taking an MAOI is seen in the clinic with a blood pressure of 170/96 mm Hg. What will the nurse ask this patient? a. Whether any antihypertensive medications are used b. Whether the patient drinks grapefruit juice c. To list all foods eaten that day d. Whether SSRIs are taken in addition to the MAOI

c. To list all foods eaten that day Patients taking an MAOI should be counseled to follow strict dietary restrictions and to avoid all foods containing tyramine. Patients who consume such foods when taking an MAOI experience a hypertensive episode. Antihypertensive medications, given with an MAOI, will result in hypotension. Grapefruit juice does not alter the metabolism of an MAOI. SSRIs and MAOIs, when administered together, cause a serotonin syndrome.

14. A patient in the emergency department is given intravenous diazepam (Valium) for seizures. When the seizures stop, the nurse notes that the patient is lethargic and confused and has a respiratory rate of 10 breaths per minute. The nurse will expect to administer which of the following? a. Flumazenil (Romazicon) b. Gastric lavage c. Respiratory support d. Toxicology testing

c. Respiratory support When benzodiazepines are administered IV, severe effects, including profound hypotension, respiratory arrest, and cardiac arrest, can occur. Respiration should be monitored, and the airway must be managed if necessary. Flumazenil (Romazicon) is a competitive benzodiazepine receptor antagonist and is used to reverse the sedative effects but may not reverse respiratory depression. Gastric lavage would not be effective, because the benzodiazepine has been given IV. Without further indication of the ingestion of other drugs, toxicology testing is not a priority.

4. A patient reports having occasional periods of tremors, palpitations, nausea, and a sense of fear. To treat this condition, the nurse anticipates the provider will prescribe a drug in which drug class? a. Benzodiazepines b. Monoamine oxidase inhibitors c. Selective serotonin reuptake inhibitors d. Tricyclic antidepressants

c. Selective serotonin reuptake inhibitors This patient is showing characteristics of panic disorder. All three major classes of antidepressants are effective, but selective serotonin reuptake inhibitors are first-line drugs. Benzodiazepines are second-line drugs and are rarely used because of their abuse potential. MAOIs are effective but are difficult to use because of side effects and drug and food interactions. Tricyclic antidepressants are second-line drugs, and their use is recommended only after a trial of at least one SSRI has failed.

57. A patient with schizophrenia has been taking an oral FGA for 1 week. The patient has been taking the drug daily in two divided doses. The individual complains of daytime drowsiness. The patient's family reports a decrease in the person's hostility and anxiety but states that the patient remains antisocial with disordered thinking. What will the nurse tell the patient and the family? a. An increased dose of the drug may be needed. b. Intramuscular dosing may be needed. c. Some symptoms take months to improve. d. The entire dose may be taken at bedtime.

c. Some symptoms take months to improve. When patients begin therapy with antipsychotic medications, some symptoms resolve sooner than others. During the first week, agitation, hostility, anxiety, and tension may resolve, but other symptoms may take several months to improve. It is not necessary to increase the dose in the first week. IM dosing is indicated for patients with severe, acute schizophrenia and for long-term maintenance. Sedation is normal, and once an effective dose has been determined, the entire dose can be taken at bedtime, but not in the initial days of therapy.

13. A patient has been taking high doses of clorazepate (Tranxene) for several months for an anxiety disorder. The nurse assessing the patient observes that the patient is agitated, euphoric, and anxious. What will the nurse do? a. Double-check the chart to make sure the last dose was given. b. Request an order for a longer acting benzodiazepine. c. Suspect a possible paradoxical reaction to the clorazepate. d. Withhold the next dose until a drug level can be drawn.

c. Suspect a possible paradoxical reaction to the clorazepate. Patients taking benzodiazepines for anxiety sometimes develop paradoxical responses to the drug, which include insomnia, excitation, euphoria, heightened anxiety, and rage. A missed dose would trigger withdrawal symptoms, which would include anxiety, insomnia, sweating, tremors, and dizziness. Because this is a paradoxical reaction to the drug, a longer acting drug would make the symptoms worse. This is not caused by overdose, which would manifest as drowsiness, lethargy, and confusion, so a drug level is not warranted.

76. A patient with cancer has been taking an opioid analgesic four times daily for several months and reports needing increased doses for pain. What will the nurse tell the patient? a. PRN dosing of the drug may be more effective. b. The risk of respiratory depression increases over time. c. The patient should discuss increasing the dose with the provider. d. The patient should request the addition of a benzodiazepine to augment pain relief.

c. The patient should discuss increasing the dose with the provider. This patient is developing tolerance, which occurs over time and is evidenced by the need for a larger dose to produce the effect formerly produced by a smaller dose. This patient should be encouraged to request an increased dose. PRN dosing is less effective than scheduled, around-the-clock dosing. The risk of respiratory depression decreases over time as patients develop tolerance to this effect. Benzodiazepines are CNS depressants and should not be given with opioids, as they increase the risk of oversedation.

16. A patient who travels frequently for business reports occasional instances of being unable to fall asleep. The patient tells the nurse that job demands require staying up late and then getting up early for meetings. The nurse expects that the provider will prescribe which medication for this patient? a. Flurazepam b. Trazodone (Desyrel) c. Zaleplon (Sonata) d. Zolpidem (Ambien)

c. Zaleplon (Sonata) Zaleplon (Sonata) works well for people who have trouble falling asleep and, because of its short duration of action, can be taken late at night without causing a hangover or next-day sedation early in the morning. Zolpidem (Ambien) has a longer duration and is a good choice for patients who have difficulty maintaining sleep. Flurazepam has a long duration of action. Trazodone causes daytime grogginess.

66. A prescriber orders sumatriptan (Imitrex) for a patient for a migraine headache. Before administration of this drug, it would be important for the nurse to assess whether the patient: a. has a family history of migraines. b. has taken acetaminophen in the past 3 hours. c. has taken ergotamine in the past 24 hours. d. is allergic to sulfa compounds.

c. has taken ergotamine in the past 24 hours. Sumatriptan, other triptans, and ergot alkaloids all cause vasoconstriction and should not be combined, or excessive and prolonged vasospasm could result. Sumatriptan should not be used within 24 hours of an ergot derivative and another triptan. A family history is important, but it is not vital assessment data as it relates to this scenario. Acetaminophen has no drug-to-drug interaction with sumatriptan. Sulfa is not a component of sumatriptan and therefore is not relevant.

35. A young adult patient has been taking an antidepressant medication for several weeks and reports having increased thoughts of suicide. The nurse questions further and learns that the patient has attempted suicide more than once in the past. The patient identifies a concrete plan for committing suicide. The nurse will contact the provider to discuss: a. changing the medication to another drug class. b. discontinuing the medication immediately. c. hospitalizing the patient for closer monitoring. d. requiring more frequent clinic visits for this patient.

c. hospitalizing the patient for closer monitoring. Patients with depression often think of suicide, and during treatment with antidepressants, these thoughts often increase for a time. Patients whose risk of suicide is especially high should be hospitalized. All antidepressants carry this risk, so changing medication is not recommended. Discontinuing the medication is not recommended. More frequent clinic visits are recommended for patients with a low to moderate risk of suicide.

63. A patient who has occasional migraine headaches tells a nurse that the abortive medication works well, but she would like to do more to prevent the occurrence of these headaches. The nurse will suggest that the patient: a. ask the provider about an adjunct medication, such as prochlorperazine. b. discuss the use of prophylactic medications with the provider. c. keep a headache diary to help determine possible triggers. d. take the abortive medication regularly instead of PRN.

c. keep a headache diary to help determine possible triggers. Keeping a headache diary to try to identify triggers to migraines can be helpful when a patient is trying to prevent them and is the first step in managing headaches. Prochlorperazine is an antiemetic and does not prevent or abort migraine headaches. Prophylactic medications are used when headaches are more frequent. To prevent medication overuse headache, abortive medications should not be used more than 1 to 2 days at a time.

89. A nurse administers atropine to a patient before induction of anesthesia for a surgical procedure. When evaluating the effects of this medication, the nurse will: a. assess for excessive bronchial secretions. b. expect a reduction in the patient's anxiety. c. monitor the patient's heart rate. d. observe for muscle paralysis.

c. monitor the patient's heart rate. Anticholinergic drugs, such as atropine (Sal-Tropine), may be given to reduce the risk of bradycardia during surgery. Atropine can alter bronchial secretions, but the effect would be to reduce them, not increase them. It is not used to reduce anxiety. It does not cause muscle paralysis.

18. A patient with a history of depression and suicidal ideation is taking fluoxetine (Prozac). The patient reports difficulty maintaining sleep and is prescribed secobarbital (Seconal) as a sedative-hypnotic. The nurse preparing this patient for discharge from the hospital will: a. contact the provider to suggest an order for ramelteon (Rozerem). b. instruct the patient to use alcohol in moderation. c. request an order to change to trazodone (Desyrel) for sleep. d. suggest that the patient try alternative remedies for sleep

c. request an order to change to trazodone (Desyrel) for sleep. Trazodone is an atypical antidepressant with sedative actions and can be used to prolong sleep duration. It is useful for treating insomnia related to antidepressants such as Prozac. Ramelteon is not a drug of choice to treat patients who have difficulty maintaining sleep. Patients who are depressed or at increased risk for suicide should not take barbiturates, because overdose can readily cause death. Alcohol is contraindicated when taking barbiturates. Alternative remedies have not shown effectiveness in treating insomnia.

77. A woman in labor receives meperidine (Demerol) for pain. The nurse caring for the infant will observe the infant closely for: a. congenital anomalies. b. excessive crying and sneezing. c. respiratory depression. d. tremors and hyperreflexia.

c. respiratory depression. Use of morphine or other opioids during delivery can cause respiratory depression in the neonate, because the drug crosses the placenta. Infants should be monitored for respiratory depression and receive naloxone if needed. Opioids given during delivery do not contribute to birth defects in the newborn. Excessive crying and sneezing and tremors and hyperreflexia are all signs of neonatal opioid dependence, which occurs with long-term opioid use by the mother during pregnancy and not with short-term use of these drugs during labor.

43. A patient who has fibromyalgia is diagnosed with major depression. The provider orders a TCA. The nurse will teach this patient to: a. avoid foods containing tyrosine. b. consume alcohol in moderation. c. sit or lie down when feeling lightheaded. d. take the medication in the morning.

c. sit or lie down when feeling lightheaded. Orthostatic hypotension can occur with TCAs. Patients should be advised to sit or lie down if they feel lightheaded to prevent falls. Foods containing tyrosine cause adverse effects in patients taking MAOIs. Patients taking TCAs should be counseled to avoid all alcohol. Because TCAs cause sedation, the medication should be taken at bedtime.

87. A nurse is teaching nursing students about inhalation anesthesia and asks, "What is balanced anesthesia?" Which response by a student is correct? a. "An anesthesia that has a brief induction period with a rapid emergence from its effects." b. "An inhalation anesthesia that produces both muscle relaxation and unconsciousness." c. "An anesthesia that provides maximum analgesia with minimal respiratory side effects." d. "An anesthesia that combines other drugs with inhalation anesthesia to produce the desired effects."

d. "An anesthesia that combines other drugs with inhalation anesthesia to produce the desired effects." Balanced anesthesia is the use of a combination of drugs, along with an inhaled anesthetic, to produce effects that cannot safely be accomplished with inhalation anesthesia alone. These adjunct drugs are used to help induce anesthesia, provide muscle relaxation, and increase analgesia so that a lower and safer dose of the inhalation anesthetic can be used. The other three options describe characteristics of an ideal anesthetic, which does not exist; to get these effects, a combination of other drugs must be used.

3. A nurse is preparing a patient to change from taking lorazepam (Ativan) for anxiety to buspirone (Buspar). Which statement by the patient indicates a need for further teaching? a. "I can drink alcohol when taking Buspar, but not grapefruit juice." b. "I may need to use a sedative medication if I experience insomnia." c. "I may not feel the effects of Buspar for a few weeks." d. "I should stop taking the Ativan when I start taking the Buspar."

d. "I should stop taking the Ativan when I start taking the Buspar." Ativan should not be withdrawn quickly; it needs to be tapered to prevent withdrawal symptoms. Moreover, Buspar does not have immediate effects. Because no cross-dependence occurs with these two medications, they may be taken together while the benzodiazepine is tapered. Because Buspar does not have sedative effects, patients can consume alcohol without increasing sedation. Levels of Buspar can be increased by grapefruit juice, leading to drowsiness and a feeling of dysphoria. Buspar can cause nervousness and excitement and does not have sedative effects, so patients with insomnia must use a sedative. Buspar does not have immediate effects.

93. A patient will receive intravenous midazolam (Versed) combined with fentanyl while undergoing an endoscopic procedure. The nurse is explaining the reasons for this to a nursing student before the procedure. Which statement by the student indicates understanding of the teaching? a. "The patient may appear anxious and restless during the procedure." b. "The patient will be unconscious during the procedure." c. "The patient will not need cardiorespiratory support during the procedure." d. "The patient will not remember the procedure."

d. "The patient will not remember the procedure." Midazolam, combined with an opioid analgesic, is used for conscious sedation for minor surgeries and endoscopic procedures. It does not cause anesthesia during this state, which is characterized by sedation, analgesia, amnesia, and lack of anxiety. The patient will not remember the events even though the person will be able to respond to commands during the procedure. The patient will appear unperturbed and passive and not anxious or restless. The patient is sedated, not unconscious. Midazolam can cause dangerous cardiorespiratory effects, including respiratory depression and respiratory and cardiac arrest

10. A hospitalized patient who is given one dose of flurazepam continues to show drowsiness the next day. A nursing student asks the nurse the reason for this, because the drug's half-life is only 2 to 3 hours. Which response by the nurse is correct? a. "Benzodiazepines commonly cause residual effects lasting into the day after the dose is given." b. "The patient is having a paradoxical reaction to this medication." c. "This patient must have developed a previous tolerance to benzodiazepines." d. "When this drug is metabolized, the resulting compound has longer lasting effects."

d. "When this drug is metabolized, the resulting compound has longer lasting effects." Flurazepam has a half-life of 2 to 3 hours; however, its metabolite has a long half-life, so giving the drug results in long-lasting effects. Barbiturates, not benzodiazepines, are commonly associated with residual, or hangover, effects. A paradoxical reaction to a sedative would manifest as insomnia, euphoria, and excitation, not drowsiness. Tolerance means that the patient would need increased amounts of a drug to get the desired effects and would not have prolonged effects of the medication.

65. A patient who uses ergotamine (Ergomar) to abort migraine headaches reports nausea and vomiting with the headaches. What will the nurse tell the patient? a. "Ask your provider about using another antimigraine medication." b. "Nausea and vomiting are signs of ergotamine toxicity." c. "These symptoms occur with migraine headaches and will diminish over time." d. "You should talk to your provider about an adjunct antiemetic medication."

d. "You should talk to your provider about an adjunct antiemetic medication." Nausea and vomiting, which occur with migraines, can increase with ergotamine use, because the drug can stimulate the chemoreceptor trigger zone. Patients should be treated with metoclopramide or a phenothiazine antiemetic. It is not necessary to change antimigraine medications at this time. Nausea and vomiting do not indicate ergotamine toxicity. These symptoms will not diminish over time.

1. An agitated, extremely anxious patient is brought to the emergency department. The prescriber orders a benzodiazepine. The nurse understands that benzodiazepines are used in this clinical situation based on which principle? a. Benzodiazepines have a very short half-life. b. Physical dependence is not a risk when taking benzodiazepines. c. Benzodiazepines are known to cure generalized anxiety. d. Benzodiazepines have a rapid onset of action.

d. Benzodiazepines have a rapid onset of action. The patient is clearly in a state of extreme, uncontrolled anxiety. Benzodiazepines are the drug of choice for acute episodes of anxiety because of their rapid onset of action. Benzodiazepines do not have a very short half-life. Benzodiazepines are associated with physical dependence. Benzodiazepines do not cure generalized anxiety, nor do any other drugs.

91. A patient in the postanesthesia recovery unit received ketamine (Ketalar) for right open reduction internal fixation surgery. What drug would be beneficial as a premedication to help minimize adverse reactions? a. Thiopental sodium (Pentothal) b. Sevoflurane (Ultane) c. Atropine (Sal-Tropine) d. Diazepam (Valium)

d. Diazepam (Valium) To minimize the effects of ketamine, the patient should be premedicated with diazepam or midazolam to reduce the risk of an adverse reaction. Thiopental sodium would further sedate the patient. Sevoflurane would further complicate sedation and would not be indicated. Atropine would not reduce the risk of an adverse reaction.

74. A patient who has had abdominal surgery has been receiving morphine sulfate via a patient-controlled analgesia (PCA) pump. The nurse assesses the patient and notes that the patient's pupils are dilated and that the patient is drowsy and lethargic. The patient's heart rate is 84 beats per minute, the respiratory rate is 10 breaths per minute, and the blood pressure is 90/50 mm Hg. What will the nurse do? a. Discuss possible opiate dependence with the patient's provider. b. Encourage the patient to turn over and cough and take deep breaths. c. Note the effectiveness of the analgesia in the patient's chart. d. Prepare to administer naloxone and possibly ventilatory support.

d. Prepare to administer naloxone and possibly ventilatory support. Opioid toxicity is characterized by coma, respiratory depression, and pinpoint pupils. Although pupils are constricted initially, they may dilate as hypoxia progresses, which also causes blood pressure to drop. This patient has a respiratory rate of fewer than 12 breaths per minute, dilated pupils, and low blood pressure; the patient also is showing signs of central nervous system (CNS) depression. The nurse should prepare to give naloxone and should watch the patient closely for respiratory collapse. Patients with opioid dependence show withdrawal symptoms when the drug is discontinued. When postoperative patients have adequate analgesia without serious side effects, encouraging patients to turn, cough, and breathe deeply is appropriate. This patient is probably relatively pain free, but providing emergency treatment is the priority.

53. A patient is taking an FGA for schizophrenia. The nurse notes that the patient has trouble speaking and chewing and observes slow, wormlike movements of the patient's tongue. The nurse recognizes which adverse effect in this patient? a. Acute dystonia b. Akathisia c. Parkinsonism d. Tardive dyskinesia

d. Tardive dyskinesia Tardive dyskinesia can occur in patients during long-term therapy with FGAs. This patient shows signs of this adverse effect. Acute dystonia is characterized by severe spasm of muscles in the face, tongue, neck, or back and by opisthotonus. Akathisia is characterized by constant motion. Parkinsonism is characterized by bradykinesia, drooling, tremor, rigidity, and a shuffling gait.

70. A patient who has a history of asthma experiences three or four migraine headaches each month. The patient uses sumatriptan (Imitrex) as an abortive medication and has developed medication overuse headaches. The patient asks the nurse what can be done to prevent migraines. The nurse will suggest that the patient discuss which preventive medication with the provider? a. Botulinum toxin b. Meperidine (Demerol) c. Timolol d. Topiramate (Topamax)

d. Topiramate (Topamax) Topiramate can be used for migraine prophylaxis, and its benefits appear equal to those of the first line beta blockers. Botulinum toxin can be used for migraine prophylaxis in patients who have 15 or more headaches a month. Meperidine may be used as abortive therapy but has addictive potential. Timolol is a beta blocker; this patient has asthma, and because beta blockers cause bronchoconstriction, these agents are not recommended.

9. A patient who has been using secobarbital for several months to treat insomnia tells the nurse that the prescriber has said the prescription will be changed to temazepam (Restoril) because it is safer. The patient asks why this agent is safer. The nurse is correct in telling the patient that temazepam: a. does not depress the central nervous system. b. shows no respiratory depression, even in toxic doses. c. mimics the actions of a central nervous system inhibitory neurotransmitter. d. only potentiates the action of endogenous gamma-aminobutyric acid (GABA).

d. only potentiates the action of endogenous gamma-aminobutyric acid (GABA). Benzodiazepines potentiate the actions of GABA, and because the amount of GABA in the CNS is finite, these drugs' depressive effect on the CNS is limited. Benzodiazepines depress the CNS but not to the extent that barbiturates do. Benzodiazepines are weak respiratory depressants at therapeutic doses and moderate respiratory depressants at toxic doses. Barbiturates mimic GABA; therefore, because they produce CNS depression, this effect is limited only by the amount of barbiturate administered.

94. A patient receives a neuromuscular blocking agent as an adjunct to inhalation anesthesia. When caring for this patient, it is important for the nurse to remember that neuromuscular blocking agents: a. cause vagal slowing of the heart. b. increase the required dose of inhalation anesthetics. c. increase the depth of unconsciousness. d. prevent contraction of the diaphragm.

d. prevent contraction of the diaphragm. Neuromuscular blocking agents enhance skeletal muscle relaxation so that the dose of inhalation anesthetics can be reduced to a safer amount. Because these drugs prevent contraction of all skeletal muscles, including the diaphragm, mechanical ventilation is required to support respiration. These agents do not cause vagal slowing of the heart. They reduce the required dose of inhalation agents. They do not affect the level of consciousness

100. A nurse is assisting a physician who is preparing to suture a superficial laceration on a patient's leg. The physician asks the nurse to draw up lidocaine with epinephrine. The nurse understands that epinephrine is used with the lidocaine to: a. allow more systemic absorption to speed up metabolism of the lidocaine. b. increase the rate of absorption of the lidocaine. c. improve perfusion by increasing blood flow to the area. d. prolong anesthetic effects and reduce the risk of systemic toxicity from lidocaine.

d. prolong anesthetic effects and reduce the risk of systemic toxicity from lidocaine. Epinephrine causes vasoconstriction, which reduces local blood flow and delays systemic absorption of lidocaine, which prolongs local anesthetic effects and reduces the risk of systemic toxicity. Epinephrine slows the rate of absorption. Epinephrine delays systemic absorption of lidocaine, so metabolism is slowed and the effects are prolonged in the periphery. Epinephrine does not increase local blood flow.

38. A patient who has been taking a monoamine oxidase inhibitor (MAOI) for depression for several months tells the provider that the medication has not helped with symptoms. The provider plans to switch the patient to an SSRI. The nurse will teach this patient to: a. start taking the SSRI and stop the MAOI when symptoms improve. b. start taking the SSRI and then gradually withdraw the MAOI. c. stop taking the MAOI and wait 5 weeks before starting the SSRI. d. stop taking the MAOI 2 weeks before starting the SSRI.

d. stop taking the MAOI 2 weeks before starting the SSRI. MAOIs increase 5-HT availability, thus greatly increasing the risk of serotonin syndrome. MAOIs should be withdrawn at least 14 days before an SSRI is started. An SSRI should never be given at the same time as an MAOI. It is not necessary to wait 5 weeks before starting an SSRI.

51. A patient taking a first-generation antipsychotic (FGA) medication develops severe parkinsonism and is treated with amantadine (Symmetrel). The amantadine is withdrawn 2 months later, and the parkinsonism returns. The nurse will expect the provider to: a. give anticholinergic medications. b. make a diagnosis of idiopathic parkinsonism. c. resume the amantadine indefinitely. d. try a second-generation antipsychotic (SGA).

d. try a second-generation antipsychotic (SGA). Neuroleptic-induced parkinsonism is treated with some of the same drugs used for idiopathic parkinsonism, such as amantadine. If parkinsonism is severe, switching to an SGA may help, because the risk of parkinsonism is much lower with these drugs. An anticholinergic medication may be used initially. A recurrence of parkinsonism when the drug is withdrawn does not indicate idiopathic parkinsonism. These drugs should not be used indefinitely

79. A patient is brought to the emergency department by friends, who report finding the patient difficult to awaken. The friends report removing two fentanyl transdermal patches from the patient's arm. On admission to the emergency department, the patient has pinpoint pupils and a respiratory rate of 6 breaths per minute. A few minutes after administration of naloxone, the respiratory rate is 8 breaths per minute and the patient's pupils are dilated. The nurse recognizes these symptoms as signs of: a. a mild opioid overdose. b. decreased opioid drug levels. c. improved ventilation. d. worsening hypoxia.

d. worsening hypoxia. The classic triad of symptoms of opioid overdose are coma, respiratory depression, and pinpoint pupils. The pupils may dilate as hypoxia worsens, and this symptom, along with continued respiratory depression (fewer than 12 breaths per minute), indicates worsening hypoxia. Fentanyl is a strong opioid, so this is not likely to be a mild overdose, because the patient was wearing two patches. Fentanyl continues to be absorbed even after the patches are removed because of residual drug in the skin, so the drug levels are not likely to be decreasing. The patient does not have improved ventilation, because the respiratory rate is still fewer than 12 breaths per minute.

80. A patient with chronic pain has been receiving morphine sulfate but now has decreased pain. The prescriber changes the medication to pentazocine (Talwin). The nurse will monitor the patient for: a. euphoria. b. hypotension. c. respiratory depression. d. yawning and sweating

d. yawning and sweating Pentazocine is an agonist-antagonist opioid, and when given to a patient who is physically dependent on morphine, it can precipitate withdrawal. Yawning and sweating are early signs of opioid withdrawal. Pentazocine does not produce euphoria, hypotension, or respiratory depression.


Ensembles d'études connexes

Legal environment of business Exam 2

View Set

The Presidency of George W. Bush

View Set

Cardiovascular System (Mastering A&P)

View Set

SY0-401:1 TS Quiz Network Security

View Set

Taylor Review Questions - Wounds/Skin Integrity

View Set